You are on page 1of 121

1. Your adult patient has partial- and full- ● D.

Cyanide
thickness burn injuries to his anterior chest, Rationale: Answer: C. If a patient has
anterior abdomen, and entire right arm. ingested methyl salicylates, his breath will
Using the rule of nines, the percent of total have a wintergreen odor.
body surface area involved can be
estimated at: 1. A 65 year old male patient has
● A. 18%. experienced full-thickness electrical burns
● B. 27%. on the legs and arms. As the nurse you
● C. 45%. know this patient is at risk for the following:
● D. 50%. Select all that apply:
Rationale: Answer: B. The anterior chest A. Acute kidney injury
and abdomen constitute 18% of the BSA, B. Dysrhythmia
and the entire right arm is 9%, for a total of C. Iceberg effect
27%. D. Hypernatremia
E. Bone fractures
2. A patient admitted to the ED is suspected F. Fluid volume overload
of taking an overdose of atropine. Which The answers are A, B, C, and E. Electric
clinical finding would you look for? burns are due to an electrical current
● A. Kussmaul respirations passing through the body that leads to
● B. Diarrhea and nausea and damage to the skin but also the muscles
vomiting and bones that are underneath the skin.
● C. Hot, dry skin and dilated pupils The patient is at risk for AKI (acute kidney
● D. Extrapyramidal tremors injury) because when the muscles become
Rationale: Answer: C. A patient who has affected they release myoglobin and the red
overdosed on atropine will have blood cells release hemoglobin in the blood,
flushed, dry skin and dilated pupils. which can collect in the kidneys leading to
injury. In addition, the heart's electrical
3. Your patient has a core body temperature system can become damaged leading to
of 80° F (26.7° C). Which classification of dysrhythmia. The iceberg effect can present
hypothermia is this? as well because the extent of damage is not
● A. Low clearly visible on the skin (there can be
● B. Mild severe damage underneath). In addition, if
● C. Moderate the electrical current is strong enough it can
● D. Severe lead to bone fractures (specifically cervical
Rationale: Answer: D. Severe hypothermia spine injuries) due to the severe contraction
is a core body temperature of of the muscles involved.
less than 82.4° F ( 28° C).

4. An unconscious patient is admitted to the 2. True or False: A patient who experiences


ED with a very strong wintergreen odor on an alkali chemical burn is easier to treat
his breath. What might he have ingested? because the skin will neutralize the
● A. Ethanol chemical rather than with an acidic chemical
● B. Acetone burn.
● C. Methyl salicylates True
False that is burned (use the rule of nine to
False: Alkali burns are harder to treat than calculate), depth of the burn, age, location
acidic chemical burns because the skin will of the burn, and patient’s medical history.
neutralize the acidic burn. The patient in option B has 40.5% TSBA
burned (option A 27%, C: 4.5%, D: 9%).
Remember that the higher the total of the
3. As the nurse providing care to a patient body surface area that is burned the higher
who experienced a full-thickness electrical the risk of complications due to an increase
burn you know to monitor the patient's urine in capillary permeability (swelling,
for: hypovolemic shock etc.). In addition, the
A. Hemoglobin and myoglobin location of the burn is a major issue with the
B. Free iron and white blood cells patient in option B. The burns are on the
C. Protein and red blood cells head and neck and front and back of the
D. Potassium and Urea torso. Therefore, with head and neck burns
The answer is A. Patients who've always think about respiratory issues
experienced a severe electrical burn or full- because the airway can become
thickness burns are at risk for acute kidney compromised due to swelling or an
injury. This is because the muscles can inhalation injury. And with torso burns that
experience damage from the electrical are on the front and back, the patient is at
current leading them to release myoglobin. risk for circumferential burns that can lead
In addition, the red blood cells will release to further respiratory compromise. The other
hemoglobin. These substances will collect options have burns that are isolated.
in the kidneys leading to acute tubular
necrosis (hence leading to AKI). Therefore,
the nurse should monitor the patient's urine 5. The _____________ layer of the skin
for these substances. helps regulate our body temperature.
A. Epidermis
B. Dermis
4. Select the patient below who is at MOST C. Hypodermis
risk for complications following a burn: D. Fascia
A. A 42 year old male with partial-thickness The answer is C. This layer contains fatty
burns on the front of the right and left arms tissue, veins, arteries, nerves and helps
and legs. insulate the muscles, bones, organs and
B. A 25 year old female with partial- helps REGULATE our body temperature.
thickness burns on the front of the head and
neck and front and back of the torso.
C. A 36 year old male with full-thickness 6. You receive a patient who has
burns on the front of the left arm. experienced a burn on the right leg. You
D. A 10 year old with superficial burns on note the burn contains small blisters and is
the right leg. extremely pinkish red and shiny/moist. The
The answer is B. When thinking about patient reports severe pain. You document
which patient will have the MOST this burn as:
complications following a burn think about: A. 1st Degree (superficial)
percentage of the total body surface area B. 2nd Degree (partial-thickness)
C. 3rd Degree (full-thickness) the flow rate during the FIRST 8 hours
D. 4th Degree (deep full-thickness) (mL/hr) based on the total you calculated?
The answer is B. These are the classic A. 921 mL/hr
characteristics of a 2nd degree (partial- B. 938 mL/hr
thickness) burn. C. 158 mL/hr
D. 789 mL/hr
The answer is A: 921 mL/hr....First calculate
7. Based on the depth of the burn in figure 1 the total amount of fluid needed with the
(picture is above), you would expect to find: formula: Total Amount of LR = 4 mL x BSA
A. report of sensation to only pressure % x pt's weight in kg. The pt's weight 63 kg.
B. blanching BSA percentage: 58.5%...Front and back of
C. anesthetization to feeling right and left leg (36%), front of right arm
D. extreme pain (4.5%), anterior trunk (18%) which equals
The answer is C. This is a 3rd degree to 4th 58.5%. ......4 x 58.5 x 63 = 14,742
degree burn (full-thickness) and the nerves mL......Remember during the FIRST 8 hours
that detect pain are destroyed. The patient 1/2 of the solution is infused, which will be
would have no feeling or experiences an 14,742 divided by 2 = 7371 mL......Hourly
extreme decrease sensation to pain. Rate: 7371 divide by 8 equals 921 mL/hr

8. A 58 year old female patient has 10. A patient has a burn on the back of the
superficial partial-thickness burns to the torso that is extremely red and painful but
anterior head and neck, front and back of no blisters are present. When you pressed
the left arm, front of the right arm, posterior on the skin it blanches. You document this
trunk, front and back of the right leg, and as a:
back of the left leg. Using the Rule of Nines, A. 1st degree (superficial) burn
calculate the total body surface area B. 2nd degree (partial-thickness) burn
percentage that is burned? C. 3rd degree (full-thickness) burn
A. 63% D. 4th degree (deep full-thickness) burn
B. 81% The answer is A. These are the classic
C. 72% characteristics of a 1st degree, superficial
D. 54% burn.
The answer is A. Anterior head and neck
(4.5%), front and back of the left arm (9%),
front of the right arm (4.5%), posterior trunk 11. A patient has experienced full-thickness
(18%), front and back of the right leg (18%), burns to the face and neck. As the nurse it
back of the left leg (9%) which equals 63%. is priority to:
A. Prevent hypothermia
B. Assess the blood pressure
9. A 30 year old female patient has deep C. Assess the airway
partial thickness burns on the front and back D. Prevent infection
of the right and left leg, front of right arm, The answer is C. Due to the location of the
and anterior trunk. The patient weighs 63 burns (face and neck), the patient is at
kg. Use the Parkland Burn Formula: What is major risk for respiratory issues due to
damage to the upper airways and the risk of elevate the extremity ABOVE heart level to
an inhalation injury. decrease swelling and assess distal pulses
in the extremity to confirm circulation is
present.
12. A patient is in the acute phase of burn
management. The patient experienced full-
thickness burns to the perineum and sacral 14. A patient arrives to the ER with full-
area of the body. In the patient’s plan of thickness burns on the front and back of the
care, which nursing diagnosis is priority at torso and neck. The patient has no spinal
this time? injuries but is disoriented and coughing up
A. Impaired skin integrity black sooty sputum. Vital signs are: oxygen
B. Risk for fluid volume overload saturation 63%, heart rate 145, blood
C. Risk for infection pressure 80/56, and respiratory rate 39. As
D. Ineffective coping the nurse you will:
The answer is C. The patient is now in the A. Place the patient in High Fowler’s
acute phase where fluid resuscitation was positon.
successful and ends with wound closure. B. Prep the patient for escharotomy.
Therefore, during this stage diuresis occurs C. Prep the patient for fasciotomy.
(so fluid volume deficient could occur NOT D. Prep the patient for intubation.
overload) and INFECTION. The location of E. Place a pillow under the patient’s neck.
the burns increases the risk of infection F. Obtain IV access at two sites.
because these areas naturally harbor G. Restrict fluids.
bacteria. Therefore, this takes priority The answers are A, B, D, and F. After
because during this phase wound healing is reading this scenario the location of the
promoted. burns and the patient’s presentation should
be jumping out at you. The patient is at risk
for circumferential burns due to the location
13. The nurse notes a patient has full- of the burns and the depth (full-
thickness circumferential burns on the right thickness….will have eschar present that
leg. The nurse would: select all that apply will restrict circulation or here in this
A. Place cold compressions on the burn and example the ability of the patient to breathe
elevate the right leg below the heart level in and out). Based on the patient's VS, we
B. Assess the distal pulses in the right see that the respiratory effort is
extremity compromised majorly AND that there is a
C. Elevate the right leg above the heart risk of inhalation injury since the patient is
level coughing up black sooty sputum. Therefore,
D. Place gauze securely around the leg to the nurse should place the patient in high
prevent infection Fowler’s position to help with respiratory
The answer is B and C. The patient has effort (unless contraindicated with spinal
burns that completely surround the front and injuries), prep the patient for escharotomy
back of the right leg. This can lead to (this will cut the eschar and help relieve
compartment syndrome where the edema pressure and allow for breathing) and prep
from the burn compromises circulation to for intubation to help with the respiratory
the distal extremity. The nurse should distress. In addition, obtain IV access in at
least two sites for fluid
replacement….remember the first 24 hours
after a burn a patient is at risk for 17. A patient experienced a full-thickness
hypovolemic shock. burn 72 hours ago. The patient's vital signs
are within normal limits and urinary output is
50 mL/hr. This is known as what phase of
15. A patient arrives to the ER due to burn management?
experiencing burns while in an enclosed A. Emergent
warehouse. Which assessment findings B. Acute
below demonstrate the patient may have C. Rehabilitative
experienced an inhalation injury? The answer is B. This phase starts when
A. Carbonaceous sputum capillary permeability has returned to
B. Hair singeing on the head and nose normal and the patient's vitals are within
C. Lhermitte's Sign normal limits and ends with wound closure.
D. Bright red lips The phase after this is rehabilitative.
E. Hoarse voice
F. Tachycardia
The answers are A, B, D, E, and F. These 18. What are some patient priorities during
are all signs of a possible inhalation injury. the emergent phase of burn management?
Bright red lips and tachycardia are present A. Fluid volume
in carbon monoxide poisoning as well. B. Respiratory status
C. Psychosocial
D. Wound closure
16. You're providing education to a group of E. Nutrition
local firefighters about carbon monoxide The answer is A and B. This phase starts
poisoning. Which statement is correct about from the onset of the burn and ends with the
the pathophysiology regarding this restoration of capillary permeability. Wound
condition? closure, and nutrition would be during the
A. "Patients are most likely to present with acute phase, and would continue into the
cyanosis around the lips and face." rehabilitative phase. Psychosocial would be
B. "In this condition, carbon monoxide binds in the rehab phase.
to the hemoglobin of the red blood cell
leading to a decrease in the ability of the
hemoglobin to carry oxygen to the body." 19. During the emergent phase of burn
C. "Carbon monoxide poisoning leads to a management, you would expect the
hyperoxygenated state, which causes following lab values:
hypercapnia." A. Low sodium, low potassium, high
D. "Carbon monoxide binds to the glucose, low hematocrit
hemoglobin of the red blood cell and B. High sodium, low potassium, low
prevents the transport of carbon dioxide out glucose, high hematocrit
of the blood, which leads to poisoning." C. High sodium, high potassium, high
The answer is B. This is the only correct glucose, low hematocrit
statement about carbon monoxide D. Low sodium, high potassium, high
poisoning. glucose, high hematocrit
The answer is D. Think about the increase A. Swelling and pain on the area distal to
in the capillary permeability that happens the burn
with severe burns, which causes the plasma B. Burning, gnawing sensation pain in the
to leave the intravascular system and enter stomach and vomiting
the interstitial tissue: Low sodium..why: C. Dark red or gray sores on the soles of
sodium leaves with the plasma to the the feet
interstitial tissue and drops the levels in the D. Difficulty swallowing and gagging
blood; High potassium...why? damaged The answer is B. This is a type of ulcer that
cells lysis and leak potassium which occurs in the stomach, duodenum, due to a
increases the leave in the blood; high high amount of stress on the body from a
glucose...why? stress response leads the burn. The blood supply to the factors that
liver to release glycogen and this increases help protect the stomach lining from gastric
levels; high hematocrit...why? when the erosion decreases and this allows for ulcers
plasma leaves the intravascular system (the to form.
fluid) it causes the blood to become more
concentrated so hematocrit increases (this
will decrease when the patient's fluid is 22. During the acute phase of burn
replaced). management, what is the best diet for a
patient who has experienced severe burns?
A. High fiber, low calories, and low protein
20. A patient is receiving IV Lactated B. High calorie, high protein and
Ringers 950 mL/hr post 18 hours after a carbohydrate
receiving a severe burn. The patient urinary C. High potassium, high carbohydrate, and
output is 20 mL/hr. As the nurse your next low protein
nursing action is to: D. Low sodium, high protein, and restrict
A. Increase the IV fluids fluids to 1 liter per day
B. Continue to monitor the patient The answer is B. This type of diet promotes
C. Decrease the IV fluids wound healing and meets the caloric
D. Notify the physician of this finding demands of the body.
The answer is D. The patient's urinary
output is too low and needs more fluids. It
should be at least 30 mL/hr. Therefore, the 23. You're assisting the nursing assistant
nurse must notify the physician for further with repositioning a patient with full-
orders. The nurse can NOT increase or thickness burns on the neck. Which action
decrease IV fluids without a physician's by the nursing assistant requires you to
order. intervene?
A. The nursing assistant elevates the head
of the bed above 30 degrees.
21. A patient who is being treated for partial B. The nursing assistant places a pillow
thickness burns on 60% of the body is now under the patient's head.
in the acute phase of burn management. C. The nursing assistant places rolled
The nurse assesses the patient for a towels under the patient's shoulders.
possible Curling's Ulcer. What signs and D. The nursing assistant covers the patient
symptoms can present with this condition? with sterile linens.
The answer is B. If a patient has severe 26. Your patient with severe burns is due to
burns to the neck (head as well) a pillow have a dressing change. You will pre-
should NOT be used under the head medicate the patient prior to the dressing
because this can cause wound contractions. change. The patient has standing orders for
Instead rolled towels should be placed all the medications below. Which medication
under the shoulders. is best for this patient?
A. IM morphine
B. PO morphine
24. A patient has full-thickness burns on the C. IV morphine
front and back of both arm and hands. It is D. Subq morphine
nursing priority to: The answer is C. The best route that is
A. Elevate and extend the extremities predictable and easily absorbed is via the IV
B. Elevate and flex the extremities route in burn victims.
C. Keep extremities below heart level and
extended
D. Keep extremities level with the heart 27. After receiving report on a patient
level and flexed receiving treatment for severe burns, you
The answer is A. This position will decrease perform your head-to-toe assessment. On
edema, which will help prevent arrival to the patient's room you note the
compartment syndrome. room temperature to be 75'F. You will:
A. Decrease the temperature by 5-10
degrees to prevent hyperthermia.
25. A patient has an emergency B. Leave the temperature setting.
escharotomy performed on the right leg. C. Increase the temperature to a minimum
The patient has full-thickness of 85'F.
circumferential burns on the leg. Which The answer is C. Patients with severe burns
finding below demonstrates the procedure can NOT regulate their temperature and are
was successful? at risk for hypothermia. The room
A. The patient can move the extremity. temperature should be a minimum of 85'F.
B. The right foot's capillary refill is less than
2 seconds.
C. The patient reports a new sensation of 28. You are about to provide care to a
extreme pain. patient with severe burns. You will don:
D. The patient has a positive babinski reflex. A. gloves
The answer is B. Escharotomy is performed B. goggles
when a full-thickness burn, due to eschar C. gown
(which is burned tissue that is hard), is D. N-95 mask
compromising blood flow to the distal E. surgical mask
extremity. The eschar is cut and this F. shoe covers
relieves pressure and allows blood to flow to G. hair cover
the extremity. The answer is A, C, E, F, and G. Before
providing care to a patient with severe burns
the nurse would want to wear protective
isolation apparel like: gloves, gown, surgical
mask, shoe covers, and hair cover. This
protects the patient from potential infection. 2. A 68 year old male patient has partial
thickness burns to the front and back of the
right and left leg, front of right arm, and
29. While collecting a medical history on a anterior trunk. Using the Rule of Nines,
patient who experienced a severe burn, calculate the total body surface area
which statement by the patient's family percentage that is burned?
member requires nursing intervention? A. 40.5%
A. "He takes medication for glaucoma". B. 49.5%
B. "I think it has been 10 years or more C. 58.5%
since he had a tetanus shot." D. 67.5%
C. "He was told he had COPD last year." The answer is C. Front and back of right
D. "He smokes 2 packs of cigarettes a day." and left leg (36%), front of right arm (4.5%),
The answer is B. Patients who have had anterior trunk (18%) which equals 58.5%.
burns need a tetanus shot if they have not
had a vaccine within the past 5 to 10 years.
3. A 35 year old male patient has full
thickness burns to the anterior and posterior
30. A patient is presenting with bright red head and neck, front of left leg, and
lips, headache, and nausea. The physician perineum. Using the Rule of Nines,
suspects carbon monoxide poisoning. As calculate the total body surface area
the nurse, you know the patient needs: percentage that is burned?
A. Oxygen nasal cannula 5-6 Liters A. 37%
B. 100% oxygen via non-rebreather mask B. 14.5%
C. Continuous Bipap C. 29%
D. Venturi mask 6 L oxygen D. 19%
The answer is B. This is the treatment for The answer is D. Anterior and posterior
carbon monoxide poisoning. head and neck (9%), front of left leg (9%),
perineum (1%) which equals 19%.
1. A 25 year old female patient has
sustained burns to the back of the right arm,
posterior trunk, front of the left leg, anterior 4. A 66 year old female patient has deep
head and neck, and perineum. Using the partial-thickness burns to both of the legs on
Rule of Nines, calculate the total body the back, front and back of the trunk, both
surface area percentage that is burned? arms on the front and back, and front and
A. 46% back of the head and neck. Using the Rule
B. 37% of Nines, calculate the total body surface
C. 36% area percentage that is burned?
D. 28% A. 72%
The answer is B. Back of right arm (4.5%), B. 63%
posterior trunk (18%), front of left leg (9%), C. 81%
anterior head and neck (4.5%) and D. 45%
perineum (1%) which equals 37%. The answer is C. Both of the legs on the
back (18%), front and back of the trunk
(36%), both arms on the front and back
(18%), front and back of the head and neck
(9%) which equals 81%.

5. A 58 year old female patient has


superficial partial-thickness burns to the
anterior head and neck, front and back of
the left arm, front of the right arm, posterior
trunk, front and back of the right leg, and
back of the left leg. Using the Rule of Nines,
calculate the total body surface area
percentage that is burned?
A. 63%
B. 81%
C. 72%
D. 54%
The answer is A. Anterior head and neck
(4.5%), front and back of the left arm (9%),
front of the right arm (4.5%), posterior trunk
(18%), front and back of the right leg (18%),
back of the left leg (9%) which equals 63%.

6. A 35 year old male patient has superficial


partial-thickness burns to the anterior right
arm, posterior left leg, and anterior head
and neck. Using the Rule of Nines, calculate
the total body surface area percentage that
1. A 45 year old female patient has
is burned?
superficial partial thickness burns on the
A. 36%
posterior head and neck, front of the left
B. 9%
arm, front and back of the right arm,
C. 18%
posterior trunk, front and back of the left leg,
D. 29%
and back of right leg. The patient weighs 91
The answer is C. Anterior right arm (4.5%),
kg. Use the Parkland Burn Formula to
posterior left leg (9%), and anterior head
calculate the total amount of Lactated
and neck (4.5%) which equals 18%.
Ringers that will be given over the next 24
hours?
A. 22,932 mL
B. 26,208 mL
C. 16,380 mL
D. 12,238 mL
The answer is A: 22,932 mL. Formula: Total
Amount of LR = 4 mL x BSA % x pt's weight
in kg. Pt's weight 91 kg. BSA percentage: The answer is A: 921 mL/hr....First calculate
63%... posterior head and neck (4.5%), front the total amount of fluid needed with the
of the left arm (4.5%), front and back of the formula: Total Amount of LR = 4 mL x BSA
right arm (9%), posterior trunk (18%), front % x pt's weight in kg. The pt's weight 63 kg.
and back of the left leg (18%), back of right BSA percentage: 58.5%...Front and back of
leg (9%) equals: 63%......4 x 63 x 91 = right and left leg (36%), front of right arm
22,932 mL (4.5%), anterior trunk (18%) which equals
58.5%. ......4 x 58.5 x 63 = 14,742
mL......Remember during the FIRST 8 hours
2. A 30 year old female patient has deep 1/2 of the solution is infused, which will be
partial thickness burns on the back of the 14,742 divided by 2 = 7371 mL......Hourly
right arm, posterior trunk, front of the left Rate: 7371 divide by 8 equals 921 mL/hr
leg, anterior head and neck, and perineum.
The patient weighs 150 lbs. Use the
Parkland Burn Formula to calculate the total 4. A 59 year old male patient has full
amount of Lactated Ringers that will be thickness burns on both of the legs on the
given over the next 24 hours? back, front and back of the trunk, both arms
A. 14,960 mL on the front and back, and front and back of
B. 12,512 mL the head and neck. The patient weighs 186
C. 10,064 mL lbs. Use the Parkland Burn Formula: You've
D. 16,896 mL already infused fluids during the first 8
The answer is C: 10,064 mL Formula: Total hours. Now what will you set the flow rate
Amount of LR = 4 mL x BSA % x pt's weight during the next 16 hours (mL/hr) based on
in kg. Pt's weight 150 lbs....convert to the total you calculated?
kg....150 divided by 2.2 = 68 kg. BSA A. 563 mL/hr
percentage: 37%...Back of right arm (4.5%), B. 854 mL/hr
posterior trunk (18%), front of left leg (9%), C. 289 mL/hr
anterior head and neck (4.5%) and D. 861 mL/hr
perineum (1%) which equals 37%......4 x 37 The answer is D: 861 mL/hr First calculate
x 68 = 10,064 mL the total amount of fluid needed with the
formula: Total Amount of LR = 4 mL x BSA
% x pt's weight in kg. The pt's weight 186
3. A 30 year old female patient has deep lbs...need to convert to kg: 186 divided by
partial thickness burns on the front and back 2.2 = 85 kg. BSA percentage: 81%...Both of
of the right and left leg, front of right arm, the legs on the back (18%), front and back
and anterior trunk. The patient weighs 63 of the trunk (36%), both arms on the front
kg. Use the Parkland Burn Formula: What is and back (18%), front and back of the head
the flow rate during the FIRST 8 hours and neck (9%) which equals 81%.......4 x 81
(mL/hr) based on the total you calculated? x 85 = 27,540 mL. You've already infused
A. 921 mL/hr half of the solution during the first 8
B. 938 mL/hr hours...so 13,770 mL is left and it needs to
C. 158 mL/hr be infused over 16 hours. Hourly rate:
D. 789 mL/hr 13,770 mL divided by 16 hours equals 861
mL/hr
3. Tall peaked T-waves, flat P-waves,
prolonged PR intervals and widened QRS
5. A 29 year old male patient has superficial complexes can present in which of the
partial thickness burns on the anterior right following conditions?
arm, posterior left leg, and anterior head A. Hypocalemia
and neck. The patient weighs 78 kg. Use B. Hypercalemia
the Parkland Burn Formula to calculate the C. Hypokalemia
total amount of Lactated Ringers that will be D. Hyperkalemia
given over the next 24 hours? The answer is D: Hyperkalemia
A. 11,232 mL
B. 5,616 mL
C. 2,808 mL 2. Which of the following is not a symptom
D. 16,848 mL of hyperkalemia?
The answer is B: 5,616 mL. Formula: Total A. Positive Chvostek’s sign
Amount of LR = 4 mL x BSA % x pt's weight B. Decreased blood pressure
in kg. Pt's weight 78 kg. BSA percentage: C. Muscle twitches/cramps
18%...Anterior right arm (4.5%), posterior D. Weak and slow heart rate
left leg (9%), and anterior head and neck The answer is A: Positive Chvostek’s sign
(4.5%) which equals 18%.....4 x 18 x 78 =
5,616 mL

1. A patient has a potassium level of 9.0. 6. A patient with nasogastric suctioning is


Which nursing intervention is priority? experiencing diarrhea. The patient is
A. Prepare the patient for dialysis and place ordered a morning dose of Lasix 20mg IV.
the patient on a cardiac monitor Patient’s potassium level is 3.0. What is
B. Administer Spironolactone your next nursing intervention?
C. Place patient on a potassium restrictive A. Hold the dose of Lasix and notify the
diet doctor for further orders
D. Administer a laxative B. Administered the Lasix and notify the
The answer is A: Prepare the patient for doctor for further orders
dialysis and place the patient on a cardiac C. Turn off the nasogastric suctioning and
monitor administered a laxative
D. No intervention is need the potassium
level is within normal range
4. Which patient is at risk for hyperkalemia? The answer is A: Hold the dose of Lasix and
A. Patient with Parathyroid cancer notify the doctor for further orders
B. Patient with Cushing’s Syndrome
C. Patient with Addison’s Disease
D. Patient with breast cancer 5. A patient’s potassium level is 3.0. Which
The answer is C: Patient with Addison’s foods would you encourage the patient to
Disease consume?
A. Cheese, collard greens, and fish
B. Avocados, strawberries, and potatoes
C. Tofu, oatmeal, and peas
D. Peanuts, bread, and corn
The answer is B: Avocados, strawberries, 9. A patient has a potassium level of 2.0.
and potatoes What would you expect to be ordered for
this patient?
A. Potassium 30 meq IV push
7. A patient is presenting with an orthostatic B. Infusion of Potassium intravenously
blood pressure of 80/40 when she stands C. An oral supplement of potassium
up, thready and weak pulse of 58, and D. Intramuscular injection of Potassium
shallow respirations. In addition, the patient The answer is B: Infusion of Potassium
has been having frequent episodes of intravenously
vomiting and nausea and is taking
hydrochlorothiazide. Which of the following Nurse Ejay is assigned to telephone triage.
findings would explain the patient’s A client called who was stung by a
condition? honeybee and is asking for help. The client
A. Potassium level of 7.0 reports pain and localized swelling but has
B. Potassium level of 3.5 no respiratory distress or other symptoms of
C. Potassium level of 2.4 anaphylactic shock. What is the appropriate
D. None of the options are correct initial action that the nurse should direct the
The answer is C: Potassium level of 2.4 client to perform?

A. Removing the stinger by scraping it


8. Which patient is at a potential risk for B. Applying a cold compress
Digoxin toxicity? C. Taking an oral antihistamine
A. A patient with Cushing’s syndrome taking D. Calling 911
Laxis 20 mg IV twice a day Correct
B. A patient with a calcium level of 8.9 Correct Answer: A. Removing the stinger by
C. A patient with a potassium level of 3.8 scraping it.
D. A patient presenting with painful muscle
spasms and positive Trousseau’s sign Since the stinger will continue to release
The answer is A: A patient with Cushing’s venom into the skin, removing the stinger
syndrome taking Laxis 20 mg IV twice a day should be the first action that the nurse
should direct to the client. Within the first
few minutes after the sting, the stinger
9. Which of the following is indicative of an should be removed via scraping with a
EKG change in a case of hypokalemia? credit card rather than squeezing/tweezing
A. Widened QRS complex and prolonged to avoid further venom exposure.
PR interval
B. Prolonged ST interval and Widened T- Option B: Uncomplicated local reactions can
wave be treated with supportive care (ice packs,
C. Tall T-waves and depressed ST segment NSAIDs/APAP for pain, H1/H2 blocker).
D. ST depression and inverted T-wave Cold compress follows the administration of
The answer is D: ST depression and antihistamine. Large local reactions should
inverted T-wave also be treated with supportive care along
with glucocorticoids (usually a burst course
of prednisone 40 to 60 mg per day for 3 to 5
days) to decrease the inflammatory Option A: The RN delegates only those
response and improve symptoms. tasks for which he or she believes the other
Option C: After removing the stinger, an health care worker has the knowledge and
antihistamine is administered. H1 and H2 skill to perform, taking into consideration
antagonists block the effects of histamine training, cultural competence, experience
decreasing pruritus, erythema, and urticaria. and facility/agency policies and procedures.
Corticosteroids (prednisone, Option C: This area should be filled with
methylprednisolone, dexamethasone) act to nurses who are experienced with hospital
decrease inflammation and immune routines and policies and have the ability to
response to the antigen. locate equipment immediately. There is both
Option D: The caller should be further individual accountability and organizational
advised about symptoms that require 911 accountability for delegation. Organizational
assistance. Systemic reactions accountability for delegation relates to
(anaphylaxis) are life-threatening and providing sufficient resources, including
should be managed as such. ABCs first. sufficient staffing with an appropriate staff
The airway can be lost within seconds to mix.
minutes, so intubate early. As with any Option D: Few places are more hectic than
anaphylactic reaction, epinephrine, a Hospital Emergency Room. Clearly,
corticosteroids, H1 and H2 antagonists, and delegating important nursing tasks is the
intravenous (IV) fluids should be given only plausible way for short-staffed
immediately. emergency rooms to meet the challenges of
2. Question providing quality patient care. All decisions
Nurse Anna is an experienced travel nurse related to delegation and assignment are
who was recently employed and is assigned based on the fundamental principles of
to the emergency unit. In her first week of protection of the health, safety, and welfare
the job, which of the following area is the of the public.”
most appropriate assignment for her? 3. Question
A client arrives at the emergency
A. Triage department who suffered multiple injuries
B. Ambulatory section from a head-on car collision. Which of the
C. Trauma team following assessment should take the
D. Psychiatric care highest priority to take?
Correct
Correct Answer: B. Ambulatory section A. Unequal pupils
B. Irregular pulse
The ambulatory section deals with clients C. Ecchymosis in the flank area
with relatively stable conditions. The D. A deviated trachea
decision of whether or not to delegate or Correct
assign is based upon the RN’s judgment Correct Answer: D. A deviated trachea
concerning the condition of the patient, the
competence of all members of the nursing A deviated trachea is a symptom of tension
team and the degree of supervision that will pneumothorax, which will result in
be required of the RN if a task is delegated. respiratory distress if left untreated. The first
question in the ESI triage algorithm for A. Administer oxygen therapy via nasal
triage nurses asks whether “the patient cannula
requires immediate life-saving interventions” B. Notify the physician
or simply “is the patient dying?” The nurse C. Complete history taking
determines this by looking to see if the D. Put the client on ECG monitoring
patient has a patent airway, if the patient is Correct
breathing, and does the patient has a pulse. Correct Answer: A. Administer oxygen
therapy via nasal cannula.
Option A: Another scale used by nurses in
the assessment is if the patient is meeting The priority goal is to increase myocardial
criteria for a true level 1 trauma is the AVPU oxygenation. Place the patient on a cardiac
(alert, verbal, pain, unresponsive) scale. monitor, establish intravascular access (IV)
The scale is used to evaluate if the patient access, give 162 mg to 325 mg chewable
had a recent or sudden change in the level aspirin, clopidogrel, or ticagrelor (unless
of consciousness and needs immediate bypass surgery is imminent), control pain
intervention. and consider oxygen (O2) therapy.
Option B: The nurse evaluates the patient,
checking pulse, rhythm, rate, and airway Option B: Patients with non-ST elevation
patency. Is there concern for inadequate myocardial infarction (NSTEMI) and
oxygenation? Is this person unstable angina should be admitted for
hemodynamically stable? Does the patient cardiology consultation and workup.
need any immediate medication or Patients with stable angina may be
interventions to replace volume or blood appropriate for outpatient workup.
loss? Does this patient have pulselessness, Option C: Carefully review the patient’s
apnea, severe respiratory distress, oxygen medical history for cardiac history,
saturation below 90, acute mental status coagulopathies, and kidney disease. Ask
changes, or unresponsiveness? about family history, especially cardiac, and
Option C: If the patient is not categorized as ask about social histories like drug use and
a level 1, the nurse then decides if the tobacco use.
patient should wait or not. This is Option D: These actions are also
determined by three questions; is the appropriate and should be performed
patient in a high-risk situation, confused, immediately. Electrocardiogram (ECG)
lethargic, or disoriented? Or is the patient in preferably in the first 10 min of arrival,
severe pain or distress? The high-risk (consider serial ECGs). Patients with ST-
patient is one who could easily deteriorate, elevation on ECG patients should receive
one who could have a threat to life, limb, or immediate reperfusion therapy either
organ. pharmacologic (thrombolytics) or transfer to
4. Question the catheterization laboratory for
Nurse Kelly, a triage nurse encountered a percutaneous coronary intervention (PCI).
client who complained of mid-sternal chest 5. Question
pain, dizziness, and diaphoresis. Which of A group of people arrived at the emergency
the following nursing action should take unit by a private car with complaints of
priority? periorbital swelling, cough, and tightness in
the throat. There is a strong odor emanating
from their clothes. They report exposure to Option C: Decontamination triage is
a “gas bomb” that was set off in the house. especially important in mass casualty
What is the priority action? incidents and should not be confused with
medical triage. Decontamination triage is
A. Instruct personnel to don personal the process of determining which victims
protective equipment require decontamination and which do not.
B. Direct the clients to the cold or clean Rapidly identifying victims who may not
zone for immediate treatment require decontamination can significantly
C. Immediately remove other clients and reduce the time and resources needed for
visitors from the area mass decontamination.
D. Measure vital signs and auscultate lung Option D: Set up or assign an area or
sounds building as a safe refuge/observation area
E. Direct the clients to the decontamination for victims who do not require medical
area attention. Here they can be monitored for a
Correct delayed outbreak of symptoms or
Correct Answer: E. Direct the clients to the indications of residual contamination.
decontamination area. 6. Question
When an unexpected death occurs in the
Decontamination in a specified area is the emergency department, which task is the
priority. The decontamination and support most appropriate to delegate to a nursing
areas are established within the Warm assistant?
Zone, also referred to as the Contamination
Reduction Zone. Decontamination involves A. Assisting with postmortem care
thorough washing to remove contaminants. B. Facilitate meetings between the family
and the organ donor specialist
Option A: Personnel should don personal C. Escorting the family to a place of privacy
protective equipment before assisting with D. Help the family to collect belongings
decontamination or assessing the clients. Correct
Take precautionary measures to preserve Correct Answer: A. Assisting with
the health and safety of emergency postmortem care.
responders working within the
Contamination Reduction (Warm) Zone and Postmortem care requires some turning,
the Exclusion (Hot) Zone. This includes cleaning, lifting, and so on, and the nursing
ensuring responders wear appropriate assistant is able to assist with these duties.
personal protective equipment (PPE). The use of NAPs increasingly demands
Option B: The clients must undergo registered nurses to delegate patient care
decontamination before entering cold or tasks according to the principles of the ANA.
clean areas. In mass casualty incidents, These principles define nursing delegation
decontamination corridors can be set up as the “transfer of responsibility for the
that consist of high volume, low-pressure performance of an activity from one
water deluges. Assign personnel to individual to another while retaining
decontamination stations to control and accountability for the outcome.”
instruct victims when they enter the
decontamination area.
Option B: The RN may delegate B. Explain the need for giving cool fluids
components of care, but does not delegate C. Assist the child in removing outer
the nursing process itself. The practice of clothing
pervasive functions of assessment, D. Advise the parent to use acetaminophen
planning, evaluation, and nursing judgment (Tylenol) instead of aspirin
cannot be delegated. The decision of Correct
whether or not to delegate or assign is Correct Answer: C. Assist the child in
based upon the RN’s judgment concerning removing outer clothing.
the condition of the patient, the competence
of all members of the nursing team, and the The nursing assistant can help with the
degree of supervision that will be required of removal of outer clothing, which allows the
the RN if a task is delegated. heat to dissipate from the child’s skin. The
Option C: A licensed nurse should take client is the center of care. The needs of the
responsibility for the other tasks to help the client must be competently met with the
family begin the grieving process. The RN knowledge, skills and abilities of the staff to
delegates only those tasks for which he or meet these needs. In other words, the nurse
she believes the other health care worker who delegates aspects of care to other
has the knowledge and skill to perform, members of the nursing team must balance
taking into consideration training, cultural the needs of the client with the abilities of
competence, experience, and those to which the nurse is delegating tasks
facility/agency policies and procedures. and aspects of care, among other things
Option D: In cases of questionable death, such as the scopes of practice and the
belongings may be retained for evidence, so policies and procedures within the particular
the chain of custody would have to be healthcare facility.
maintained. The registered nurse
individualized communication regarding the Option A: Tepid baths are not usually given
delegation to the nursing assistive because of the possibility of shivering and
personnel and client situation and the rebound. Registered nurses who assign,
communication should be clear, concise, delegate, and/or provide nursing care to
correct, and complete. The registered nurse clients and groups of clients must report all
verifies comprehension with the nursing significant changes that occur in terms of
assistive personnel and that the assistant the client and their condition. For example,
accepts the delegation and the a significant change in a client’s laboratory
responsibility that accompanies it. values requires that the registered nurse
7. Question report this to the nurse’s supervisor and
The physician has ordered cooling doctor.
measures for a child with a fever who is Option B: Explaining is a teaching function
likely to be discharged when the only appropriate for a registered nurse. The
temperature comes down. Which task would staff members’ levels of education,
be appropriate to delegate to a nursing knowledge, past experiences, skills,
assistant? abilities, and competencies are also
evaluated and matched with the needs of all
A. Prepare and administer a tepid sponge of the patients in the group of patients that
bath will be cared for.
Option D: Advising is a teaching function administer fluid directly into the veins using
that is the responsibility of the registered IV therapy. IV therapy is a relatively simple
nurse. Delegation should be done according process that can be performed by nurses,
to the differentiated practice for each of the but there are serious complications
staff members. A patient care technician, a associated with it.
certified nursing assistant, a licensed Option B: Tetanus status can be addressed
practical nurse, an associate degree later. Tetanus immunization is part of the
registered nurse, and a bachelor’s degree DTaP (diphtheria, tetanus, and acellular
registered nurse should not be delegated to pertussis) vaccinations. Kids usually get: a
the same aspects of nursing care. series of four doses of DTaP vaccine before
8. Question 2 years of age. another dose at 4–6 years of
You are preparing a child for IV conscious age.
sedation before the repair of a facial Option C: The RN can provide information
laceration. What information should you about conscious sedation. Identifying
report immediately to the physician? teachable moments in clinical practice is an
effective way to increase workplace learning
A. The child suddenly pulls out the IV with all nurses playing a role, not just nurse
B. The parent is not sure regarding the educators.
child's tetanus immunization status 9. Question
C. The parent wants information about the The emergency medical service has
IV conscious sedation transported a client with severe chest pain.
D. The parent's refusal of the As the client is being transferred to the
administration of the IV sedation emergency stretcher, you note
Correct unresponsiveness, cessation of breathing,
Correct Answer: D. The parent’s refusal of and unpalpable pulse. Which of the
the administration of the IV sedation. following task is appropriate to delegate to
the nursing assistant?
The refusal of the parents is an absolute
contraindication; therefore the physician A. Assisting with the intubation
must be notified. But the autonomy of B. Placing the defibrillator pads
parents is very obviously different from the C. Doing chest compressions
autonomy of patients to make decisions for D. Initiating bag valve mask ventilation
themselves. While adult patients are Correct
generally thought to have an absolute right Correct Answer: C. Doing chest
to refuse medical treatment for themselves, compressions.
we don’t usually think that parents can
refuse all medical treatment for their Performing chest compressions are within
children. the training of a nurse assistant. Every good
certified nursing assistant should be
Option A: The RN can reestablish the IV proficient at cardiopulmonary resuscitation
access. Parents’ views might, at least in (CPR). Basic Life Support (BLS) certification
some circumstances, influence whether or is the widely-used term for any form of CPR
not treatment would be in a child’s best certification and is required for all
interests. Nurses and doctors are able to
Registered Nurses (RN) and Certified At least one representative from each group
Nursing Assistants (CNA). of ED personnel should be included
because all employees are potential targets
Option A: Due to the nature of critical care for violence in the ED. The diversity of the
patients it is inevitable that nurses working group should also be considered and
in this setting will be called on to assist with assure that each department or each
tracheal intubation. This assistant role employee is represented.
requires a high degree of knowledge and
skills in order for the intubation procedure to Option A: The policies against violence in
be executed smoothly. the workplace must be developed by
Option B: The defibrillator pads are clearly management and employee
marked; however placement should be representatives, including the health and
done by the RN or physician because of the safety committee or representative, and
potential for skin damage and electrical union, if present.
arcing. Most hospital nurses will be trained Option B: Administrative practices may also
in advisory defibrillation, while a few will be include education and training for
trained in manual defibrillation. employees. This education and training
Option C: The use of the bag valve mask would include not only information about the
requires practice, and usually a respiratory workplace’s policy and process to respond
therapist will perform the function. Bag- to incidents, but may also include how to
valve-mask ventilation can be done with one respond to an incident of violence (e.g.,
person or two, but two-person BVM emergency response, when to contact
ventilation is easier and more effective security or police, etc.).
because a tight seal must be achieved and Option C: Preventive measures generally
this usually requires two hands on the fall into three categories, workplace design,
mask. administrative practices, and work practices.
10. Question All employees should know how to respond
The nursing manager decides to form a to customers or members of the public who
committee to address the issue of violence may be angry or frustrated, such as how to
against ED personnel. Which combination de-escalate a conflict.
of employees would be best suited to fulfill 11. Question
this assignment? A client suffered an amputation of the first
and second digits in a chainsaw accident.
A. ED physicians and charge nurses Which task should be delegated to an
B. RNs, LPNs, and nursing assistants LPN/LVN?
C. Experienced RNs and experienced
paramedics A. Cleansing the amputated digits and
D. At least one representative from each placing them directly into an ice slurry
group of ED personnel B. Wrapping the cleansed digits in saline-
Incorrect moistened gauze, sealing in a plastic
Correct Answer: D. At least one container, and placing it in icy water
representative from each group of ED C. Gently cleansing the amputated digits
personnel. and the hand with povidone-iodine
D. Cleansing the digits with sterile normal delegated to members of the nursing team.
saline and placing it in a sterile cup with The delegated care must be followed up on
sterile normal saline and the staff members have to be
Incorrect supervised as they deliver care. The
Correct Answer: B. Wrapping the cleansed registered nurse remains responsible for
digits in saline-moistened gauze, sealing in and accountable for the quality,
a plastic container, and placing it in icy appropriateness, completeness, and
water. timeliness of all of the care that is delivered.
12. Question
Once a finger amputation has occurred, A client arrives in the emergency unit and
ischemic tolerance times are 12 hours if reports that a concentrated household
warm and up to 24 hours if cold. For more cleaner was splashed in both eyes. Which
proximal amputations, these times are of the following nursing actions is a priority?
halved. The amputated part should be
covered in a normal saline-soaked gauze, A. Examine the client's visual acuity
sealed in a plastic bag, and submerged in B. Patch the eye
icy water with no direct contact with ice. If C. Use Restasis (Allergan) drops in the eye
there is direct contact with ice, it could result D. Flush the eye repeatedly using sterile
in tissue damage and render the amputated normal saline
part non-viable. Correct
Correct Answer: D. Flush the eye
Option A: The client is the center of care. repeatedly using sterile normal saline.
The needs of the client must be competently
met with the knowledge, skills, and abilities Initial emergency action during a chemical
of the staff to meet these needs. In other splash to the eye includes immediate
words, the nurse who delegates aspects of continuous irrigation of the affected eye with
care to other members of the nursing team normal saline. Immediate irrigation with
must balance the needs of the client with copious amounts of an isotonic solution as
the abilities of those to which the nurse is described previously is the mainstay of
delegating tasks and aspects of care, treatment for chemical burns. Never use
among other things such as the scopes of any substance to neutralize chemical
practice and the policies and procedures exposure as the exothermic reaction can
within the particular healthcare facility. lead to secondary thermal injuries.
Option C: Based on these characteristics
and the total client needs for the group of Option A: After irrigation, visual acuity then
clients that the registered nurse is is assessed. Irrigation should continue until
responsible and accountable for, the the pH of the eye is between 7.0 to 7.4 and
registered nurse determines and analyzes remains within this range for at least 30
all of the health care needs for a group of minutes after the irrigation has been
clients; the registered nurse delegates care discontinued.
that matches the skills of the person that the Option B: Patching the eye is not part of the
nurse is delegating to. first-line treatment of a chemical splash.
Option D: The job of the registered nurse is Irrigation should be gentle, and care should
far from done after client care has been be taken to avoid direct irrigation to the
cornea to prevent further injury. Use of a information should be reported to the
commercial irrigation lens such as a Morgan physician immediately?
lens may be helpful.
Option C: Restasis (Allergan) drops are A. The child was nauseated and vomited
used to treat dry eyes. A topical anesthetic once at home
such as tetracaine can be applied directly to B. The child has been treated several times
the eye, or 10 mL of 1% lidocaine can be for toxic substance ingestion
added to a liter of irrigating fluid, taking care C. The vitamin that was ingested contains
not to reach a toxic dose if copious irrigation iron
is required. D. The child has been treated multiple
13. Question times for injuries caused by accidents
A client was brought to the emergency Incorrect
department after suffering a closed head Correct Answer: C. The vitamin that was
injury and lacerations around the face due ingested contains iron.
to a hit-run accident. The client is
unconscious and has a minimal response to Iron is a toxic substance that can lead to
noxious stimuli. Which of the following massive hemorrhage, shock, coma, and
assessment findings if observed after few kidney failure. Iron poisoning is one of the
hours, should be reported to the physician most common toxic ingestion and one of the
immediately? most deadly among children. Failure to
diagnose and treat iron poisoning can have
A. Drainage of a clear fluid from the client's serious consequences including multi-organ
nose failure and death.
B. Withdrawal of the client in response to
painful stimuli Option A: During the first stage (0.5 to 6
C. Bruises and minimal edema of the hours), the patient mainly exhibits
eyelids gastrointestinal (GI) symptoms including
D. Bleeding around the lacerations abdominal pain, vomiting, diarrhea,
Correct hematemesis, and hematochezia. The
Correct Answer: A. Drainage of a clear fluid second stage (6 to 24 hours) represents an
from the client’s nose apparent recovery phase, as the patient’s
GI symptoms may resolve despite toxic
Option A: Clear drainage from the client’s amounts of iron absorption.
nose indicates that there is a leakage of Option B: Patients who have GI symptoms
CSF and should be reported to the that resolve after a short period of time and
physician immediately. have normal vital signs require supportive
14. Question care and an observation period, as it may
A 5-year-old client was admitted to the represent the second stage of iron toxicity.
emergency unit due to the ingestion of an Patients who remain asymptomatic 4 to 6
unknown amount of chewable vitamins for hours after ingestion or those who have not
children at an unknown time. Upon ingested a potentially toxic amount do not
assessment, the child is alert and with no require any treatment for iron toxicity.
symptoms. Which of the following Option D: This information needs further
investigation but will not change the
immediate diagnostic testing or treatment treatment depending upon the size of the
plan. Patients who have GI symptoms that aneurysm and/or symptomatology.
resolve after a short period of time and have The woman with lower left quadrant pain is
normal vital signs require supportive care at risk for a life-threatening ectopic
and an observation period, as it may pregnancy. An ectopic pregnancy occurs
represent the second stage of iron toxicity. when fetal tissue implants outside of the
15. Question uterus or attaches to an abnormal or
The following clients come to the scarred portion of the uterus. Ectopic
emergency department complaining of pregnancies carry high rates of morbidity
acute abdominal pain. Prioritize them for and mortality if not recognized and treated
care in order of the severity of the promptly.
conditions. The 15-year-old boy needs evaluation to
rule out appendicitis. It should be
A 59-year-old man with a pulsating considered in any patient with acute
abdominal mass and sudden onset of abdominal pain without a prior
persistent abdominal or back pain, which appendectomy. The diagnosis must be
can be described as a tearing sensation made as quickly as possible because with
within the past hour. time, the rate of rupture increases.
A 27-year-old woman complaining of The woman with vomiting needs evaluation
lightheadedness and severe sharp left lower for gallbladder problems, which appear to
quadrant pain who reports she is possibly be worsening. Disruption of the
pregnant. gallbladder’s normal physiology can result in
A 15-year-old boy with a low-grade fever, a significant medical burden. Over 20 million
right lower quadrant pain, vomiting, nausea, Americans suffer from gallbladder disease
and loss of appetite for the past few days. and cholecystectomy is one of the most
A 43-year-old woman with moderate right common surgeries performed.
upper quadrant pain who has vomited small Lastly, the woman with mid-epigastric pain
amounts of yellow bile and whose is suffering from an ulcer, but follow-up
symptoms have worsened over the week. diagnostic testing can be scheduled with a
A 57-year-old woman who complains of a primary care provider. It is important to
sore throat and gnawing midepigastric pain understand this disease process is both
that is worse between meals and during the preventable and treatable. Patients may be
night. treated differently depending on the etiology
Incorrect of their gastric ulcer.
The correct order is shown above. 16. Question
The following clients are presented with
Rationale: signs and symptoms of heat-related illness.
Which of them needs to be attended first?
The client with a pulsating mass has an
abdominal aneurysm that may rupture and A. A relatively healthy homemaker who
he may decompensate easily. An abdominal reports that the air conditioner has been
aortic aneurysm is a life-threatening broken for days and who manifest fatigue,
condition that requires monitoring or hypotension, tachypnea, and profuse
sweating.
B. An elderly person who complains of Option B: The elderly client is at risk for heat
dizziness and syncope after standing in the syncope and should be advised to rest in a
sun for several hours to view a parade. cool area and avoid similar situations. Heat
C. A homeless person who is a poor syncope is the temporary, self-limited
historian; has altered mental status, poor dizziness, weakness, or loss of
muscle coordination, and hot, dry ashen consciousness during prolonged standing or
skin; and whose duration of heat exposure positional changes in a hot environment,
is unknown. including physical activity. The thinking is
D. A marathon runner who complains of that it is due to a combination of
severe leg cramps and nausea, and dehydration, pooling of blood in the venous
manifests weakness, pallor, diaphoresis, system, decreased cardiac filling, and low
and tachycardia. blood pressure, which leads to decreased
Correct cerebral blood flow.
Correct Answer: C. A homeless person who Option D: The runner is experiencing heat
is a poor historian; has altered mental cramps, which can be managed with fluid
status, poor muscle coordination, and hot, and rest. Heat cramps: include involuntary
dry ashen skin; and whose duration of heat spasmodic contractions of large muscle
exposure is unknown. groups as opposed to an isolated muscle
spasm/cramp that can also occur during or
The signs and symptoms manifested by the after exertion. This condition is due to a
homeless person indicate that a heat stroke relative deficiency of sodium, potassium,
is happening, a medical emergency, which chloride, or magnesium. Other symptoms
can lead to brain damage. Also, there must may include nausea, vomiting, fatigue,
be clinical signs of central nervous system weakness, sweating, and tachycardia.
dysfunction that may include ataxia, 17. Question
delirium, or seizures, in the setting of An anxious female client complains of chest
exposure to hot weather or strenuous tightness, tingling sensations, and
physical exertion. Patients who present with palpitations. Deep, rapid breathing, and
heat stroke typically have vital sign carpal spasms are noted. Which of the
abnormalities to include an elevated core following priority action should the nurse do
body temperature, sinus tachycardia, first?
tachypnea, a widened pulse pressure, and a
quarter of patients will be hypotensive. A. Provide oxygen therapy
B. Notify the physician immediately
Option A: The homemaker is experiencing C. Administer anxiolytic medication as
heat exhaustion, which can be managed by ordered
fluids and cooling measures. It is important D. Have the client breathe into a brown
to differentiate where the patient is on the paper bag
heat illness continuum. The signs and Incorrect
symptoms of heat exhaustion may present Correct Answer: D. Have the client breathe
similarly include cramping, fatigue, into a brown paper bag
dizziness, nausea, vomiting, headache. If
progression to end-organ damage occurs it The client is suffering from hyperventilation
then becomes heat injury. secondary to anxiety, the initial action is to
let the client breathe in a paper bag that will B. Administer Naloxone (Narcan) 4 mg as
allow the rebreathing of carbon dioxide. The ordered
idea behind breathing into a paper bag or C. Contact the family to get information
mask is that rebreathing exhaled air helps about the client
the body put CO2 back into the blood. D. Obtain an order for the determination of
blood alcohol level
Option A: Acute anxiety may require Incorrect
treatment with a benzodiazepine. Chronic Correct Answer: A. Administer IV fluid
anxiety treatment consists of incorporated with Vitamin B1 as ordered.
psychotherapy, pharmacotherapy, or a
combination of both. Anxiety disorders The client has symptoms of alcohol abuse
appear to be caused by an interaction of and there is a risk for Wernicke syndrome,
biopsychosocial factors. Genetic which is caused by a deficiency in Vitamin
vulnerability interacts with situations that are B. Thiamine deficiency (vitamin B1) is
stressful or traumatic to produce clinically common in patients with alcohol
significant syndromes. dependence. Cognitive impairments may be
Option B: Anxiety is one of the most an early consequence of thiamine
common psychiatric disorders but the true deficiency. Wernicke’s encephalopathy is
prevalence is not known as many people do underdiagnosed and undertreated.
not seek help or clinicians fail to make the
diagnosis. Anxiety is one of the most Option B: Multiple drug abuse is not
common psychiatric disorders in the general uncommon; however, there is currently
population. Specific phobia is the most nothing to suggest an opiate overdose that
common with a 12-month prevalence rate of requires the administration of naloxone.
12.1%. Social anxiety disorder is the next Naloxone is indicated for the treatment of
most common, with a 12-month prevalence opioid toxicity, specifically to reverse
rate of 7.4%. respiratory depression from opioid use. It is
Option C: Selective serotonin reuptake useful in accidental or intentional overdose
inhibitors (SSRIs), serotonin-norepinephrine and acute or chronic toxicity.
reuptake inhibitors (SNRIs), Option C: Teens and young adults are at
benzodiazepines, tricyclic antidepressants, higher risk for binge drinking, which can
mild tranquilizers, and beta-blockers treat cause alcohol poisoning. Binge drinking is a
anxiety disorders. pattern of drinking that raises the blood
18. Question alcohol level within a short period of time.
An intoxicated client comes into the Though it varies from person to person,
emergency unit with uncooperative binge drinking is usually defined as four
behavior, mild confusion, and slurred drinks for women and five drinks for men in
speech. The client is unable to provide a a two-hour period.
good history but he verbalizes that he has Option D: Additional information or the
been drinking a lot. Which of the following is results of the blood alcohol testing are part
a priority action of the nurse? of the management but should not delay the
immediate treatment. A blood alcohol test
A. Administer IV fluid incorporated with may be used to find out if the patient has
Vitamin B1 as ordered alcohol poisoning, a life-threatening
condition that happens when your blood Option D: These are important for the long-
alcohol level gets very high. Alcohol term management of this case. Health
poisoning can seriously affect basic body professionals provide an opportunity for
functions, including breathing, heart rate, victims of domestic violence to obtain help.
and temperature. Of those injured by domestic violence, over
19. Question 75% continue to experience abuse. Over
A nurse is providing discharge instructions half of battered women who attempt suicide
to a woman who has been treated for will try again; often they are successful with
contusions and bruises due to domestic the second attempt.
violence. What is the priority intervention for 20. Question
this client? In the work setting, what is the primary
responsibility of the nurse in preparation for
A. Arranging transportation to a safe house disaster management, that includes natural
B. Advising the client about contacting the disasters and bioterrorism incidents?
police
C. Making an appointment to follow up on A. Being aware of the signs and symptoms
the injuries of potential agents of bioterrorism
D. Making a referral to a counselor B. Making ethical decisions regarding
Incorrect exposing self to potentially lethal
Correct Answer: A. Arranging transportation substances
to a safe house. C. Being aware of the agency's emergency
response plan
Safety is a priority for this client and she D. Being aware of what and how to report
should not return to a place where violence to the Centers for Disease Control and
could recur. Make sure a safe environment Prevention
is provided. Offer shelter options, legal Correct
services, counseling, and facilitate such Correct Answer: C. Being aware of the
referral. agency’s emergency response plan.

Option B: If the patient does not want to go In disaster preparedness, the nurse should
to a shelter, provide telephone numbers for know the emergency response plan. This
domestic violence or crisis hotlines and gives guidance that includes the roles of the
support services for potential later use. team members, responsibilities, and
Provide the patient with instructions but be mechanisms of reporting. Emergency
mindful that written materials may pose a preparedness encompasses diverse fields
danger once the patient returns home. within the hospital and regional settings.
Option C: Without proper social service and Planning membership groups should
mental health intervention, all forms of address key aspects across these fields
abuse can be recurrent and escalating including but not limited to: public safety,
problems, and the prognosis for recovery is facilities, logistics, pharmacy, transportation,
poor. Without treatment, domestic and clinical patient care, non-clinical patient
family violence usually recurs and escalates care, media/public relations,
in both frequency and severity. communications, radiation, infection control,
and administration.
pneumothorax happens when there is an
Option A: Planning/Prevention focuses on opening in the chest wall ( from a gun shot,
providing protection from disasters on both stabbing etc.) that creates a passage
the domestic and international levels in an between the outside air and intrapleural
attempt to limit the loss of life and reduce space. This allow air to pass back and forth
the financial impact of disaster response. during inspiration and expiration. The body
Planning includes care, evacuation, and will shunt air through the opening in the
environmental planning and response chest well instead of the trachea (if the
standards. opening on the chest is large enough) which
Option B: Risk Assessment identifies high will create a "sucking" sound.
priority and vulnerability areas and directs
mitigation efforts. The goal of risk
assessment is the identification of the 2. In regards to the patient in the question
possible disasters that challenge the area above, which of the following options below
including both internal and external is a nursing intervention you would provide
disasters, collecting resource inventory, to this patient?
identifying a facility or region’s A. Place the patient in supine position
vulnerabilities based on location and B. Place a non-occlusive dressing over the
resources, and generating a priorities list. chest wound
Option D: The Centers for Disease Control C. Place a sterile occlusive dressing over
and Prevention (CDC) provides education the chest wound and tape it on three sides
and information regarding specific disaster D. Prepare the patient for a thoracentesis
types, including infectious diseases, The answer is C. The nursing intervention
chemical and radiation exposure, and would be to place a sterile occlusive
natural disaster or weather-related dressing over the wound and tape it on 3
incidents. sides (leaving one side NOT taped). This
will allow exhaled air to leave the opening
1. A patient is admitted with a chest wound but seal over the opening when inhaling
and experiencing extreme dyspnea, (hence not letting pressure build in the
tachycardia, and hypoxia. The chest wound intrapleural space and prevent a tension
is located on the left mid-axillary area of the pneumothorax).
chest. On assessment, you note there is
unequal rise and fall of the chest with
absent breath sounds on the left side. You 3. A patient is diagnosed with a primary
also note a "sucking" sound when the spontaneous pneumothorax. Which of the
patient inhales and exhales. The patient's following is NOT a correct statement about
chest x-ray shows a pneumothorax. What this type of pneumothorax?
type of pneumothorax is this known as? A. It can be caused by the rupture of a
A. Closed pneumothorax pulmonary bleb.
B. Open pneumothorax B. It can occur in patients who are young,
C. Tension pneumothorax tall and thin without a history of lung
D. Spontaneous pneumothorax disease.
The answer is B. This description is of an
open pneumothorax. An open
C. Smoking increases the chances of a
patient developing a spontaneous
pneumothorax. 6. You're providing care to a patient with a
D. It is most likely to occur in patients with pneumothorax who has a chest tube. On
COPD, asthma, and cystic fibrosis. assessment of the chest tube system, you
The answer is D. All options are correct note there is no fluctuation of water in the
about primary spontaneous pneumothorax water seal chamber as the patient inhales
EXCEPT D. This describes SECONDARY and exhales. You check the system for
spontaneous pneumothorax not primary. kinks and find none. What is your next
nursing action?
A. Keep monitoring the patient because this
4. Which of the following is a LATE sign of is a normal finding.
the development of a tension B. Increase wall suction to the system until
pneumothorax? the water fluctuates in the water seal
A. Hypotension chamber.
B. Tachycardia C. Assess patient's lung sounds to assess if
C. Tracheal deviation the affected lung has re-expanded.
D. Dyspnea D. Notify the physician.
The answer is C. With a tension The answer is C. It is normal for the water
pneumothorax, you will quickly see seal chamber to tidal up and down as the
hypotension, tachycardia, and dyspnea as patient breathes in and out. If there is no
the mediastinum shifts from the extra fluctuation of water in the water seal
pressure in the intrapleural space on the chamber there may be a kink in the tubing
affected side. A late sign of a tension or the lung has re-expanded. Therefore, it is
pneumothorax is that the trachea will important to check the system for kinks and
eventually shift to the unaffected side. if there are none then check the patient's
lung sounds to see if lung sounds are
present on the affected side (hence the lung
5. While caring for a patient with a has re-expanded).
suspected pneumothorax, you note there
are several areas on the patient’s skin that
appear to be “bulging” out. These “bulging” 7. A patient is receiving mechanical
areas are located on the patient’s neck, ventilation with PEEP. The patient had
face, and abdomen. On palpation on these developed a tension pneumothorax. Select
areas, you note they feel "crunchy". When ALL the signs and symptoms that can
charting your findings you would refer to this present with this condition:
finding as? A. Hypotension
A. Subcutaneous paresthesia B. Jugular Venous Distention
B. Pigment molle C. Bradycardia
C. Subcutaneous emphysema D. Tracheal deviation
D. Veisalgia E. Hyperemia
The answer is C. This known as F. Tachypnea
subcutaneous emphysema or subcutaneous The answers are A, B, D, and F.
crepitation. Hypotension, JVD, tracheal deviation, and
tachypnea can all be present in a tension B. A tension pneumothorax is a medical
pneumothorax. The other options are not emergency and is treated with needle
usually present. decompression.
C. Tracheal deviation is an early sign of a
tension pneumothorax
8. A patient has a chest tube for treatment D. An open pneumothorax is the only cause
of a pneumothorax in the left lung. Which of a tension pneumothorax.
finding during your assessment requires The answer is B. The only correct statement
immediate nursing intervention? about a tension pneumothorax is option B.
A. The water seal chamber has intermittent Option A is wrong because this condition
bubbling. happens when an opening to the
B. The patient has slight tracheal deviation intrapleural space creates a ONE-way (not
to the right side. two-way) valve which causes pressure to
C. The water seal chamber fluctuates while build up in the space leading to shifting of
the patient inhales and exhales. the mediastinum. Option C is wrong
D. The patient complains of tenderness at because tracheal deviation is a LATE (not
the chest tube insertion site. early) sign of a tension pneumothorax.
The answer is B. A patient with a chest tube Option D is wrong because not only can an
is at risk for a tension pneumothorax due to open pneumothorax cause a tension
the risk of pressure building up in the pneumothorax but a closed or spontaneous
intrapleural space. Therefore, the nurse pneumothroax can as well.
would want to monitor the patient for this
and if tracheal deviation is present this is a
major sign a tension pneumothorax. All the 10. A patient receiving treatment for a
other options are normal findings. The water pneumothorax calls on the call light to tell
seal chamber will have intermittent (not you something is wrong with their chest
excessive) bubbling because of the air that tube. When you arrive to the room you note
will be leaving the intrapleural space. The that the drainage system has fallen on its
water seal chamber will flucutate up and side, and there is a large crack in the
down when the patient breathes in and out, system. What is your next PRIORITY?
and it is normal for the patient to have A. Place the patient in supine position and
tenderness at the insertion site of the chest clamp the tubing.
tube. B. Notify the physician immediately.
C. Disconnect the drainage system and get
a new one.
9. Which statement is CORRECT about a D. Disconnect the tubing from the drainage
tension pneumothorax? system and insert the tubing 1 inch into a
A. This condition happens when an opening bottle of sterile water and obtain a new
to the intrapleural space creates a two-way system.
valve which causes pressure to build up in The answer is D. A new system needs to be
the space leading to shifting of the obtained, however, in order to maintain a
mediastinum. water seal until the new system arrives you
will need to place the tubing 1 inch in sterile
water or sterile saline to regain a water seal.
1. You are providing care to a patient with a suction but uses a suction monitor bellow
chest tube. On assessment of the drainage that balances the wall suction and you can
system, you note continuous bubbling in the adjust water suction pressure using the
water seal chamber and oscillation. Which rotary suction dial on the side of the system.
of the following is the CORRECT nursing It allows for higher suction pressure levels,
intervention for this type of finding? has no bubbling sounds, and water does not
A. Reposition the patient because the tubing evaporate from it as with other systems.
is kinked. A. Mediastinal chest tube system
B. Continue to monitor the drainage system. B. Dry suction chest tube system
C. Increase the suction to the drainage C. Wet suction chest tube system
system until the bubbling stops. D. Dry-Wet suction chest tube system
D. Check the drainage system for an air
leak. 4. The patient in room 2569 calls on the call
Continuous bubbling in the water seal light to tell you something is wrong with his
chamber is NOT normal and indicates there chest tube. When you arrive to the room
is an air leak. However, oscillation of the you note that the drainage system has fallen
water in the water seal chamber is normal. on its side and is leaking drainage onto the
floor from a crack in the system. What is
your next PRIORITY?
2. A patient is receiving positive pressure A. Place the patient in supine position and
mechanical ventilation and has a chest clamp the tubing.
tube. When assessing the water seal B. Notify the physician immediately.
chamber what do you expect to find? C. Disconnect the drainage system and get
A. The water in the chamber will increase a new one.
during inspiration and decrease during D. Disconnect the tubing from the drainage
expiration. system and insert the tubing 1 inch into a
B. There will be continuous bubbling noted bottle of sterile water and obtain a new
in the chamber. system.
C. The water in the chamber will decrease Option D is the best choice. A new system
during inspiration and increase during needs to be obtained, however, in order to
expiration. maintain a water seal until the new system
D. The water in the chamber will not move. arrives you will need to place the tubing 1
When a patient is receiving mechanical inch in sterile water or sterile saline to
ventilation the water in the water seal regain a water seal.
chamber will oscillate oppositely than if the
patient were breathing on their own.
Therefore, the water in the chamber will 5. You're assessing a patient who is post-
decrease during inspiration and increase opt from a chest tube insertion. On
during expiration. assessment, you note there is 50 cc of
serosanguinous fluid in the drainage
chamber, fluctuation of water in the water
3. What type of chest tube system does this seal chamber when the patient breathes in
statement describe? This chest drainage and out, and bubbling in the suction control
system has no water column to control
chamber. Which of the following is the most B. Place a sterile dressing over the site and
appropriate nursing intervention? tape it on three sides and notify the
A. Document your findings as normal. physician.
B. Assess for an air leak due to bubbling C. Attempt to re-insert the tube.
noted in the suction chamber. D. Keep the site open to air and notify the
C. Notify the physician about the drainage. physician.
D. Milk the tubing to ensure patency of the
tubes. 8. A patient is about to have their chest tube
The assessment findings are normal. All the removed by the physician. As the nurse
other options are incorrect. assisting with the removal, which of the
following actions will you perform? Select-
all-that-apply:
6. A patient is recovering from a A. Educate the patient how to take a deep
pneumothorax and has a chest tube breath out and inhale rapidly while the tube
present. Which of the following is an in being removed.
appropriate finding when assessing the B. Gather supplies needed which will
chest tube drainage system? include a petroleum gauze dressing per
A. Intermittent bubbling may be noted in the physician preference.
water seal chamber. C. Place the patient in Semi-Fowler's
B. 200 cc of drainage per hour is expected position.
during recovery of a pneumothorax. D. Have the patient take a deep breath,
C. The chest tube is positioned at the exhale, and bear down during removal of
patient's chest level to facilitate drainage. the tube.
D. All of these options are appropriate E. Pre-medicate prior to removal as ordered
findings. by the physician.
The answer is A. It is normal to find F. Place the patient is prone position after
intermittent (NOT CONTINUOUS) bubbling removal.
in the water seal chamber if the patient is Option A: is wrong because this is not how
recovery from a pneumothorax. Remember the Valsalva Maneuver is performed (the
that a pneumothorax is an AIR leak correct way is detailed in option D). Option
between the lung and chest wall....therefore F: is wrong as well because this position
air will escape into the water seal chamber would not faciltate breathing...Fowler's
causing intermittent bubbles. position is best after removal.

7. While helping a patient with a chest tube 9. A patient with a chest tube has no
reposition in the bed, the chest tube fluctuation of water in the water seal
becomes dislodged. What is your immediate chamber. What could be the cause of this?
nursing intervention? A. This is an expected finding.
A. Stay with the patient and monitor their B. The lung may have re-expanded or there
vital signs while another nurse notifies the is a kink in the system.
physician. C. The system is broken and needs to be
replaced.
D. There is an air leak in the tubing.
1. Question discussed after the first few years after
A 23-year-old male client who has had a injury. This option is often used to “improve
full-thickness burn is being discharged from both the function and the cosmetic
the hospital. Which information is most appearance of burn scars”.
important for the nurse to provide prior to 2. Question
discharge? A client who is admitted after a thermal burn
injury has the following vital signs: blood
A. How to maintain home smoke detectors pressure, 70/40; heart rate, 140 beats/min;
B. Joining a community reintegration respiratory rate, 25/min. He is pale in color
program and it is difficult to find pedal pulses. Which
C. Learning to perform dressing changes action will the nurse take first?
D. Options available for scar removal
Correct A. Start intravenous fluids.
Correct Answer: C. Learning to perform B. Check the pulses using a Doppler
dressing changes device.
C. Obtain a complete blood count (CBC).
Teaching the patient and his family to D. Obtain an electrocardiogram (ECG).
perform dressing changes is critical for the Correct
goal of progression towards independence. Correct Answer: A. Start intravenous fluids.
Proper management of burn injury through
proper dressing changes helps prevent Hypovolemic shock is a common cause of
wound deterioration. Encouragement of the death in the emergent phase of clients with
patient and his family members in serious injuries. Administration of fluids can
participating in dressing changes and treat this problem. For burns classified as
wound care helps prepare for the patient’s severe (> 20% TBSA), fluid resuscitation
eventual discharge and home care needs. should be initiated to maintain urine output
All other choices (below) are important > 0.5 mL/kg/hour.
during the rehabilitation stage but dressing
changes is a priority. Option C: Following a severe burn injury,
significant hematologic changes occur that
Option A: Teach on the importance of are reflected in complete blood count (CBC)
installing and maintaining smoke detectors measurements. A CBC will be taken to
on every level of the home and changing ascertain if a cardiac or bleeding problem is
batteries periodically to help prevent fires. causing these vital signs. However, these
Option B: Surviving a burn injury has a are not actions that the nurse would take
tremendous psychological impact on the immediately.
patient and family. The nurse plays a key Option B: Checking pulses would indicate
role in helping the patient adapt. Providing perfusion to the periphery but this is not an
referrals to social services and counseling immediate nursing action. Carefully check
helps the patient during his rehabilitation pulses in any extremity with circumferential
phase. burns. These burns can act as tourniquets
Option D: Discussion about burn as burn-associated edema begins, leading
reconstruction treatment after the scars to compartment syndrome.
have healed or matured is usually
Option D: In patients with extensive burns, it How will the nurse position a client with a
is sometimes a challenge to monitor the burn wound to the posterior neck to prevent
ECG, because the lack of natural skin and contractures?
application of protective ointments prevent
the adherence of the ECG discs. A. Have the client turn the head from side
3. Question to side.
A 40-year-old male client who was burned B. Keep the client in a supine position
was admitted under your care. Assessment without the use of pillows.
reveals he has crackles, respiratory rate of C. Keep the client in a semi-Fowler's
40/min, and is coughing up blood-tinged position with her or his arms elevated.
sputum. What action will the nurse take D. Place a towel roll under the client's neck
first? or shoulder.
Incorrect
A. Administer digoxin Correct Answer: A. Have the client turn the
B. Perform chest physiotherapy head from side to side.
C. Monitor urine output
D. Place the client in an upright position Deformities and contractures can often be
Correct prevented by proper positioning.
Correct Answer: D. Place the client in an Maintaining proper body alignment when
upright position the patient is in bed is vital. The function
that would be disrupted by a contracture to
Pulmonary edema can result from fluid the posterior neck is flexion. Moving the
resuscitation given for burn treatment. This head from side to side prevents such a loss
can occur even in a young healthy person. of flexion. This movement is what would
Placing the client in an upright position can prevent contractures from occurring.
relieve lung congestion immediately before
other measures can be carried out. Option B: The client should not only be in a
supine position but there should be a
Option A: Digoxin may be given later to movement to avoid contractures. Splinting
increase cardiac contractility to prevent and proper positioning will also help achieve
backup of fluid into the lungs. However, the prevention of contractures. As a matter
digoxin has the potential to cause of importance, movement should be
bradyarrhythmias. incorporated into the patient’s daily routine
Option B: Chest physiotherapy will not get from their inception to the hospital.
rid of the fluid and is not a priority among Option C: The burns are in the client’s
the choices. Chest physiotherapy is only posterior neck. Performing active or passive
applicable during the post-burn range of motion (ROM) exercises,
management of the patient. depending on the patient’s level of
Option C: Monitoring urine output is consciousness is crucial in the prevention of
important. However, it is not an immediate these complications.
intervention. Use the patient’s urine output Option D: Placing a towel roll under the
and physiologic response to determine if the neck might not help prevent contractures.
volume is adequate for resuscitation. Immobilization is only allowed when a part
4. Question of the body has just been grafted. Even
then, the area must be kept in an Ten hours after the client with 50% burns is
antideformity position. admitted, her blood glucose level is 142
5. Question mg/dL. What is the nurse’s best action?
On assessment, the nurse notes that the
client has burns inside the mouth and is A. Documents the finding
wheezing. Several hours later, the wheezing B. Obtains a family history of diabetes
is no longer heard. What is the nurse’s next C. Repeats the glucose measurement
action? D. Stop IV fluids containing dextrose
Correct
A. Documenting the findings Correct Answer: A. Documents the finding
B. Loosening any dressings on the chest
C. Raising the head of the bed Neural and hormonal compensation to the
D. Preparing for intubation stress of the burn injury in the emergent
Correct phase increases liver glucose production
Correct Answer: D. Preparing for intubation and release. An acute rise in the blood
glucose level is an expected client response
Clients with severe inhalation injuries may and is helpful in the generation of energy
sustain such progressive obstruction that needed for the increased metabolism that
they may lose the effective movement of air. accompanies this trauma.
When this occurs, wheezing is no longer
heard and neither are breath sounds. The Option B: A family history of diabetes could
client requires the establishment of an make her more of a risk for the disease, but
emergency airway. The swelling usually this is not a priority at this time. The
precludes intubation. secondary assessment shouldn’t begin until
the primary assessment is complete;
Option A: Documentation of findings should resuscitative efforts are underway; and
be done after the interventions. There may lines, tubes, and catheters are placed.
be only a small window of opportunity to Option C: The glucose level is not high
easily place an ET tube because edema enough to warrant retesting. A variety of
from burn shock may obstruct the airway. laboratory tests will be needed within the
Option B: Loosening any dressings on the first 24 hours of a patient’s admission (some
chest should be done right after the during the initial resuscitative period and
assessment of wheezes. If there is edema others after the patient is stabilized).
or evidence of burn in the upper airways, Option D: The cause of her elevated blood
assessment for whether an endotracheal glucose is not the IV fluid. Rapid and
(ET) tube is needed to maintain the airway aggressive fluid resuscitation is needed to
should be done immediately. replace intravascular volume and maintain
Option C: The head of the bed should be end-organ perfusion.
flat to prepare for intubation. Emergency 7. Question
airway intubation should be done The client has a large burned area on the
immediately after assessment to avoid right arm. The burned area appears pink,
complete obstruction of the airway due to has blisters, and is very painful. How will the
edema. nurse categorize this injury?
6. Question
A. Full-thickness C. Partial-thickness deep
B. Partial-thickness superficial D. Superficial
C. Partial-thickness deep Correct
D. Full-thickness deep Correct Answer: A. Full-thickness
Incorrect
Correct Answer: C. Partial-thickness deep The characteristics of the wounds meet the
criteria for a full-thickness injury: color that
Deep partial-thickness burns are pink or red is black, brown, yellow, white, or red; no
in color, swollen, painful, with blisters that blisters; pain minimal; outer layer firm and
may ooze a clear fluid. Deep partial- inelastic. The burn is leathery and dry.
thickness (second-degree) involves the There is minimal to no pain because of
deeper dermis. Healing occurs in 3 to 8 decreased sensation. Full-thickness burns
weeks with scarring present. heal by contracture and take greater than 8
weeks. Full-thickness burns require skin
Option A: Third-degree involves the full grafting.
thickness of skin and subcutaneous
structures. It appears white or black/brown. Option B: Superficial partial-thickness
With pressure, no blanching occurs. The (second-degree) involves the superficial
burn is leathery and dry. There is minimal to dermis. It appears red with blisters and is
no pain because of decreased sensation. wet. The erythema blanches with pressure.
Option B: The characteristics of the wound The pain associated with superficial partial-
meet the criteria for a superficial partial- thickness is severe. Healing typically occurs
thickness injury: color that is red; without within 3 weeks with minimal scarring.
blisters and pain present. Superficial (first- Option C: Deep partial-thickness (second-
degree) involves the epidermis of the skin degree) involves the deeper dermis. It
only. It appears pink to red, there are no appears yellow or white, is dry, and does
blisters, and it is dry. It is moderately not blanch with pressure. There is minimal
painful. Superficial burns heal without pain due to a decreased sensation. Healing
scarring within 5 to 10 days. occurs in 3 to 8 weeks with scarring
Option D: Blisters are not seen with full- present.
thickness burns and are rarely seen with Option D: Superficial (first-degree) involves
deep partial-thickness burns. Full-thickness the epidermis of the skin only. It appears
burns heal by contracture and take greater pink to red, there are no blisters, and it is
than 8 weeks. Full-thickness burns require dry. It is moderately painful. Superficial
skin grafting. burns heal without scarring within 5 to 10
8. Question days.
The client has burns on both legs. These 9. Question
areas appear white and leather-like. No The client has experienced an electrical
blisters or bleeding are present, and there is injury of the lower extremities. Which are
just a “small amount of pain.” How will the the priority assessment data to obtain from
nurse categorize this injury? this client?

A. Full-thickness A. Current range of motion in all extremities


B. Partial-thickness superficial B. Heart rate and rhythm
C. Respiratory rate and pulse oximetry D. Leg abducted with a foam wedge
reading Correct
D. Orientation to time, place, and person Correct Answer: B. Hip at zero flexion with
Correct leg flat
Correct Answer: B. Heart rate and rhythm.
The maximum function for ambulation
Electric current travels through the body occurs when the hip and leg are maintained
from the entrance site to the exit site and at full extension with neutral rotation.
can seriously damage all tissues between Although the client does not have to spend
the two sites. Early cardiac damage from 24 hours in this position, he or she should
electrical injury includes irregular heart rate, be in this position (in bed or standing)
rhythm, and ECG changes. It is also longer than with the hip in any degree of
important to obtain the patient’s cardiac flexion.
history, including any history of prior
arrhythmias. Option A: Anti-contracture positioning and
splinting must start from day one and may
Option A: Range of motion is also important. continue for many months post-injury. Legs
However, the priority is to make sure that should be positioned in a neutral position
the heart rate and rhythm are adequate to ensuring that the patient is not externally
support perfusion to the brain and other vital rotating at the hips.
organs. Option C: Patients rest in a position of
Option C: The airway is not at any particular comfort; this is generally a position of flexion
risk with this injury. Therefore, respiratory and also the position of contracture. Without
rate and pulse oximetry are not priority ongoing advice and help with positioning,
assessments. Any patient that was in the patient will continue to take the position
contact with a high voltage source should of contracture and can quickly lose ROM in
have continuous cardiac monitoring during multiple joints. Once contracture starts to
evaluation. develop it can be a constant battle to
Option D: These patients are specifically at achieve full movement, so preventative
risk for cardiac damage if the path of the measures to minimize contracture
current traversed the heart. One may also development are necessary.
consider CT imaging of the head if the Option D: Splinting helps maintain anti-
patient has altered mental status or contracture positioning particularly for those
associated head trauma from a fall or being patients experiencing a great deal of pain,
thrown in a blast. difficulty with compliance, or with burns in
10. Question an area where positioning alone is
A 35-year-old male client was admitted due insufficient. If the injured site is over joint
to severe burns around his right hip. Which surfaces, special precautions should be
position is most important to use to maintain taken to identify all possible joint
the maximum function of this joint? contractures.
11. Question
A. Hip maintained in 30-degree flexion The client who is burned is drooling and
B. Hip at zero flexion with leg flat having difficulty swallowing. Which action
C. Knee flexed at 30-degree angle will the nurse take first?
A 22-year-old female client with a full-
A. Assesses level of consciousness and thickness burn is being discharged to home
pupillary reactions after a month in the hospital. Her wounds
B. Ascertains the time food or liquid was are minimally opened and she will be
last consumed receiving home care. Which nursing
C. Auscultates breath sounds over the diagnosis has the highest priority?
trachea and mainstem bronchi
D. Measures abdominal girth and A. Acute Pain
auscultates bowel sounds B. Deficient Diversional Activity
Correct C. Impaired Adjustment
Correct Answer: C. Auscultates breath D. Imbalanced Nutrition: Less than Body
sounds over the trachea and mainstem Requirements
bronchi. Incorrect
Correct Answer: C. Impaired Adjustment
Inhalation injuries are present in 7% of
clients admitted to burn centers. Drooling Recovery from a burn injury requires a lot of
and difficulty swallowing can mean that the work on the part of the client and significant
client is about to lose his airway because of others. The client is seldom restored to her
this injury. The absence of breath sounds pre burn level of functioning. Adjustments to
over the trachea and mainstem bronchi changes in appearance, family structure,
indicates impending airway obstruction and employment opportunities, role, and
demands immediate intubation. functional limitations are only a few of the
numerous life-changing alterations that
Option A: Knowing the level of must be made or overcome by the client.
consciousness is important to assess
oxygenation to the brain. In most cases, Option A: By the rehabilitation phase, acute
neurologic status won’t be altered in the pain from the injury or its treatment is no
early stages of burn injury. Use the Glasgow longer a problem. This stage starts with the
Coma Scale to trend the patient’s closure of the burn and ends when the
neurologic status throughout the patient has reached the optimal level of
resuscitation. functioning. The focus is on helping the
Option B: Ascertaining time of last food patient return to a normal injury-free life.
intake is important in case intubation is Helping the patient adjust to the changes
necessary (the nurse would be more alert the injury has imposed is also a priority.
for the signs of aspiration). However, Option B: Diversional activity for pain is
assessing air exchange is the most applicable during the intermediate phase of
important intervention at this time. the burn injury. Provide diversional activities
Option D: Measuring abdominal girth is not appropriate for age and condition. This
relevant in this situation. If there is edema or helps lessen concentration on pain
evidence of burn in the upper airways, experience and refocus attention.
assess whether an endotracheal (ET) tube Option D: Imbalanced nutrition is more
is needed to maintain the airway. appropriate during the emergent and
12. Question intermediate phases of the burn injury.
Appropriate guides to proper caloric intake
include 25 kcal/kg body weight, plus 40 kcal total body surface area (%TBSA) burned ×
per percentage of TBSA burn in the adult. patient’s weight in kilograms = total amount
As the burn wound heals, the percentage of of fluid given in the first 24 hours.
burned areas is reevaluated to calculate Option D: Cimetidine does not prevent
prescribed dietary formulas, and appropriate kidney damage. Acute renal failure is one of
adjustments are made. the major complications of burns and it is
13. Question accompanied by a high mortality rate. Most
The client with a new burn injury asks the renal failures occur either immediately after
nurse why he is receiving intravenous the injury or at a later period when sepsis
cimetidine (Tagamet). What is the nurse’s develops.
best response? 14. Question
A 12-year-old male with facial burns asks
A. "Tagamet will stimulate intestinal the nurse if he will ever look the same.
movement." Which response is best for the nurse to
B. "Tagamet can help prevent hypovolemic provide?
shock."
C. "This will help prevent stomach ulcers." A. "With reconstructive surgery, you can
D. "This drug will help prevent kidney look the same."
damage." B. "We can remove the scars with the use
Correct of a pressure dressing."
Correct Answer: C. “This will help prevent C. "You will not look exactly the same."
stomach ulcers.” D. "You shouldn't start worrying about your
appearance right now."
Ulcerative gastrointestinal disease may Incorrect
develop within 24 hours after a severe burn Correct Answer: C. “You will not look
as a result of increased hydrochloric acid exactly the same.”
production and decreased mucosal barrier.
This process occurs because of the Many clients have unrealistic expectations
sympathetic nervous system stress of reconstructive surgery and envision an
response. Cimetidine inhibits the production appearance identical or equal in quality to
and release of hydrochloric acid. the preburn state. Denial may be prolonged
and be an adaptive mechanism because the
Option A: Gastrointestinal stimulants are patient is not ready to cope with personal
drugs that increase motility of the problems.
gastrointestinal smooth muscle, without
acting as a purgative. These drugs have Option A: Be realistic and positive during
different mechanisms of action but they all treatments, in health teaching, and in setting
work to move the contents of the goals within limitations. This enhances trust
gastrointestinal tract faster. and rapport between patient and nurse.
Option B: Patients with burns of more than Option B: Pressure dressings prevent
20% – 25% of their body surface should be further scarring. They cannot remove scars.
managed with aggressive IV fluid The client and family should be taught the
resuscitation to prevent “burn shock.” Four expected cosmetic outcomes. Provide hope
mL lactated ringers solution × percentage within the parameters of the individual
situation; do not give false reassurance. Option B: Increasing the temperature in the
This promotes a positive attitude and room may make the client more
provides opportunities to set goals and plan comfortable, but the priority is finding out if
for the future based on reality. the client has sepsis and treating it before it
Option D: Acknowledge and accept the becomes a shock situation.
expression of feelings of frustration, Option C: Increasing the rate of intravenous
dependency, anger, grief, and hostility. Note fluids may be done to replace fluid losses
withdrawn behavior and use of denial. with diarrhea, but is not the priority action.
Acceptance of these feelings as a normal Fluid resuscitation replaces lost fluids and
response to what has occurred facilitates electrolytes and helps prevent complications
resolution. It is not helpful or possible to (shock, acute tubular necrosis).
push the patient before he is ready to deal Replacement formulas vary but are based
with the situation. on the extent of injury, amount of urinary
15. Question output, and weight.
The client with open burn wounds begins to 16. Question
have diarrhea. The client is found to have a The family of a client who has been burned
below-normal temperature, with a white asks at what point the client will no longer
blood cell count of 4000/mm3. Which is the be at greater risk for infection. What is the
nurse’s best action? nurse’s best response?

A. Continuing to monitor the client A. "As soon as he finishes his antibiotic


B. Increasing the temperature in the room prescription."
C. Increasing the rate of the intravenous B. "As soon as his albumin level returns to
fluids normal."
D. Preparing to do a workup for sepsis C. "When fluid remobilization has started."
Correct D. "When the burn wounds are closed."
Correct Answer: D. Preparing to do a Correct
workup for sepsis. Correct Answer: D. “When the burn wounds
are closed.”
These findings are associated with systemic
gram-negative infection and sepsis. To Intact skin is a major barrier to infection and
verify that sepsis is occurring, cultures of other disruptions in homeostasis. No matter
the wound and blood must be taken to how much time has passed since the burn
determine the appropriate antibiotic to be injury, the client remains at high risk for
started. infection as long as any area of skin is
open.
Option A: Continuing just to monitor the
situation can lead to septic shock. Indicators Option A: Even after the course of treatment
of sepsis (often occurs with full-thickness of antibiotics, the patient is still at risk for
burn) requiring prompt evaluation and infection if the wounds remain open.
intervention. Changes in sensorium, bowel Examine wounds daily, note and document
habits, and the respiratory rate usually changes in appearance, odor, or quantity of
precede fever and alteration of laboratory drainage.
studies.
Option B: Albumin levels are monitored if Option A: If there is a fireplace, make sure
there is significant edema. Implement the chimney is checked and cleaned by a
appropriate isolation techniques as professional once a year. Use a metal or
indicated. Depending on the type or extent glass screen that is large enough to prevent
of wounds and the choice of wound escaping embers. Make sure home heating
treatment (open versus closed), isolation sources are clean and in working order.
may range from a simple wound and/or skin Many home fires are started by poorly
to complete or reverse to reduce the risk of maintained furnaces or stoves, cracked or
cross-contamination and exposure to rusted furnace parts, or chimneys with
multiple bacterial flora. creosote buildup.
Option C: Fluid resuscitation replaces lost Option C: Space heaters can be a cause of
fluids and electrolytes and helps prevent fire if clothing, bedding, and other
complications (shock, acute tubular flammable objects are nearby. Make sure to
necrosis). Once initial fluid resuscitation has always keep anything that gives off heat at
been accomplished, a steady rate of fluid least 3 feet away from flammable materials
administration is preferred to boluses, which or items. Heating equipment, like space
may increase interstitial fluid shifts and heaters, are involved in 1 of every 6 home
cardiopulmonary congestion. fires. Furthermore, 1 in every 5 home fire
17. Question deaths and half of all fires caused by home
The nurse is conducting a home safety heating occur between December and
class. It is most important for the nurse to February.
include which information in the teaching Option D: Carbon monoxide detectors
plan? should also be placed inside the house, not
only in the garage. A person can be
A. Have chimneys swept every 2 years. poisoned by a small amount of CO over a
B. Keep a smoke detector in each longer period of time or by a large amount
bedroom. of CO over a shorter amount of time.
C. Use space heaters instead of gas 18. Question
heaters. The nurse provides wound care for a client
D. Use carbon monoxide detectors only in 48 hours after a burn injury. To achieve the
the garage. desired outcome of the procedure, which
Incorrect nursing action will be carried out first?
Correct Answer: B. Keep a smoke detector
in each bedroom. A. Applies silver sulfadiazine (Silvadene)
ointment
Everyone should use smoke detectors and B. Covers the area with an elastic wrap
carbon monoxide detectors in their home C. Places a synthetic dressing over the
environment (just not in a garage). area
Recommendations are that each bedroom D. Removes loose nonviable tissue
has a separate smoke detector. Test smoke Correct
alarms every month. If they’re not working, Correct Answer: D. Removes loose
change the batteries. nonviable tissue
The first step in this process is removing
exudates and necrotic tissue. Burn patients Minor burns are common occurrences. The
are at high risk for infection, especially drug- use of space heaters can cause a fire if
resistant infection, which often results in clothing, bedding, and other flammable
significantly longer hospital stays, delayed objects are near them. Make sure to always
wound healing, higher costs, and higher keep anything that gives off heat at least 3
mortality feet away from flammable materials or
items.
Option A: Since the adoption of topical
antibiotics, such as mafenide in the 1960s Option A: Chimneys should be swept each
and silver sulfadiazine in the 1970s, and of year to prevent creosote build-up and
early excision and grafting in the 1970s and resultant fire. If there is a fireplace, make
thereafter, systemic infections and mortality sure the chimney is checked and cleaned
have consistently decreased. However, by a professional once a year. Use a metal
Gram-positive and Gram-negative bacterial or glass screen that is large enough to
infections still remain one of the most prevent escaping embers.
common causes of mortality following burn Option B: Burn injuries do not commonly
injury. occur from microwave cooking, but rather
Option B: While many factors must be when taking food from it. Thermal burns are
considered in dressing selection, the goals skin injuries caused by excessive heat,
in selecting the most appropriate dressing typically from contact with hot surfaces, hot
should include providing protection from liquids, steam, or flame. Most burns are
contamination (bacterial or otherwise) and minor and patients can be treated as
from physical damage, allowing gas outpatients or at local hospitals.
exchange and moisture retention, and Option C: Lastly, sunscreen agents are
providing comfort to enhance functional recommended to prevent sunburn. A broad-
recovery. spectrum sunscreen with an SPF of at least
Option C: The selection of an appropriate 30 should be applied 30 minutes before sun
dressing depends on several factors, exposure and every 90 minutes after that.
including depth of burn, condition of the 20. Question
wound bed, wound location, desired The nurse uses topical gentamicin sulfate
moisture retention and drainage, required (Garamycin) on a client’s burn injury. Which
frequency of dressing changes, and cost. laboratory value will the nurse monitor?
19. Question
The nurse should teach the community that A. Creatinine
a minor burn injury could be caused by what B. Red blood cells
common occurrence? C. Sodium
D. Magnesium level
A. Chimney sweeping every year Correct
B. Cooking with a microwave oven Correct Answer: A. Creatinine
C. Use of sunscreen agents
D. Use of space heaters Gentamicin is nephrotoxic and sufficient
Incorrect amounts can be absorbed through burn
Correct Answer: D. Use of space heaters wounds to affect kidney function. Any client
receiving gentamicin by any route should D. Wash hands on entering the client's
have kidney function monitored. In burn room.
patients, the systemic absorption of topical Correct
gentamicin may be enhanced, and one Correct Answer: D. Wash hands on entering
should be watchful for the potential the client’s room.
repercussions.
Infection can occur when microorganisms
Option B: Topical gentamicin will not affect from another person or the environment are
the red blood cell count. The gentamicin is transferred to the client. Handwashing with
prone to accumulate in the renal proximal soap and water is the best way to get rid of
tubular cells and can cause damage. germs in most situations. Emphasize and
Hence, mild proteinuria and reduction of the model good handwashing techniques for all
glomerular filtration rate are potential individuals coming in contact with the
consequences of gentamicin use, achieving patient.
14% of gentamicin users in a review.
Option C: Topical gentamicin will not affect Option A: Tissue destruction and altered
sodium. In cases of renal impairment, defense mechanisms increase the risk of
dosing adjustment should be made based developing tetanus or gas gangrene,
on the glomerular filtration rate (GFR); for a especially in deep burns such as those
high-dose, extended interval dosing caused by electricity.
approach, the dose can be preserved, but Option B: Examine wounds daily, note and
the interval between doses should increase document changes in appearance, odor, or
in line with GFR decrease. quantity of drainage. Indicators of sepsis
Option D: Topical gentamicin will not affect (often occurs with full-thickness burn)
the magnesium level. Renal function should requiring prompt evaluation and
be evaluated twice-weekly in patients intervention.
without previous renal disease through Option C: Although all the interventions
serum creatinine and blood urea nitrogen. listed can help reduce the risk of infection,
Periodic microscopic urinalysis is also vital hand washing is the most effective
to detect proteinuria and casts, which may technique for preventing infection
indicate kidney injury. transmission. Airway obstruction and/or
respiratory distress can occur very quickly
1. Question or may be delayed, e.g., up to 48 hr after a
The RN has assigned a client who has an burn.
open burn wound to the LPN. Which 2. Question
instruction is most important for the RN to Three days after a burn injury, the client
provide the LPN? develops a temperature of 100° F, a white
blood cell count of 15,000/mm3, and a
A. Administer the prescribed tetanus toxoid white, foul-smelling discharge from the
vaccine. wound. The nurse recognizes that the client
B. Assess wounds for signs of infection. is most likely exhibiting symptoms of which
C. Encourage the client to cough and condition?
breathe deeply.
A. Acute phase of the injury
B. Autodigestion of collagen D. Repositions the client onto the right side
C. Granulation of burned tissue Correct
D. Wound infection Correct Answer: B. Documents the finding
Correct
Correct Answer: D. Wound infection Decreased or absent peristalsis is an
expected response during the emergent
Color change, purulent, foul-smelling phase of burn injury as a result of neural
drainage, increased white blood cell count, and hormonal compensation to the stress of
and fever could all indicate infection. injury. No currently accepted intervention
Indicators of sepsis (often occurs with full- changes this response. It is not the highest
thickness burn) requiring prompt evaluation priority of care at this time.
and intervention. Changes in sensorium,
bowel habits, and the respiratory rate Option A: Do not give the patient laxative.
usually precede fever and alteration of The emergent phase starts with the onset of
laboratory studies. burn injury and lasts until the completion of
fluid resuscitation or a period of about the
Option A: These symptoms will not be seen first 24 hours. During the emergent phase,
in the acute phase of the injury. Assess and the priority of patient care involves
document size, color, depth of wound, maintaining an adequate airway and
noting necrotic tissue and condition of the treating the patient for burn shock.
surrounding skin. Option C: Increased capillary permeability,
Option B: Autodigestion of collagen will not protein shifts, inflammatory process, and
increase the body temperature or cause evaporative losses greatly affect circulating
foul-smelling wound discharge. Monitor vital volume and urinary output, especially during
signs for fever, increased respiratory rate the initial 24–72 hr after burn injury. Fluid
and depth in association with changes in replacement formulas partly depend on
sensorium, presence of diarrhea, decreased admission weight and subsequent changes.
platelet count, and hyperglycemia with Option D: Maintain proper body alignment
glycosuria. with supports or splints, especially for burns
Option C: Granulation of tissue will not over joints. This promotes functional
increase the body temperature or cause positioning of extremities and prevents
foul-smelling wound discharge. Examine contractures, which are more likely over
wounds daily, note and document changes joints.
in appearance, odor, or quantity of 4. Question
drainage. What intervention will the nurse implement
3. Question to reduce a client’s pain after a burn injury?
Twelve hours after the client was initially
burned, bowel sounds are absent in all four A. Administering morphine 4 mg
abdominal quadrants. Which is the nurse’s intravenously.
best action? B. Administering hydromorphone (Dilaudid)
4 mg intramuscularly.
A. Administers a laxative C. Applying ice to the burned area
B. Documents the finding D. Avoiding tactile stimulation
C. Increases the IV flow rate Correct
Correct Answer: A. Administering morphine D. "I am still at risk for infection after the
4 mg intravenously. procedure."
Incorrect
Drug therapy for pain management requires Correct Answer: A. “Because the graft is my
opioid and nonopioid analgesics. The own skin, there is no chance it won’t ‘take.'”
burned patient may require around-the-
clock medication and dose titration. IV Factors other than tissue type, such as
method is often used initially to maximize circulation and infection, influence whether
drug effect. and how well a graft will work. The client
should be prepared for the possibility that
Option B: The IV route is used because of not all grafting procedures will be
problems with absorption from the muscle successful. Graft survival depends on the
and stomach. Concerns of patient addiction diffusion of nutrients and oxygen from the
or doubts regarding the degree of pain wound bed known as imbibition.
experienced are not valid during the Inosculation then follows when the blood
emergent/acute phase of care, but narcotics vessels of the graft and from the wound bed
should be decreased as soon as feasible grow together to make end-to-end contact.
and alternative methods for pain relief Lastly, neovascularization occurs when new
initiated. blood vessels grow from the wound bed into
Option C: For the client to avoid shivering, the graft.
the room must be kept warm and heat
should be applied. Maintain comfortable Option B: The donor sites will be painful
environmental temperature, provide heat after the surgery. Silicone gel sheets, along
lamps, heat-retaining body coverings. with pressure dressings, have shown a
Temperature regulation may be lost with dramatic decrease in pain, pruritis, and scar
major burns. External heat sources may be thickness six months after burn injury.
necessary to prevent chilling. Option C: There can be scarring in the area
Option D: Tactile stimulation can be used where the skin is removed for grafting. Burn
for pain management. Provide basic comfort scars are a common occurrence after skin
measures: massage of uninjured areas, grafting and can cause anxiety, depression,
frequent position changes. This promotes pain, itching, altered pigmentation,
relaxation and reduces muscle tension and temperature intolerance, and decreased
general fatigue. range of motion secondary to scar
5. Question contracture. Scar formation is propagated
What statement indicates the client needs by deficiencies in the biosynthetic and
further education regarding skin grafting tissue degradation pathway during wound
(allografting)? healing.
Option D: The client is still at risk for
A. "Because the graft is my own skin, there infection. Early failure of graft survival is
is no chance it won't 'take." attributable to seroma and hematoma
B. "For the first few days after surgery, the formation, which lifts the graft off the wound
donor sites will be painful." bed, preventing imbibition. Other factors
C. "I will have some scarring in the area that lead to graft failure include shearing
when the skin is removed for grafting."
forces, edematous tissue, and infected separately; do not allow burned ear to touch
tissue. scalp). This identifies the presence of
6. Question healing (granulation tissue) and provides for
When providing care for a client with an early detection of burn-wound infection.
acute burn injury, which nursing intervention Infection in a partial-thickness burn may
is most important to prevent infection by cause conversion of burn to full-thickness
auto contamination? injury.
Option D: Emphasize and model good
A. Avoiding sharing equipment such as handwashing techniques for all individuals
blood pressure cuffs between clients coming in contact with the patient. This
B. Changing gloves between wound care prevents cross-contamination and reduces
on different parts of the client's body the risk of acquired infection.
C. Using the closed method of burn wound 7. Question
management Which assessment finding assists the nurse
D. Using proper and consistent in confirming inhalation injury?
handwashing
Incorrect A. Brassy cough
Correct Answer: B. Changing gloves B. Decreased blood pressure
between wound care on different parts of C. Nausea
the client’s body. D. Headache
Correct
Autocontamination is the transfer of Correct Answer: A. Brassy cough
microorganisms from one area to another
area of the same client’s body, causing Brassy cough and wheezing are some signs
infection of a previously uninfected area. seen with inhalation injury. Damage to
Use gowns, gloves, masks, and strict airway tissue causes increased mucus
aseptic techniques during direct wound care production, edema, denudation of
and provide sterile or freshly laundered bed epithelium, and mucosal ulceration and
linens or gowns. hemorrhage. Obstruction of airflow is often
the effect caused by tissue edema
Option A: Although all techniques listed can narrowing the passageways and
help reduce the risk of infection, only mucus/blood/fluid impeding airflow.
changing gloves between carrying out
wound care on different parts of the client’s Option B: Patients with carbon monoxide
body can prevent auto contamination. poisoning may exhibit hypotension. As
Depending on the type or extent of wounds carboxyhemoglobin (COHgb) levels rise, the
and the choice of wound treatment (open cerebral blood vessels dilate, and both
versus closed), isolation may range from a coronary blood flow and capillary density
simple wound and/or skin to complete or increase. Cardiac effects, especially
reverse to reduce the risk of cross- ventricular arrhythmias occur. Ventricular
contamination and exposure to multiple arrhythmias are implicated as the cause of
bacterial flora. death most often in CO poisoning.
Option C: Prevent skin-to-skin surface Option C: Most commonly, patients with
contact (wrap each burned finger or toe carbon monoxide poisoning will present with
headache (more than 90%), dizziness, research data are up to two years post-
weakness, and nausea. Patients may be injury, the heart rate of severely burned
tachycardic and tachypneic. children was still 120% of predicted
Option D: Patients may have systemic compared to normal values for children.
symptoms like a headache, delirium, Option C: Swelling tends to occur soon after
hallucinations, and may even be comatose. injury and generally decreases after 48–72
Many different etiologies may cause hours, although this timescale can vary. The
changes in mental status including hypoxia, extent and location of the swelling will
hypercarbia, or asphyxiant exposure depend on how the burn was caused and
(carbon monoxide, hydrogen cyanide). But the location and depth of the burn injury. It
headaches can also be seen with carbon is very important that the swelling is
monoxide poisoning. reduced as soon as possible.
8. Question 9. Question
Which finding indicates that fluid Which finding indicates to the nurse that a
resuscitation has been successful for a client with a burn injury has a positive
client with a burn injury? perception of his appearance?

A. Hematocrit = 60% A. Allowing family members to change his


B. Heart rate = 130 beats/min dressings
C. Increased peripheral edema B. Discussing future surgical reconstruction
D. Urine output = 50 mL/hr C. Performing his own morning care
Incorrect D. Wearing the pressure dressings as
Correct Answer: D. Urine output = 50 mL/hr ordered
Correct
The fluid remobilization phase improves Correct Answer: C. Performing his own
renal blood flow, increases diuresis, and morning care
restores blood pressure and heart rate to
more normal levels, as well as laboratory Indicators that the client with a burn injury
values. This phase occurs on days 1-3 and has a positive perception of his appearance
requires an accurate fluid resuscitation and include the willingness to touch the affected
thorough evaluation for other injuries and body part. Self-care activities such as
comorbid conditions. morning care foster feelings of self-worth,
which are closely linked to body image.
Option A: Hematocrit can indirectly reflect
the resuscitation effect in the burn shock Option A: Encourage the patient and SO to
stage. Whether hematocrit level can be view wounds and assist with care as
lowered to 0.45-0.50 during the first 24 appropriate. This promotes acceptance of
hours after burn may be an important index the reality of injury and of change in body
for evaluation of fluid resuscitation effect in and image of self as different.
the early shock stage after severe burn. Option B: Discussing future reconstruction
Option B: The average daily heart rate was would not indicate a positive perception of
elevated in burn patients up to two years appearance. Assist the patient to identify
post-burn. Heart rate was elevated despite the extent of actual change in appearance
any afforded resuscitative efforts. While and body function. This helps begin the
process of looking to the future and how life push the patient before ready to deal with
will be different. the situation.
Option D: Wearing the dressing will assist in Option D: Assist the patient to identify the
decreasing complications, but will not extent of actual change in appearance and
increase self-perception. However, give body function. Feelings of grief, loss,
positive reinforcement of progress and anxiety, anger, fear, and guilt are all normal
encourage endeavors toward attainment of feelings that can occur. Clients need to
rehabilitation goals. Words of know that problems of physical care and
encouragement can support the psychological stresses may be
development of positive coping behaviors. overwhelming.
10. Question 11. Question
Which finding indicates to the nurse that the Which finding is characteristic during the
client understands the psychosocial impact emergent period after a deep full-thickness
of his severe burn injury? burn injury?

A. "It is normal to feel depressed." A. Blood pressure of 170/100 mm Hg


B. "I will be able to go back to work B. Foul-smelling discharge from wound
immediately." C. Pain at site of injury
C. "I will not feel anger about my situation." D. Urine output of 10 mL/hr
D. "Once I get home, things will be normal." Correct
Correct Correct Answer: D. Urine output of 10 mL/hr
Correct Answer: A. “It is normal to feel
depressed.” During the fluid shift of the emergent period,
blood flow to the kidney may not be
During the recovery period, and for some adequate for glomerular filtration. As a
time after discharge from the hospital, result, urine output is greatly decreased.
clients with severe burn injuries are likely to Urine output of 0.5 mL/kg or about 30 – 50
have psychological problems that require mL/hr in adults and 0.5-1.0 mL/kg/hr in
intervention. Depression is one of these children less than 30kg is a good target for
problems. adequate fluid resuscitation.

Option B: Assist the patient and the family Option A: Blood pressure is usually low.
to express their feelings of grief and guilt. During this inflammatory response, there is
The patient and the family may initially be a fluid loss that can cause a sharp and
most concerned about the patient’s death potentially deadly drop in blood pressure
and/or feel guilty, believing that in some way known as shock.
they could have prevented the incident. Option B: A foul-smelling discharge does
Option C: Acknowledge and accept the not occur during the emergent phase. Third-
expression of feelings of frustration, degree involves the full thickness of skin
dependency, anger, grief, and hostility. Note and subcutaneous structures. It appears
withdrawn behavior and use of denial. white or black/brown. With pressure, no
Acceptance of these feelings as a normal blanching occurs. The burn is leathery and
response to what has occurred facilitates dry.
resolution. It is not helpful or possible to
Option C: Pain does not occur with deep Option D: Chemical burns do not cause
full-thickness burns. There is minimal to no inhalation injury and a disrupted breathing
pain because of decreased sensation. Full- pattern. The most common findings
thickness burns heal by contracture and represent structural changes to the tissue
take greater than 8 weeks. Full-thickness directly affected, for example, the eye, oral
burns require skin grafting. mucosa, skin, esophagus, and lower
12. Question intestinal system, especially the stomach
Which is the priority nursing diagnosis and pylorus, respiratory system, among
during the first 24 hours for a client with others.
chemical burns to the legs and arms that 13. Question
are red in color, edematous, and without Which laboratory result, obtained on a client
pain? 24 hours post-burn injury, will the nurse
report to the physician immediately?
A. Decreased Tissue Perfusion
B. Disturbed Body Image A. Arterial pH, 7.32
C. Risk for Disuse Syndrome B. Hematocrit, 52%
D. Risk for Ineffective Breathing Pattern C. Serum potassium,7.5 mmol/L (mEq/L)
Correct D. Serum sodium, 131 mmol/L (mEq/L)
Correct Answer: A. Decreased Tissue Correct
Perfusion Correct Answer: C. Serum potassium,7.5
mmol/L (mEq/L)
During the emergent phase, fluid shifts into
interstitial tissue in burned areas. When the The serum potassium level is changed to
burn is circumferential on an extremity, the the degree that serious life-threatening
swelling can compress blood vessels to responses could result. With such a rapid
such an extent that circulation is impaired rise in the potassium level, the client is at
distal to the injury, causing decreased tissue high risk of experiencing severe cardiac
perfusion and necessitating the intervention dysrhythmias and death.
of an escharotomy.
Option A: Acid-base studies were carried
Option B: Disturbed body image can out on 76 consecutive burn patients
develop. Assist the patient to identify the admitted within 36 hours of injury.
extent of actual change in appearance and Admission blood pH and base excess (BE)
body function. This helps begin the process values all decreased in a linear relationship
of looking to the future and how life will be to the extent of the burn. Blood Pco-2
different. changes were unrelated to the extent of the
Option C: Disuse syndrome can develop. burn. Significant acidosis developed within 2
Risk for disuse syndrome may be related to hours of burn injury.
the physiological changes brought about by Option B: The hematocrit (Hct) is the
physical inactivity. These changes may percentage of the volume of the whole
include a decrease in muscle strength, blood that is made up of red blood cells. In
limited joint movement, and loss of bone burns, the patient has lost a lot of fluid from
density. However, this is not a priority leaky blood vessels. There are more red
diagnosis at this time. cells than fluid so the hematocrit is high.
Option D: Serum sodium is abnormal, but Option C: Clients who can eat solid foods
not to the same degree of severity, and should ingest as many calories as possible.
would be expected in the emergent phase Appropriate guides to proper caloric intake
after a burn injury. Severe cutaneous include 25 kcal/kg body weight, plus 40 kcal
injuries such as burn injuries and blast per percentage of TBSA burn in the adult.
injuries result in the loss of both water and As the burn wound heals, the percentage of
sodium. For burn patients, hypernatremia burned areas is reevaluated to calculate
that occurs within a few days of injury may prescribed dietary formulas, and appropriate
be associated with an increased risk of adjustments are made.
death. Option D: The nurse needs to work with a
14. Question nutritionist to provide a high-calorie, high-
Which nursing intervention is likely to be protein diet to help with wound healing.
most helpful in providing adequate nutrition Refer to a dietitian or nutrition support team.
while the client is recovering from a thermal This may be useful in establishing individual
burn injury? nutritional needs (based on weight and body
surface area of injury) and identifying
A. Allowing the client to eat whenever he or appropriate routes.
she wants 15. Question
B. Beginning parenteral nutrition high in Which statement best exemplifies the
calories client’s understanding of rehabilitation after
C. Limiting calories to 3000 kcal/day a full-thickness burn injury?
D. Providing a low-protein, high-fat diet
Incorrect A. "I am fully recovered when all the
Correct Answer: A. Allowing the client to eat wounds are closed."
whenever he or she wants. B. "I will eventually be able to perform all
my former activities."
Clients should request food whenever they C. "My goal is to achieve the highest level
think that they can eat, not just according to of functioning that I can."
the hospital’s standard meal schedule. D. "There is never full recovery from a
Ascertain food likes and dislikes. Encourage major burn injury."
SO to bring food from home, as appropriate. Correct
This provides the patient or SO a sense of Correct Answer: C. “My goal is to achieve
control; enhances participation in care and the highest level of functioning that I can.”
may improve intake.
Although a return to pre-burn functional
Option B: Parenteral nutrition may be given levels is rarely possible, burned clients are
as a last resort because it is invasive and considered fully recovered or rehabilitated
can lead to infectious and metabolic when they have achieved their highest
complications. Total parenteral nutrition possible level of physical, social, and
(TPN) maintains nutritional intake and emotional functioning. The technical
meets metabolic needs in presence of rehabilitative phase of rehabilitation begins
severe complications or sustained with wound closure and ends when the
esophageal or gastric injuries that do not client returns to her or his highest possible
permit enteral feedings. level of functioning.
to prevent hypertrophic scarring and
Option A: The final stage in caring for a contractures from forming. Hypertrophic
patient with a burn injury is the rehabilitative burn scars pose a challenge for burn
stage. This stage starts with the closure of survivors and providers. In many cases,
the burn and ends when the patient has they can severely limit a burn survivor’s
reached the optimal level of functioning. The level of function, including work and
focus is on helping the patient return to a recreational activities.
normal injury-free life. Helping the patient
adjust to the changes the injury has Option B: Although the mask does provide
imposed is also a priority. protection of sensitive, newly healed skin
Option B: Early compliance is essential to and grafts from sun exposure, this is not the
ensure the best possible long-term outcome purpose of wearing the mask. A widespread
and also to ease pain and assist with modality of prevention and treatment of
exercise regimes. Patients need to adhere hypertrophic scarring is the utilization of
to a positioning regime in the early stages of pressure garment therapy (PGT).
healing and this takes teamwork and Option C: The pressure garment will not
dedication. alter the risk of infection. At present, PGT is
Option D: Rehabilitation of burns patients is the standard first-line therapy for
a continuum of active therapy starting from hypertrophic burn scars in many centers
admission. There should be no delineation due to its non-invasive characteristics and
between an ‘acute phase’ and a presumed desirable treatment effects with
‘rehabilitation phase’ as this idea can few associated complications.
promote the inequality of secondary Option D: Scars will still be present. This
disjointed scar management and/or treatment modality continues to be a
functional rehabilitation team. clinically accepted practice. It is the most
16. Question common therapy used for the treatment and
Which statement indicates that a client with prevention of abnormal scars after burn
facial burns understands the need to wear a injury particularly in North America, Europe,
facial pressure garment? and Scandinavia where it is considered
routine practice and regarded as the
A. "My facial scars should be less severe preferred conservative management with
with the use of this mask." reported thinning and better pliability
B. "The mask will help protect my skin from ranging from 60% to 85%.
sun damage." 17. Question
C. "This treatment will help prevent The client with a dressing covering the neck
infection." is experiencing some respiratory difficulty.
D. "Using this mask will prevent scars from What is the nurse’s initial action?
being permanent."
Incorrect A. Administer oxygen.
Correct Answer: A. “My facial scars should B. Loosen the dressing.
be less severe with the use of this mask.” C. Notify the emergency team.
D. Document the observation as the only
The purpose of wearing the pressure action.
garment over burn injuries for up to 1 year is Incorrect
Correct Answer: B. Loosen the dressing B. Presence of small, pale pink bumps in
the wound beds
Respiratory difficulty can arise from external C. Decreased white blood cell count
pressure. The first action in this situation D. Increased serum creatinine level
would be to loosen the dressing and then Correct
reassess the client’s respiratory status. Correct Answer: D. Increased serum
Generally, it is recommended that pressure creatinine level
should be maintained between 20 and 30
mm Hg, which is above capillary pressure Gentamicin is nephrotoxic and sufficient
but less than what would diminish peripheral amounts can be absorbed through burn
blood circulation. wounds to affect kidney function. Any client
receiving gentamicin by any route should
Option A: It is unnecessary to administer have kidney function monitored.
oxygen. Wearing pressure garments is Characteristically, gentamicin reaches high
uncomfortable and challenging; problems concentrations in the renal cortex and the
with movement, appearance, fit, comfort, inner ear.
swelling of extremities, rashes, and
blistering are common; consequently, low Option A: Gentamicin does not stimulate
compliance with PGT is to be expected. pain in the wound. The gentamicin is prone
Option C: The nurse may intervene first. to accumulate in the renal proximal tubular
However, monitoring of pressure exerted by cells and can cause damage. Hence, mild
pressure garments is currently difficult and proteinuria and reduction of the glomerular
time-consuming, and not routinely done and filtration rate are potential consequences of
currently, the optimal pressure magnitude gentamicin use, achieving 14% of
for PGT remains unsolved. gentamicin users in a review.
Option D: The nurse may loosen the Option B: The small, pale pink bumps in the
dressing to help the client breathe. Recent wound bed are areas of re-epithelialization
evidence suggests that pressure garment and not an adverse reaction. Renal function
therapy is effective for the prevention and/or should be evaluated twice-weekly in
treatment of abnormal scarring after burn patients without previous renal disease
injury but that the clinical benefit is restricted through serum creatinine and blood urea
to those patients with moderate or severe nitrogen. Periodic microscopic urinalysis is
scarring. also vital to detect proteinuria and casts,
18. Question which may indicate kidney injury.
During the acute phase, the nurse applied Option C: The possible hypersensitivity
gentamicin sulfate (topical antibiotic) to the manifestations of gentamicin are urticaria,
burn before dressing the wound. The client eosinophilia, delayed-type hypersensitivity
has all the following manifestations. Which reaction (Stevens-Johnson syndrome and
manifestation indicates that the client is toxic epidermal necrolysis), angioedema,
having an adverse reaction to this topical and anaphylactic shock. The clinical
agent? manifestations should guide the treatment
strategy.
A. Increased wound pain 30 to 40 minutes 19. Question
after drug application
Which intervention is most important to use early detection of burn-wound infection.
to prevent infection by autocontamination in Infection in a partial-thickness burn may
the burned client during the acute phase of cause conversion of burn to full-thickness
recovery? injury.
Option D: Emphasize and model good
A. Changing gloves between wound care handwashing techniques for all individuals
on different parts of the client’s body. coming in contact with the patient. This
B. Avoiding sharing equipment such as prevents cross-contamination and reduces
blood pressure cuffs between clients. the risk of acquired infection.
C. Using the closed method of burn wound 20. Question
management. The burned client relates the following
D. Using proper and consistent history of previous health problems. Which
handwashing. one should alert the nurse to the need for
Correct alteration of the fluid resuscitation plan?
Correct Answer: A. Changing gloves
between wound care on different parts of A. Seasonal asthma
the client’s body B. Hepatitis B 10 years ago
C. Myocardial infarction 1 year ago
Autocontamination is the transfer of D. Kidney stones within the last 6 month
microorganisms from one area to another Incorrect
area of the same client’s body, causing Correct Answer: C. Myocardial infarction 1
infection of a previously uninfected area. year ago.
Use gowns, gloves, masks, and strict
aseptic techniques during direct wound care It is likely the client has a diminished cardiac
and provide sterile or freshly laundered bed output as a result of the old MI and would
linens or gowns. be at greater risk for the development of
congestive heart failure and pulmonary
Option B: Although all techniques listed can edema during fluid resuscitation.
help reduce the risk of infection, only
changing gloves between carrying out Option A: Asthma is a chronic inflammatory
wound care on different parts of the client’s disease of the airways, characterized by
body can prevent autocontamination. recurrent episodes of airflow obstruction
Depending on the type or extent of wounds resulting from edema, bronchospasm, and
and the choice of wound treatment (open increased mucus production. Commonly
versus closed), isolation may range from a associated with seasonal allergies (allergic
simple wound and/or skin to complete or rhinitis) and eczema (atopic dermatitis),
reverse to reduce the risk of cross- these three conditions form what is known
contamination and exposure to multiple as the atopic triad.
bacterial flora. Option B: The incubation period of an acute
Option C: Prevent skin-to-skin surface hepatitis B virus infection is approximately
contact (wrap each burned finger or toe 12 weeks, with a majority of patients
separately; do not allow burned ear to touch experiencing mild illness and less than 1%
scalp). This identifies the presence of experiencing fulminant hepatic failure. After
healing (granulation tissue) and provides for acute infection resolves, the majority of
adult patients and a small percentage of burn is essentially sterile, and every attempt
infected infants develop antibodies against should be made to keep it so. The burn
the hepatitis B surface antigen and end up wound should be thoroughly cleaned with
recovering fully. soap and water or mild antibacterial wash
Option D: Nephrolithiasis, or kidney stones, such as dilute chlorhexidine.
is the most common condition affecting the Obtain a sample for wound culture. Burn
urinary system, affecting about 12% of the wound infections are one of the most
world population, with a yearly incidence of important and potentially serious
600,000 in America. It is the result of a complications that occur in the acute period
crystal or crystalline concretion traveling following injury
from the kidney through the genitourinary An antibacterial cream such as silver nitrate
system. is applied to the area to attain the maximum
effect of the medication. Flamazine is silver
1. Question sulfadiazine cream and is applied topically
Nurse Malcolm is performing a sterile on the burn wound. It is effective against
dressing change on a client with a gram-negative bacteria including
superficial partial-thickness burn on the Pseudomonas.
shoulder and back. Arrange the steps in the Lastly, cover the wound using a sterile
order in which each should be performed. dressing. Depending on how healing is
progressing, dressing changes thereafter
Administer Tramadol (Tramal) 50 mg IV. should be every three to five days. If the
Debride the wound of eschar using gauze Jelonet dressing has become adherent, it
sponges. should be left in place to avoid damage to
Obtain a sample for wound culture. the delicate healing epithelium. If Flamazine
Apply silver nitrate ointment. is used it should be changed on alternate
Cover the wound using a sterile gauze days.
dressing. 2. Question
Incorrect Which of the following medications given to
The correct order is shown above a 12-year-old client for the treatment of
deep partial-thickness burn is the most
Rationale: important to double-check with another
licensed nurse before administering it?
Pain medication is administered prior to the
dressing change since the type of burn will A. Aloe Vera Relief Burn spray.
be painful during the procedure. Opioids B. Silver Sulfadiazine ointment.
may be required initially to control pain, but C. Omeprazole 20 mg slow IV push.
once first aid measures have been effective D. Amitriptyline (Elavil) 50 mg PO.
non-steroidal anti-inflammatory drugs such Incorrect
as ibuprofen or co-dydramol taken orally will Correct Answer: D. Amitriptyline (Elavil) 50
suffice. mg PO.
Then the wound is debrided before getting
the sample for culture to prevent other Amitriptyline (Elavil) is useful in the
bacteria that can contaminate the actual management of neuropathic pain following
wound. It is important to realize that a new burn injury and since it is an antidepressant
if given with a child, utmost precaution is Correct Answer: C. Urine output
given. The FDA has issued a black box
warning regarding the use of amitriptyline in Of all the options, urine output is the most
adolescents and young adults (ages less reliable indicator for determining the
than 24 years). It can increase the risk of adequacy of fluid resuscitation. Urine output
suicidal ideation and behavior. of 0.5 mL/kg or about 30 – 50 mL/hr in
adults and 0.5-1.0 mL/kg/hr in children less
Option A: Omeprazole is indicated for the than 30kg is a good target for adequate fluid
short-term treatment of peptic ulcer disease resuscitation.
in adults where most patients heal within
four weeks. Patients with duodenal ulcer Option A: Heart rate, mental status, and
disease and H. pylori infection disease that capillary refill may be affected by the
is active for up to one year may benefit from underlying disease process and are less
combination therapy that includes reliable markers. The actual endpoint of
omeprazole with clarithromycin, amoxicillin, fluid therapy in shock is to optimize tissue
and metronidazole. perfusion. However, this parameter is not
Option B: Silver sulfadiazine is a medication measured directly. Surrogate endpoints
used in the prevention, management, and include clinical indicators of end-organ
treatment of burn wound infections. It is a perfusion and measurements of preload.
heavy metal topical agent with antibacterial Option B: Because of compensatory
properties. Typically burn dressings consist vasoconstriction, mean arterial pressure
of topical silver sulfadiazine combined with (MAP) is only a rough guideline; organ
fine mesh gauze and are usable in both the hypoperfusion may be present despite
inpatient and outpatient settings. apparently normal values. Because urine
Option C: All health facilities practice output does not provide a minute-to-minute
double-checking of medications prior to indication, measures of preload may be
administration, Of all the medications given, helpful in guiding fluid resuscitation for
Amitriptyline is the most important to critically ill patients.
double-check with another licensed nurse. Option D: Patient’s vital signs, mental
3. Question status, capillary refill, and urine output must
The nurse is administering fluids be monitored and fluid rates adjusted
intravenously as ordered to a client who accordingly. Recent literature has raised
acquired a full-thickness burn injury on the concerns about complications from over-
abdomen. To determine the sufficiency of resuscitation described as “fluid creep.”
fluid resuscitation, the nurse would monitor Again, adequate fluid resuscitation is the
which of the following would provide the goal.
most reliable parameter for determining 4. Question
adequacy? Nurse Rodrigo is receiving an endorsement
from the burn unit. Which of the following
A. Level of consciousness clients should he assess first?
B. Peripheral pulses
C. Urine output A. A client who has just been transferred
D. Vital signs from the PACU after having allograft.
Incorrect
B. A client admitted 1 week ago with a preserve residual elements capable of
superficial-thickness burn on the buttocks regeneration. Burns of the face and neck
which has been waiting for 2 hours to rarely fall into aesthetic units, and
receive discharge instructions. professional judgment must determine how
C. A client who has just arrived from the the excision should be performed. Grafting
emergency department with burns on the should be in aesthetic units if at all possible,
neck and chest. and sheet grafts should be used on the
D. A client with deep partial-thickness face.
burns on both thighs who is complaining of 5. Question
severe and continuous pain. Which of the following route should the
Correct nurse expect the pain medication to be
Correct Answer: C. A client who has just given to a client who was admitted with
arrived from the emergency department with extensive burns?
burns on the neck and chest.
A. Oral
Burns of the neck and chest are associated B. Intramuscular
with inflammation and swelling of the C. Subcutaneous
airway. Hence this patient requires the most D. Intravenous
immediate attention. Although a patient may Correct
be capable of spontaneous breathing in the Correct Answer: D. Intravenous
early hours after a burn, compromise of the
airway can still develop. This may be due to For clients with major burns, the
external pressure on the airway, with edema intravascular route is the preferred choice of
developing in the head and neck region, or medication administration. Patient-
in the upper airway due to inhalation of hot controlled analgesia (PCA) with IV opioids is
gases, including steam or aspiration of hot a safe and efficient method of achieving
liquids. flexible analgesia in burn-injured patients.
Studies comparing PCA with other routes of
Option A: Split-thickness grafts can cover administration have shown mixed results as
the extensive defects created after scar to benefit and patient satisfaction.
release. These grafts will need meticulous
attention to achieve complete and early Option A: Oral NSAIDs and acetaminophen
wound closure, but thereafter prolonged are mild analgesics that exhibit a ceiling
splinting will be important to maintain effect in their dose-response relationship.
release and prevent contracture. Such limitations render these agents
Option B: In the superficial burn, wound unsuitable for the treatment of typical,
dressings suffice. These can be biological severe burn pain. Oral NSAIDS and
or synthetic. Some medicated elements can acetaminophen are of benefit in treating
prevent secondary infection. Superficial minor burns, usually in the outpatient
facial wounds can also be treated exposed, setting.
with or without the application of topical Option B: In intramuscular drug
ointments. administration, the absorption of the drug is
Option D: Opioids may be given to this determined by the bulk of the muscle and its
patient. These burns need to be shaved to vascularity. The onset and duration of the
action of the drug is not adjustable. In case respiratory depression develops which may
of inadvertent scenarios such as result from cerebral hypoxia.
anaphylaxis, burns, or neurovascular Option C: Carbon monoxide levels of 11%
injuries, intravenous (IV) assess needs to to 20% result in flushing. The classic
be secured symptoms of cherry red nail beds and
Option C: Subcutaneous injections are mucous membranes are not “classic” and
another form of the parenteral route of are usually post-mortem findings. Patients
medication and are administered to the may also develop ataxia, apraxia,
layer of skin referred to as cutis, just below incontinence, and cortical blindness.
the dermis and epidermis layers. Option D: Cardiac effects, especially
Subcutaneous tissue has few blood vessels; ventricular arrhythmias occur. Ventricular
therefore, the medications injected undergo arrhythmias are implicated as the cause of
absorption at a slow, sustained rate. death most often in CO poisoning. There is
6. Question evidence that myocardial impairment begins
Nurse Cirie is caring for a client who at the relatively low level of COHgb of 20%.
suffered a smoke inhalation injury. The 7. Question
carbon monoxide report reveals a level of A client is brought to the emergency unit
35%. Based on the level, which of the with third-degree burns on the posterior
following signs should the nurse expect in trunk, right arm, and left posterior leg. Using
the client? the Rule of Nines, what is the total body
surface area (TBSA) that has been burned?
A. Seizure
B. Confusion A. 36%
C. Flushing B. 54%
D. Coma C. 45%
Incorrect D. 27%
Correct Answer: B. Confusion Correct
Correct Answer: A. 36%
Signs and symptoms of carbon monoxide
levels between 21-40% (moderate The Rule of Nines, also known as the
poisoning) include hypotension, Wallace Rule of Nines, is a tool used by
tachycardia, headache, drowsiness, trauma and emergency medicine providers
confusion, nausea, and vomiting. Mental to assess the total body surface area
status changes such as altered level of (TBSA) involved in burn patients. Based on
consciousness, disorientation, and memory the rule of nines, the posterior trunk equals
loss may occur. 18%, right arm equals 9%, and the left
posterior leg equals 9%. Therefore, a total
Option A: Carbon monoxide levels of 41% of 36%.
to 60% result in seizures. As
carboxyhemoglobin (COHgb) levels rise, the Option B: The Rule of Nines estimation of
cerebral blood vessels dilate, and both body surface area burned is based on
coronary blood flow and capillary density assigning percentages to different body
increase. If exposure continues, central areas. The entire head is estimated as 9%
(4.5% for anterior and posterior). The entire
trunk is estimated at 36% and can be further provide the patient with 100% oxygen at a
broken down into 18% for anterior higher LPM flow rate.
components and 18% for the back.
Option C: The anterior aspect of the trunk Options A and C: The nasal cannula is a
can further be divided into chest (9%) and thin tube, often affixed behind the ears and
abdomen (9%). The upper extremities total used to deliver oxygen directly to the
18% and thus 9% for each upper extremity. nostrils from a source connected with
Each upper extremity can further be divided tubing. This is the most common method of
into anterior (4.5%) and posterior (4.5%). delivery for home use and provides flow
Option D: The lower extremities are rates of 2 to 6 liters per minute (LPM)
estimated at 36%, 18% for each lower comfortably, allowing the delivery of oxygen
extremity. Again this can be further divided while maintaining the patient’s ability to
into 9% for the anterior and 9% for the utilize his or her mouth to talk, eat, etc.
posterior aspect. The groin is estimated at Option D: An air-entrainment (also known
1%. as venturi) mask can provide a pre-set
8. Question oxygen to the patient using jet mixing. As
A medicine student arrives at the the percent of inspired oxygen increases
emergency unit due to a burn injury that using such a mask, the air-to-oxygen ratio
occurred inside the laboratory and an decreases, causing the maximum
inhalation injury is suspected. Which of the concentration of oxygen provided by an air-
following is the appropriate oxygen therapy entrainment mask to be around 40%.
for the client? 9. Question
The nurse is handling a client who
A. Oxygen via nasal cannula at 5 L/min. sustained an electrical burn on the arm and
B. Oxygen via a tight-fitting, non-rebreather wrist and is scheduled for a fasciotomy.
face mask at 100% concentration. After the procedure, the nurse should
C. Oxygen via nasal cannula at 10 L/min. assess the affected extremity in which of the
D. Oxygen via Venturi mask at 30% Fi02. following, except?
Correct
Correct Answer: B. Oxygen via a tight- A. Sensation
fitting, non-rebreather face mask at 100% B. Color
concentration. C. Distal circulation
D. All of the above
If an inhalation injury is suspected, Incorrect
management includes the administration of Correct Answer: D. All of the above
oxygen via a tight-fitting, non-rebreather
face mask at 100% concentration. This is Following fasciotomy, the nurse should
prescribed until carboxyhemoglobin levels in assess pulses, color, sensation, and
the blood fall below 15%. Non-rebreathing movement of the affected extremity as well
masks have a bag attached to the mask as bleeding. A fasciotomy is an emergency
known as a reservoir bag, which inhalation procedure used to treat acute compartment
draws from to fill the mask through a one- syndrome. Acute compartment syndrome
way valve and features ports at each side often follows high energy trauma, fractures,
for exhalation, resulting in an ability to
circumferential burns, crush injuries, or even least 3-7days. This period of immobilization
a tight plaster cast. allows the autograft time to adhere to the
wound bed. Clinically, skin grafts are
Option A: Loss of sensation can signal secured into place and often bolstered until
compartment syndrome. Classical features postoperative day 5 to 7 to allow the skin
of compartment syndrome are those of graft to go through the above steps,
ischemia, pain out of proportion to the ensuring the best skin graft take.
injury, paraesthesia, pallor, paralysis, and
pain on passive movement, especially Option B: Do not place the affected leg in a
stretch of the concerned compartment. dependent position. Any buildup of fluid
Option B: Pallor is associated with between the split-thickness skin graft and
compartment syndrome. Fasciotomy wound wound bed will jeopardize skin graft take,
management begins with an inspection at including seroma, hematoma, and infection.
48 hours. If the compartments are soft, this Shear or traction injury also disrupts skin
closure is achievable by primary wound graft healing.
closure, secondary wound healing, or as Option C: Dangling of legs puts the affected
needed in approximately 50% of wounds site into a dependent position, which can
split-thickness skin grafting. cause a build-up of fluid that jeopardizes the
Option C: Distal circulation should be skin graft. The graft can have incomplete
checked to prevent ischemia. Two-point (less than 100%) take or complete nontake.
discrimination can be useful for determining Option D: Split-thickness skin grafts typically
nerve ischemia. These signs and symptoms become adherent to the recipient wound
can be challenging to assess depending on bed 5 to 7 days following skin graft
the conscious level, sensory state, and placement. The dressings placed
ability to communicate. intraoperatively are kept in place until 5 to 7
10. Question days postoperatively to minimize shear and
The nurse is caring for a client with a burn traction to the healing skin graft.
wound on the left knee and an autograft and 11. Question
skin grafting was performed. Which of the Nurse Troyzan has just received the
following activities will be prescribed for the change-of-shift report in the burn unit.
client post-op? Which of the following client requires the
most immediate care?
A. Elevation and immobilization of the
affected leg. A. A 50-year-old who was admitted with
B. Placing the affected leg in a dependent electrical burns 24 hours ago and has a
position. serum potassium level of 5 mEq/L.
C. Dangling of legs. B. A 40-year-old with partial-thickness leg
D. Bathroom privileges. burns which has a temperature of 101.9°F
Correct and blood pressure of 89/42 mm Hg.
Correct Answer: A. Elevation and C. A 30-year-old who returned from
immobilization of the affected leg. debridement surgery 3 hours ago and is
complaining of pain (Pain scale of 7/10).
Autograft placed on the lower extremity D. A 25-year-old admitted 4 days
requires elevation and immobilization for at previously with facial burns due to a house
fire that has been crying since recent 12. Question
visitors left. The nurse is caring for a client who
Incorrect sustained superficial partial-thickness burns
Correct Answer: B. A 40-year-old with on the anterior lower legs and anterior
partial-thickness leg burns which has a thorax. Which of the following does the
temperature of 101.9°F and blood pressure nurse expect to note during the
of 89/48 mm Hg. resuscitation/emergent phase of the burn
injury?
The client’s vital signs indicate that life-
threatening complications of sepsis may be A. Increased blood pressure
developing. Burn wound infections are one B. Increased hematocrit levels
of the most important and potentially serious C. Decreased heart rate
complications that occur in the acute period D. Increased urine output
following injury. If the patient’s host Correct
defenses and therapeutic measures Correct Answer: B. Increased hematocrit
(including excision of necrotic tissue and levels
wound closure) are inadequate or delayed,
microbial invasion of viable tissue occurs, The resuscitation/emergent phase begins at
which is the hallmark of an invasive burn the time of injury and ends with the
wound infection. restoration of capillary permeability, usually
at 48-72 hours following the injury. During
Option A: Classically, hyperkalemia has this phase, there is an elevation of the
been regarded as a complication in patients hematocrit levels due to hemoconcentration
with electrical burns. The etiology of from the large fluid shifts
hyperkalemia includes metabolic acidosis,
destruction of red blood cells, Option A: Blood pressure is decreased due
rhabdomyolysis, and the development of to the shifting of fluids. The acute phase of
renal failure. burns is defined as a period extending from
Option C: Oral NSAIDs and acetaminophen the onset of burns with shock to the time
are of benefit in treating minor burns, taken for wound epithelialization which
usually in the outpatient setting. For normally takes about 12 to 14 days if the
hospitalized burn patients, opioids are the management of burns is adequate. The first
cornerstone of pharmacologic pain control. 48 is the period of shock.
Patient-controlled analgesia (PCA) with IV Option C: Pulse rate is higher than normal.
opioids is a safe and efficient method of If resuscitation is carried out urgently the
achieving flexible analgesia in burn-injured circulatory shock is not only prevented,
patients. none of the complications of shock are
Option D: Symptoms of depression and allowed to manifest.
anxiety are common and start to appear in Option D: Initially, blood is shunted away
the acute phase of recovery. Acute stress from the kidneys, resulting in low urine
disorder (occurs in the first month) and post- output. The greatest amount of fluid loss in
traumatic stress disorder (occurs after one burn patients is in the first 24 h after injury.
month) are more common after burns than For the first 8-12 hours, there is a general
other forms of injury. shift of fluids from intravascular to interstitial
fluid compartments. This means that any direct contact with the client. Remove
fluid given during this time will rapidly leak gloves after contact with a patient and/or the
out from the intravascular compartment. surrounding environment (including medical
13. Question equipment) using proper technique to
The nurse manager is observing a new prevent hand contamination.
nursing graduate caring for a burned client Option C: Sterile sheets and linen are used
in protective isolation. The nurse manager due to the high risk of infection. Soiled
intervenes if the new nursing graduate textiles, including bedding, towels, and
planned to implement which incorrect patient or resident clothing may be
component of protective isolation contaminated with pathogenic
technique? microorganisms. However, the risk of
disease transmission is negligible if they are
A. Performing strict handwashing handled, transported, and laundered in a
techniques. safe manner.
B. Wearing protective garb, including a 14. Question
mask, gloves, cap, shoe covers, gowns, and A client is undergoing fluid replacement
a plastic apron. after being burned 20% of her body 12
C. Using sterile bed sheets and linens. hours ago. The nursing assessment reveals
D. Wearing gloves and a gown only when a blood pressure of 90/50 mm Hg, a pulse
giving direct care to the client. rate of 110 beats per minute, and a urine
Incorrect output of 25 ml over the past hour. The
Correct Answer: D. Wearing gloves and a nurse reports the findings to the physician
gown only when giving direct care to the and anticipates which of the following
client. orders?

Thorough hand washing is performed A. Increasing the amount of intravenous


before and after each contact with the burn- (IV) lactated Ringer's solution administered
injured client. During the delivery of per hour.
healthcare, avoid unnecessary touching of B. Transfusing 1 unit of packed red blood
surfaces in close proximity to the patient to cells.
prevent both contaminations of clean hands C. Administering diuretic to increase urine
from environmental surfaces and output.
transmission of pathogens from D. Changing the IV lactated Ringer's
contaminated hands to surfaces. solution into dextrose in water.
Correct
Option A: The physical action of washing Correct Answer: A. Increasing the amount
and rinsing hands under such of intravenous (IV) lactated Ringer’s solution
circumstances is recommended because administered per hour.
alcohols, chlorhexidine, iodophors, and
other antiseptic agents have poor activity The client’s urine output indicates
against spores. inadequate fluid resuscitation. Hence the
Option B: Protective garbs such as masks, physician would order an increased amount
gloves, caps, shoe covers, gowns, and a of lactated Ringer’s solution administered
plastic apron need to be worn when having hourly. Patients with burns of more than
20% – 25% of their body surface should be B. A client who has just been admitted with
managed with aggressive IV fluid burns over 30% of the body after a
resuscitation to prevent “burn shock.” Urine warehouse fire.
output of 0.5 mL/kg or about 30 – 50 mL/hr C. A client with full-thickness burns on both
in adults and 0.5-1.0 mL/kg/hr in children arms who needs assistance in positioning
less than 30kg is a good target for adequate hand splints.
fluid resuscitation. D. A client who requires discharge teaching
about nutrition and wound care after having
Option B: Blood transfusion is not used for skin grafts.
fluid resuscitation therapy unless there is an Incorrect
indication of a low hemoglobin level. This Correct Answer: A. A client with infected
response, along with decreased cardiac partial-thickness back and chest burns who
output and increased vascular resistance, has a dressing scheduled.
can lead to marked hypovolemia and
hypoperfusion called “burn shock.” This can Familiarity with the dressing change and
be managed with aggressive fluid practice of sterility by a nurse from the
resuscitation and close monitoring for surgery unit will be appropriately used
adequate, but not excessive, IV fluids. during the float in the burn unit. There are
Option C: Diuretic works by removing several options for burn dressings. Some
circulating volume, thereby further are impregnated with antimicrobials (eg,
compromising the inadequate tissue silver). Most are a form of gauze, but there
perfusion. The patient’s vital signs, mental are biosynthetic dressings with some of the
status, capillary refill, and urine output must characteristics of skin that adhere to the
be monitored and fluid rates adjusted wound and can be left in place for extended
accordingly. Again, adequate fluid periods of time.
resuscitation is the goal.
Option D: Dextrose in water will only Option B: Admission assessment requires
maintain fluid balance since it is an isotonic expertise in caring for burn patients. The
solution, therefore will not be helpful in this needs of the client must be competently met
situation. Four mL lactated ringers solution with the knowledge, skills and abilities of the
× percentage total body surface area staff to meet these needs. In other words,
(%TBSA) burned × patient’s weight in the nurse who delegates aspects of care to
kilograms = total amount of fluid given in the other members of the nursing team must
first 24 hours. balance the needs of the client with the
15. Question abilities of those to which the nurse is
Nurse Kelsey is a nurse manager assigned delegating tasks and aspects of care.
to the burn unit. Which client is best to Option C: Splinting requires expertise in
assign to an RN who has floated from the caring for burn patients. The staff members’
surgery unit? levels of education, knowledge, past
experiences, skills, abilities, and
A. A client with infected partial-thickness competencies are also evaluated and
back and chest burns who has a dressing matched with the needs of all of the patients
scheduled. in the group of patients that will be cared
for.
Option D: Discharge teaching requires Option D: Full-thickness: dry, white, or
expertise in caring for burn patients. black, no blisters, absent capillary refill, and
Validated and documented competencies absent sensation. Requires surgical repair
must also be considered prior to assignment and grafting. Full-thickness burns (third
of patient care. No aspect of care can be degree) extend through both the epidermis
assigned or delegated to another nursing and dermis and into the subcutaneous fat or
staff member unless this staff member has deeper. These burns have little or no pain,
documented evidence that they are deemed can be white, brown, or charred, and feel
competent by a registered nurse to do so. firm and leathery to palpation with no
16. Question blanching.
A client sustained burns on the back. These 17. Question
areas appear dry, blotchy cherry red, Which of the following refers to a wound
blistering, doesn’t blanch, no capillary refill, covering brought about by the donated
and reduced or absent sensation. This type human cadaver skin provided by the skin
of burn depth is classified as? bank?

A. Superficial partial-thickness burn A. Autograft.


B. Superficial dermal B. Homograft
C. Deep partial-thickness burn C. Heterograft
D. Full-thickness burn D. Xenograft
Incorrect Incorrect
Correct Answer: C. Deep partial-thickness Correct Answer: B. Homograft
burn
Homograft is a tissue graft from a donor of
Deep partial-thickness burn: blistering, dry, the same species as the recipient. Skin from
blotchy cherry red, doesn’t blanch, no organ donors can be used as a temporary
capillary refill, and reduced or absent covering. It is temporary because this skin
sensation. Generally, heals in 3-6 weeks, will eventually be rejected. This is known as
but scar formation results and skin grafting a homograft. Homografts may be required
may be required. initially if the injured area is too large to be
covered by the patient’s own skin.
Option A: Superficial partial-thickness: red,
glistening, pain, absence of blisters, and Option A: A skin graft, also known as an
brisk capillary refill. Not life-threatening and autograft, involves taking skin from an
normally heals within a week, without unburned part of the patient’s body and
scarring. Superficial burns (first degree) placing it on the wound after the burn has
involve only the epidermis and are warm, been removed.
painful, red, soft, and blanch when touched. Option C: Pigskin grafts are termed
Usually, there is no blistering. A typical xenografts, or heterografts because they
example is a sunburn. are transplanted from an organism of one
Option B: Pale pink or mottled appearance species to that of a different species. Both
with associated swelling and small blisters. allografts and xenografts are biologic
A wet, shiny, and weeping surface is also a dressings only, are ultimately rejected by
characteristic. Brisk capillary refill. the patient’s immune system, and need to
be removed prior to definitive wound Option D: At least 3 weeks after surgery,
treatment or skin grafting. avoid exercise that stretches the skin graft,
Option D: Xenograft is a graft of tissue unless the doctor gives other instructions. If
taken from a donor of one species and the graft was placed on the legs, arms,
grafted into a recipient of another species. hands, or feet, the patient may need
While xenografts are rejected before physiotherapy to prevent scar tissue from
undergoing revascularization, allografts limiting movement.
initially undergo revascularization but are 19. Question
typically rejected after approximately 10 A client is prescribed by the physician to
days because of the strong antigenicity of undergo an escharotomy. Which of the
the skin. following statements made by the nurse is
18. Question true regarding this procedure?
A client is being discharged today after
undergoing autografting. What would the A. "It is the surgical removal of a thin layer
nurse include in the discharge instructions? of the client's own unburned skin."
B. "A lengthwise incision is made through
A. Refrain from using splints. the burn eschar to relieve vasodilation."
B. Avoid smoking. C. "It is performed at the bedside and
C. Exposed the site to sunlight. without anesthesia."
D. Encourage weight-bearing exercise. D. "It is the application of topical enzyme
Correct agents directly to the wound, and these
Correct Answer: B. Avoid smoking. agents digest necrotic collagen tissue."
Correct
Smoking can decrease the blood supply to Correct Answer: C. “It is performed at the
the newly graft recipient bed interface, and bedside and without anesthesia”.
the chance of graft failure increases. The
combined effect of nicotine and carbon An escharotomy is performed at the bedside
monoxide is deadly to the healing process. and without anesthesia since nerve endings
This can result in partial or complete loss of have been destroyed by the burn injury. An
healing of the wound, skin graft, flap, or any escharotomy is an emergency surgical
combination of these. This can compromise procedure involving incising through areas
the cosmetic results of the surgery. of burnt skin to release the eschar and its
constrictive effects, restore distal circulation,
Option A: Static or primary splints are used and allow adequate ventilation.
in the acute phase for skin graft protection
after surgery or anti contracture positioning. Option A: A skin graft, also known as an
These splints are applied to adjacent intact autograft, involves taking skin from an
skin. unburned part of the patient’s body and
Option C: Healed burns or skin grafts may placing it on the wound after the burn has
be extremely sensitive to sunlight and may been removed.
sunburn more severely even after short Option B: Escharotomy involves making a
periods of time in the sun compared to lengthwise incision through the burn eschar
before the injury. Sun sensitivity after a burn to relieve vasoconstriction. The incisions
injury may last for a year or more. should extend from unburnt skin to unburnt
skin ideally, or at least into areas of more intravascular fluid loss, and large fluid shifts.
superficial burns, down to subcutaneous fat, These responses occur mostly over the first
and release any constrictions. 24 hours peaking at around six to eight
Option D: This is a selective method for hours after injury. This can be managed
debridement of necrotic tissue using an with aggressive fluid resuscitation and close
exogenous proteolytic enzyme, monitoring for adequate, but not excessive,
collagenase, to debride Clostridium IV fluids.
bacteria. Collagenase digests the collagen Option C: Continuous monitoring and
in the necrotic tissue allowing it to detach. reassessment of nutritional status with
20. Question modifications in nutritional therapy as
Rehabilitation is the final phase of burn indicated can accommodate the unique yet
care. Which of the following are the goals diverse needs of this population and support
during this phase? Select all that apply. their therapeutic goals for recovery.
Option D: Belong to the main goal during
A. Provide emotional support. the resuscitative phase. Patients with burns
B. Prevent hypovolemic shock. of more than 20% – 25% of their body
C. Promote wound healing and proper surface should be managed with aggressive
nutrition. IV fluid resuscitation to prevent “burn
D. Fluid replacement. shock.”
E. Help the client in gaining optimal Option E: A comprehensive rehabilitation
physical functioning. program is essential to decrease a patient’s
Correct post-traumatic effects and improve
Correct Answer: A, C, and E. functional independence. While different
professionals possess expertise in their own
The rehabilitation phase starts after wound specialties, there are some simple and
closure and ends upon discharge and effective methods that can be utilized to
beyond. The goals of this phase include help the patient reach their maximum
minimizing functional loss, promoting functional outcome.
psychosocial support, promoting wound
healing, and proper nutrition. 1. Question
The newly admitted client has burns on both
Option A: Patients may try to refuse legs. The burned areas appear white and
treatment as they are in pain and may not leather-like. No blisters or bleeding are
fully understand the impact of not present, and the client states that he or she
participating in their rehabilitation; they, has little pain. How should this injury be
therefore, need the support and categorized?
encouragement of the burn care
professionals to help them through this A. Superficial
difficult experience with the knowledge of B. Partial-thickness superficial
how different their quality of life can be. C. Partial-thickness deep
Option B: Inflammatory and vasoactive D. Full thickness
mediators such as histamines, Correct
prostaglandins, and cytokines are released Correct Answer: D. Full thickness
causing a systemic capillary leak,
The characteristics of the wound meet the Correct Answer: B. Partial-thickness
criteria for a full-thickness injury (color that superficial
is black, brown, yellow, white, or red; no
blisters; pain minimal; outer layer firm and The characteristics of the wound meet the
inelastic). With pressure, no blanching criteria for a superficial partial-thickness
occurs. The burn is leathery and dry. There injury (color that is pink or red; blisters; pain
is minimal to no pain because of decreased present and high). Superficial partial-
sensation. Full-thickness burns heal by thickness (second-degree) involves the
contracture and take greater than 8 weeks. superficial dermis. It appears red with
Full-thickness burns require skin grafting. blisters and is wet. The erythema blanches
with pressure. The pain associated with
Option A: Superficial (first-degree) involves superficial partial-thickness is severe.
the epidermis of the skin only. It appears Healing typically occurs within 3 weeks with
pink to red, there are no blisters, and it is minimal scarring.
dry. It is moderately painful. Superficial
burns heal without scarring within 5 to 10 Option A: Superficial (first-degree) involves
days. the epidermis of the skin only. It appears
Option B: Superficial partial-thickness pink to red, there are no blisters, and it is
(second-degree) involves the superficial dry. It is moderately painful. Superficial
dermis. It appears red with blisters and is burns heal without scarring within 5 to 10
wet. The erythema blanches with pressure. days.
The pain associated with superficial partial- Option C: Deep partial-thickness (second-
thickness is severe. Healing typically occurs degree) involves the deeper dermis. It
within 3 weeks with minimal scarring. appears yellow or white, is dry, and does
Option C: Deep partial-thickness (second- not blanch with pressure. There is minimal
degree) involves the deeper dermis. It pain due to a decreased sensation. Healing
appears yellow or white, is dry, and does occurs in 3 to 8 weeks with scarring
not blanch with pressure. There is minimal present.
pain due to a decreased sensation. Healing Option D: Third-degree involves the full
occurs in 3 to 8 weeks with scarring thickness of skin and subcutaneous
present. structures. It appears white or black/brown.
2. Question With pressure, no blanching occurs. The
The newly admitted client has a large burn is leathery and dry. There is minimal to
burned area on the right arm. The burned no pain because of decreased sensation.
area appears red, has blisters, and is very Full-thickness burns heal by contracture and
painful. How should this injury be take greater than 8 weeks. Full-thickness
categorized? burns require skin grafting.
3. Question
A. Superficial The burned client newly arrived from an
B. Partial-thickness superficial accident scene is prescribed to receive 4
C. Partial-thickness deep mg of morphine sulfate by IV push. What is
D. Full thickness the most important reason to administer the
Incorrect opioid analgesic to this client by the
intravenous route?
is constipation. This effect occurs via
A. The medication will be effective more stimulation of mu-opioid receptors on the
quickly than if given intramuscularly. myenteric plexus, which in turn inhibits
B. It is less likely to interfere with the gastric emptying and reduces peristalsis.
client’s breathing and oxygenation. 4. Question
C. The danger of an overdose during fluid Which vitamin deficiency is most likely to be
remobilization is reduced a long-term consequence of a full-thickness
D. The client delayed gastric emptying. burn injury?
Correct
Correct Answer: C. The danger of an A. Vitamin A
overdose during fluid remobilization is B. Vitamin B
reduced. C. Vitamin C
D. Vitamin D
The most important reason is to prevent an Correct
overdose from accumulation of drug in the Correct Answer: D. Vitamin D
interstitial space during the fluid shift of the
emergent phase. When edema is present, Skin exposed to sunlight activates vitamin
cumulative doses are rapidly absorbed D. Partial-thickness burns reduce the
when the fluid shift is resolving. This activation of vitamin D. Activation of vitamin
delayed absorption can result in lethal blood D is lost completely in full-thickness burns.
levels of analgesics. The loss of healthy skin following a burn
injury can decrease epidermal vitamin D
Option A: Providing some pain relief has a production. Additionally, low vitamin D
high priority and giving the drug by the IV levels have been reported to have
route instead of IM, SC, or orally does continued for 7 years post-burn in pediatric
increase the rate of effect. Pain that is more outpatients.
severe and not well controlled may be
manageable with single or continuous Option A: Vitamin A deficiency (VAD) is a
doses of IV, epidural, and intrathecal highly prevalent health concern associated
formulations. Infusion dosing can vary with substantial morbidity and mortality,
significantly between patients and largely mostly affecting young children in
depends on how naive or tolerant they are impoverished regions throughout the world.
to opiates. Insufficient intake of absorption leads to
Option B: Respiratory depression is among deficiency and compromise of essential
the more serious adverse reactions with physiologic processes.
opiate use that is especially important to Option B: Vitamin B12 deficiency can lead
monitor in the postoperative patient to hematologic and neurological symptoms.
population. Extreme caution is necessary Vitamin B12 is stored in excess in the liver,
with severe respiratory depression and decreasing the likelihood of deficiency.
asthma exacerbation cases since morphine However, in cases in which vitamin B12
can further decrease the respiratory drive. cannot be absorbed, for example, due to
Option D: Delayed gastric emptying is not a dietary insufficiency, malabsorption, or lack
side effect of morphine. Among the more of intrinsic factor, hepatic stores are
common unwanted effects of morphine use depleted, and deficiency ensues.
Option C: Vitamin C deficiency, also known slower healing, and/or to other
as scurvy, is a disease primarily associated complications.
with socioeconomic status and access to Option B: Accordingly, patients with burn
food. Signs and symptoms are often readily injury cannot be considered recovered when
visible in individuals who develop this the wounds have healed; instead, burn
disease. The classic constellation of injury leads to long-term profound
corkscrew hairs, perifollicular hemorrhage, alterations that must be addressed to
and gingival bleeding is highly suggestive of optimize quality of life.
vitamin C deficiency. Option D: Burns to the genitals correspond
5. Question to approximately 2% of all burn patients in
Which client factors should alert the nurse North American case series. The majority of
to potential increased complications with a those cases are associated with greater
burn injury? burned body surface areas, in which direct
fire and scalding are the most frequent
A. The client is a 26-year-old male. causes. Burn management begins with
B. The client has had a burn injury in the opportune diagnosis and entails making the
past. correct classification, depending on the
C. The burned areas include the hands and depth of the lesion.
perineum. 6. Question
D. The burn took place in an open field and The burned client is ordered to receive
ignited the client’s clothing. intravenous cimetidine, an H2 histamine
Correct blocking agent, during the emergent phase.
Correct Answer: C. The burned areas When the client’s family asks why this drug
include the hands and perineum. is being given, what is the nurse’s best
response?
Burns of the perineum increase the risk for
sepsis. Burns of the hands require special A. “To increase urine output and prevent
attention to ensure the best functional kidney damage.”
outcome. Complications are related to the B. “To stimulate intestinal movement and
extension of the burn. Burns to the genitalia prevent abdominal bloating.”
and perineum are severe conditions that all C. “To decrease hydrochloric acid
urologists should be familiar with and know production in the stomach and prevent
how to manage. Fluid resuscitation is the ulcers.”
initial step in treating these patients and is D. “To inhibit loss of fluid from the
followed by topical dressings in the case of circulatory system and prevent hypovolemic
superficial burns. shock.”
Correct
Option A: Irrespective of the type of burn Correct Answer: C. “To decrease
injury, the aged population shows slower hydrochloric acid production in the stomach
recoveries and suffers more complications. and prevent ulcers.”
Age-associated immune dysfunction,
immunosenescence, may predispose the Ulcerative gastrointestinal disease may
elderly burn patients to more infections, develop within 24 hours after a severe burn
as a result of increased hydrochloric acid
production and decreased mucosal barrier. Hypokalemia is most likely to occur during
Cimetidine inhibits the production and the fluid remobilization period as a result of
release of hydrochloric acid. dilution, potassium movement back into the
cells, and increased potassium excreted
Option A: Adequate fluid therapy is crucial into the urine with the greatly increased
in maintaining renal function. Monitoring by urine output. In an attempt to prevent
urine output or Swan-Ganz catheterization hypokalemia it is advised to add ’20-30
and thermodilution cardiac output mEq/1 of potassium to the hypotonic fluids
determination is useful in the circulatory in order to compensate for urinary losses
management of severely burned patients. and intracellular shift; it is also mandatory to
Albumin infusion increases plasma volume correct precipitating factors such as
by 37% and normalizes elevated basal increased pH, hypomagnesemia, and
levels of aldosterone and plasma renin several drugs.
activity.
Option B: Other management for severe Option A: In major burns. intravascular
burns includes nasogastric tube placement volume is lost in burned and unburned
as most patients will develop ileus. Foley tissues: this process is due to an increase in
catheters should be placed to monitor urine vascular permeability, increased interstitial
output. Cardiac and pulse oximetry osmotic pressure in burn tissue. and cellular
monitoring are indicated. Pain control is edema. with the most significant shifts
best managed with IV medication. occurring in the first hours. Hyponatremia is
Option D: Patients with burns of more than frequent, and the restoration of sodium
20% – 25% of their body surface should be losses in the burn tissue is, therefore,
managed with aggressive IV fluid essential hyperkalemia is also characteristic
resuscitation to prevent “burn shock.” A of this period because of the massive tissue
variety of formulas exist, like Brooke, necrosis.
Galveston, Rule of Ten, etc.4, but the most Option B: The early post-resuscitation
common formula is the Parkland Formula. phase is a period of transition from the
This formula estimates the amount of fluid shock phase to the hypermetabolic phase,
given in the first 24 hours, starting from the and fluid strategies should change radically
time of the burn. with a view to restoring losses due to water
7. Question evaporation. The main changes in this
At what point after a burn injury should the period are hypernatremia, hypocalcemia,
nurse be most alert for the complication of hypokalemia, hypomagnesemia, and
hypokalemia? hypophosphatemia.
Option D: The acute phase of burns is
A. Immediately following the injury defined as a period extending from the
B. During the fluid shift onset of burns with shock to the time taken
C. During fluid remobilization for wound epithelialization which normally
D. During the late acute phase takes about 12 to 14 days if management of
Correct burns is adequate.
Correct Answer: C. During fluid 8. Question
remobilization What clinical manifestation should alert the
nurse to possible carbon monoxide
poisoning in a client who experienced a intranasal hair as well as carbonaceous
burn injury during a house fire? material or soot in the mouth or sputum.
9. Question
A. Pulse oximetry reading of 80% What clinical manifestation indicates that an
B. Expiratory stridor and nasal flaring escharotomy is needed on a circumferential
C. Cherry red color to the mucous extremity burn?
membranes
D. Presence of carbonaceous particles in A. The burn is full thickness rather than
the sputum partial thickness.
Correct B. The client is unable to fully pronate and
Correct Answer: C. Cherry red color to the supinate the extremity.
mucous membranes C. Capillary refill is slow in the digits and
the distal pulse is absent.
The saturation of hemoglobin molecules D. The client cannot distinguish the
with carbon monoxide and the subsequent sensation of sharp versus dull in the
vasodilation induces a “cherry red” color of extremity.
the mucous membranes in these clients. Correct
Cherry-red skin color associated with Correct Answer: C. Capillary refill is slow in
severe carbon monoxide poisoning is seen the digits and the distal pulse is absent.
in only 2-3% of symptomatic cases. Skin
may develop erythematous lesions and Circumferential eschar can act as a
bulla, especially over bony prominences. tourniquet when edema forms from the fluid
shift, increasing tissue pressure, and
Option A: Carbon monoxide quickly binds preventing blood flow to the distal
with hemoglobin with an affinity greater than extremities, and increasing the risk for
that of oxygen to form COHb. The resulting tissue necrosis. This problem is an
decrease in arterial oxygen content and shift emergency and, without intervention, can
of the oxyhemoglobin dissociation curve to lead to loss of the distal limb. This problem
the left explains the acute hypoxic can be reduced or corrected with an
symptoms (primarily neurologic and cardiac) escharotomy.
seen in patients with acute poisoning.
Option B: Patients suffering from smoke Option A: The American Burn Association
inhalation may have symptoms of burning recommends burn center referrals for
sensation in the nose or throat (which is patients with full-thickness burns. Patients
often caused by an irritant chemical toxin), a being transferred to burn centers do not
cough with increased sputum production, need extensive debridement or topical
stridor, and dyspnea with rhonchi or antibiotics before transfer.
wheezing. Option B: Once established, burn
Option D: The other manifestations are contractures can be treated with serial
associated with inhalation injury, but not splinting, release of contracting bands with
specifically carbon monoxide poisoning. Z-plasties, incision, and skin grafting or
Physical examination should include looking excision, and resurfacing with skin grafts or
for facial burns, such as loss of facial and flaps, local rotation flaps, use of tissue
expanders, or with free flap reconstruction.
Option D: After a deep burn injury, wounds that produces exudation with an
cutaneous nerve regeneration will occur important protein loss through the burn
with the migration of new nerve fibers from wound and acute phase response of plasma
the wound bed or from the collateral protein synthesis in liver that occurs with
sprouting of nerve fibers from adjacent even a very small percentage of burn skin
uninjured areas. This nerve regeneration (0.8%) and that produces a decrease to
process is imperfect. It was reported that about 80% of normal albumin and
71% of extensively burned victims suffer prealbumin levels.
from abnormal sensations and 36% from Option B: Immunochemical studies of the
chronic pain. Recent studies on rats have sera of patients with severe burns led to the
shown that vagus nerve stimulation conclusion that as soon as within the first
improved thermal injury-induced shock two days following the trauma, tissue
symptoms. antigens sharing common components with
10. Question those of the burned and normal skin were
What additional laboratory test should be detected in the blood. The antigens in
performed on any African American client question were not detected in the sera of
who sustains a serious burn injury? healthy subjects and were not identical with
the C-reactive protein. Long-term circulation
A. Total protein of these antigens, i.e. for 2 to 3 months after
B. Tissue type antigens burning, was revealed.
C. Prostate-specific antigen Option C: For the detection of prostate
D. Hemoglobin S electrophoresis cancer, an elevated serum prostate-specific
Correct antigen is the most common laboratory
Correct Answer: D. Hemoglobin S abnormality, as the majority of men with
electrophoresis early prostate cancer have no symptoms.
However, prostate-specific antigen,
Sickle cell disease and sickle cell trait are otherwise known as PSA, is clinically
more common among African Americans. imprecise as benign and malignant
Although clients with sickle cell disease processes both can elevate the serum
usually know their status, the client with marker.
sickle cell trait may not. The fluid, 11. Question
circulatory, and respiratory alterations that Which type of fluid should the nurse expect
occur in the emergent phase of a burn injury to prepare and administer as fluid
could result in decreased tissue perfusion resuscitation during the emergent phase of
that is sufficient to cause sickling of cells, burn recovery?
even in a person who only has the trait.
Determining the client’s sickle cell status by A. Colloids
checking the percentage of hemoglobin S is B. Crystalloids
essential for any African American client C. Fresh-frozen plasma
who has a burn injury. D. Packed red blood cells
Correct
Option A: Burn patients can also have Correct Answer: B. Crystalloids
important reduction in albumin level due to a
higher vascular permeability in the burn
Although not universally true, most fluid B. Loosen the dressing.
resuscitation for burn injuries starts with C. Notify the emergency team.
crystalloid solutions, such as normal saline D. Document the observation as the only
and Ringer’s lactate. Burn patients receive a action.
larger amount of fluids in the first hours than Correct
any other trauma patients. Initial Correct Answer: B. Loosen the dressing.
resuscitation is based on crystalloids
because of the increased capillary Respiratory difficulty can arise from external
permeability occurring during the first 24 h. pressure. The first action in this situation
After that time, some colloids, but not all, would be to loosen the dressing and then
are accepted. reassess the client’s respiratory status.
Generally, it is recommended that pressure
Option A: Colloids are not generally used should be maintained between 20 and 30
during the fluid shift phase because these mm Hg, which is above capillary pressure
large particles pass through the leaky but less than what would diminish peripheral
capillaries into the interstitial fluid, where blood circulation.
they increase the osmotic pressure.
Increased osmotic pressure in the interstitial Option A: It is unnecessary to administer
fluid can worsen the capillary leak syndrome oxygen. Wearing pressure garments is
and make maintaining the circulating fluid uncomfortable and challenging; problems
volume even more difficult. with movement, appearance, fit, comfort,
Option C: Fresh frozen plasma appears to swelling of extremities, rashes, and
be a useful and effective immediate burn blistering are common; consequently, low
resuscitation fluid but its benefits must be compliance with PGT is to be expected.
weighed against its costs, and risks of viral Option C: The nurse may intervene first.
transmission and acute lung injury. However, monitoring of pressure exerted by
Option D: The burn client rarely requires pressure garments is currently difficult and
blood during the emergent phase unless the time-consuming, and not routinely done and
burn is complicated by another injury that currently, the optimal pressure magnitude
involves hemorrhage. Ongoing blood loss, for PGT remains unsolved.
anemia, hypoxia, and cardiac disease are Option D: The nurse may loosen the
the most common reasons for blood dressing to help the client breathe. Recent
transfusion in burn patients. Other important evidence suggests that pressure garment
causes include age, percentage of burn therapy is effective for the prevention and/or
(TBSA), need for further operation, treatment of abnormal scarring after burn
presence of acute respiratory distress injury but that the clinical benefit is restricted
syndrome, sepsis, and evidence of cardiac to those patients with moderate or severe
ischemia. scarring.
12. Question 13. Question
The client with a dressing covering the neck The client who experienced an inhalation
is experiencing some respiratory difficulty. injury 6 hours ago has been wheezing.
What is the nurse’s best first action? When the client is assessed, wheezes are
no longer heard. What is the nurse’s best
A. Administer oxygen. action?
B. Document the finding as the only action.
A. Raise the head of the bed. C. Ask the client if anyone in her family has
B. Notify the emergency team. diabetes mellitus.
C. Loosen the dressings on the chest. D. Slow the intravenous infusion of
D. Document the findings as the only dextrose 5% in Ringer’s lactate.
action. Correct
Correct Correct Answer: B. Document the finding as
Correct Answer: B. Notify the emergency the only action.
team.
Neural and hormonal compensation to the
Clients with severe inhalation injuries may stress of the burn injury in the emergent
sustain such progressive obstruction that phase increases liver glucose production
they may lose the effective movement of air. and release. An acute rise in the blood
When this occurs, wheezing is no longer glucose level is an expected client response
heard and neither are breath sounds. The and is helpful in the generation of energy
client requires the establishment of an needed for the increased metabolism that
emergency airway and the swelling usually accompanies this trauma.
precludes intubation.
Option A: The glucose level is not high
Option A: Raising the head of the bed would enough to alert the emergency team. A
be not much help because of the obstructed variety of laboratory tests will be needed
airway. Airway protection should include within the first 24 hours of a patient’s
considering early and preemptive intubation admission (some during the initial
for patients with inhalation injury. resuscitative period and others after the
Option C: Dressings may be loosened, but patient is stabilized).
emergency intubation would still be needed. Option C: A family history of diabetes could
Airway edema may occur suddenly as make her more of a risk for the disease, but
edema worsens, and often, the upper this is not a priority at this time. The
airways develop injury and obstruction secondary assessment shouldn’t begin until
earliest, prior to the parenchymal injury. the primary assessment is complete;
Option D: This is not a normal finding. There resuscitative efforts are underway; and
may be accessory muscle usage, lines, tubes, and catheters are placed.
tachypnea, cyanosis, stridor, and Option D: Infusion of an IV fluid containing
rhonchi/rales/wheezing. Findings of stridor dextrose may further increase the client’s
or upper airway turbulence/noise are often a blood glucose. The ideal burn resuscitation
sign of impending airway compromise, and is the one that effectively restores plasma
prompt intubation should be strongly volume, with no adverse effects. Isotonic
considered. crystalloids, hypertonic solutions, and
14. Question colloids have been used for this purpose,
Ten hours after the client with 50% burns is but every solution has its advantages and
admitted, her blood glucose level is 90 disadvantages. None of them is ideal, and
mg/dL. What is the nurse’s best action? none is superior to any of the others.
15. Question
A. Notify the emergency team.
On admission to the emergency department tachycardic and tachypneic. They may
the burned client’s blood pressure is 90/60, exhibit hypotension. Mental status changes
with an apical pulse rate of 122. These such as confusion, altered level of
findings are an expected result of what consciousness, disorientation, and memory
thermal injury-related response? loss may occur.
16. Question
A. Fluid shift Twelve hours after the client was initially
B. Intense pain burned, bowel sounds are absent in all four
C. Hemorrhage abdominal quadrants. What is the nurse’s
D. Carbon monoxide poisoning best action?
Correct
Correct Answer: A. Fluid shift A. Reposition the client onto the right side.
B. Document the finding as the only action.
The physiologic effect of histamine release C. Notify the emergency team.
in injured tissues is a loss of vascular D. Increase the IV flow rate.
volume to the interstitial space, with a Correct
resulting decrease in blood pressure. After a Correct Answer: B. Document the finding as
burn, fluid shifts from vascular to interstitial the only action.
and intracellular spaces because of
increased capillary pressure, increased Decreased or absent peristalsis is an
capillary and venular permeability, expected response during the emergent
decreased interstitial hydrostatic pressure, phase of burn injury as a result of neural
chemical inflammatory mediators, and and hormonal compensation to the stress of
increased interstitial protein retention. injury. After the mid and late 1990s, the idea
of staged food intake was advanced: a
Option B: Intense pain and carbon small amount of light fluid is started several
monoxide poisoning increase blood hours after burn so as to not only
pressure. Superficial dermal burns are supplement nutrition but stimulate GI
initially the most painful. Even the slightest peristalsis and improve GI blood supply.
change in the air currents moving past the Once bowel sound resumes, the amount of
exposed superficial dermis causes a patient food can be increased.
to experience excruciating pain. Without the
protective covering of the epidermis, nerve Option A: Post-burn GI dysfunction is
endings are sensitized and exposed to caused by multiple factors, and therefore
stimulation. maintaining GI function is a systematic
Option C: Hemorrhage is unusual in a burn engineering project. The therapeutic
injury. The difference with a burn is the heat strategy should not rely on a single
actually stops the blood from flowing. A treatment or a single drug.
small bit of blood may ooze out at first, but it Option C: It is suggested that small doses of
won’t actually bleed much. dopamine should be administered to dilate
Option D: Most commonly, patients with the renal and GI vessels, and free oxygen
carbon monoxide poisoning will present with radical clearing agents to attenuate
headache (more than 90%), dizziness, ischemia/reperfusion injury in the process of
weakness, and nausea. Patients may be resuscitation. These comprehensive
resuscitation measures played an important pattern. An immediate and rapid increase in
role in protecting GI function, helping the water content of burned tissue is seen in
resume bowel sound earlier and digestive the first hour after burn injury.
function. Option C: Inadequate fluid resuscitation is
Option D: In some patients in whom fluid the most common cause of diminished
resuscitation was not implemented distal pulses in the newly burned patient.
effectively for various reasons, wound Another potential cause of diminished
surface infection often caused severe injury pulses is peripheral edema, which develops
to the GI function, or even toxic paralytic in many severe burn patients due to the
ileus palsy, greatly increasing toxin large fluid volumes needed for resuscitation.
absorption and bacterial superinfection. Option D: Hyponatraemia is frequent, and
17. Question the restoration of sodium losses in the burn
Which clinical manifestation indicates that tissue is, therefore, essential hyperkalemia
the burned client is moving into the fluid is also characteristic of this period because
remobilization phase of recovery? of the massive tissue necrosis. Following a
severe burn injury, significant hematologic
A. Increased urine output, decreased urine changes occur that are reflected in
specific gravity complete blood count (CBC)
B. Increased peripheral edema, decreased measurements. Over the first week after
blood pressure injury, HGB and HCT decreased. This
C. Decreased peripheral pulses, slow decrease was due to loss of red blood cells.
capillary refill WBC counts were initially elevated but
D. Decreased serum sodium level, decreased over the first 4 days. PLT also
increased hematocrit decreased over the first 4 days.
Correct 18. Question
Correct Answer: A. Increased urine output, What is the priority nursing diagnosis during
decreased urine specific gravity the first 24 hours for a client with full-
thickness chemical burns on the anterior
The “fluid remobilization” phase improves neck, chest, and all surfaces of the left arm?
renal blood flow, increasing diuresis and
restoring fluid and electrolyte levels. The A. Risk for Ineffective Breathing Pattern
increased water content of the urine B. Decreased Tissue Perfusion
reduces its specific gravity. Injured C. Risk for Disuse Syndrome
capillaries heal approximately 24 to 36 D. Disturbed Body Image
hours after a burn, so intravascular fluid loss Correct
typically ceases at this time, and fluid Correct Answer: C. Risk for Disuse
begins to shift back into the intravascular Syndrome
compartment. This stage is called the fluid
remobilization period. During the emergent phase, fluid shifts into
interstitial tissue in burned areas. When the
Option B: Edema develops when the rate at burn is circumferential on an extremity, the
which fluid is filtered out of the capillaries swelling can compress blood vessels to
exceeds the flow in the lymph vessels. such an extent that circulation is impaired
Edema formation often follows a biphasic distal to the injury, necessitating the
intervention of an escharotomy. Chemical All these findings are abnormal; however,
burns do not cause inhalation injury. only the serum potassium level is changed
to the degree that serious, life-threatening
Option A: Chemical burns do not cause responses could result. With such a rapid
inhalation injury and a disrupted breathing rise in the potassium level, the client is at
pattern. The most common findings high risk of experiencing severe cardiac
represent structural changes to the tissue dysrhythmias and death.
directly affected, for example, the eye, oral
mucosa, skin, esophagus, and lower Option A: Serum sodium is abnormal, but
intestinal system, especially the stomach not to the same degree of severity, and
and pylorus, respiratory system, among would be expected in the emergent phase
others. after a burn injury. Severe cutaneous
Option B: During the emergent phase, fluid injuries such as burn injuries and blast
shifts into interstitial tissue in burned areas. injuries result in the loss of both water and
When the burn is circumferential on an sodium. For burn patients, hypernatremia
extremity, the swelling can compress blood that occurs within a few days of injury may
vessels to such an extent that circulation is be associated with increased risk of death.
impaired distal to the injury, causing Option C: Acid-base studies were carried
decreased tissue perfusion and out on 76 consecutive burn patients
necessitating the intervention of an admitted within 36 hours of injury.
escharotomy. Admission blood pH and base excess (BE)
Option D: Disturbed body image can values all decreased in a linear relationship
develop. Assist the patient to identify the to the extent of the burn. Blood Pco-2
extent of actual change in appearance and changes were unrelated to the extent of the
body function. This helps begin the process burn. Significant acidosis developed within 2
of looking to the future and how life will be hours of burn injury.
different. Option D: The hematocrit (Hct) is the
19. Question percentage of the volume of the whole
All of the following laboratory test results on blood that is made up of red blood cells. In
a burned client’s blood are present during burns, the patient has lost a lot of fluid from
the emergent phase. Which result should leaky blood vessels. There are more red
the nurse report to the physician cells than fluid so the hematocrit is high.
immediately? 20. Question
The client has experienced an electrical
A. Serum sodium elevated to 131 mmol/L injury, with the entrance site on the left hand
(mEq/L) and the exit site on the left foot. What are
B. Serum potassium 7.5 mmol/L (mEq/L) the priority assessment data to obtain from
C. Arterial pH is 7.32 this client on admission?
D. Hematocrit is 52%
Correct A. Airway patency
Correct Answer: B. Serum potassium 7.5 B. Heart rate and rhythm
mmol/L (mEq/L) C. Orientation to time, place, and person
D. Current range of motion in all extremities
Correct
Correct Answer: B. Heart rate and rhythm Correct Answer: D. “In what exact place or
space were you when you were burned?”
Electric current travels through the body
from the entrance site to the exit site and The risk for inhalation injury is greatest
can seriously damage all tissues between when flame burns occur indoors in small,
the two sites. Early cardiac damage from poorly ventilated rooms. The composition of
electrical injury includes irregular heart rate, smoke varies with each fire depending upon
rhythm, and ECG changes. It is also the materials being burned, the amount of
important to obtain the patient’s cardiac oxygen available to the fire, and the nature
history, including any history of prior of the fire. It is important to elucidate
arrhythmias. whether the exposure was to smoke,
flames, and/or possible chemicals (both
Option A: The airway is not at any particular industrial and household). Duration of
risk with this injury. Any patient that was in exposure, the location of exposure (such as
contact with a high voltage source should if it was in an enclosed space), and any loss
have continuous cardiac monitoring during of consciousness are all important as well.
evaluation.
Option C: These patients are specifically at Option A: Although smoking increases the
risk for cardiac damage if the path of the risk for some problems, it does not
current traversed the heart. One may also predispose the client for an inhalation injury.
consider CT imaging of the head if the History-taking should be complete and
patient has altered mental status or thorough. Burn patients may have extensive
associated head trauma from a fall or being external injuries, but smoke inhalation may
thrown in a blast. affect those with no outward signs of burns.
Option D: Range of motion is also Option B: Workup of smoke inhalation injury
important. However, the priority is to make may include serial chest radiographs (often
sure that the heart rate and rhythm are negative early in smoke inhalation injury)
adequate to support perfusion to the brain and computed tomography (CT) chest. A
and other vital organs. delay in the onset of symptoms is not
uncommon, and clinicians should educate
1. Question patients on the possibility of delayed
In assessing the client’s potential for an symptom onset post-exposure. The delayed
inhalation injury as a result of a flame burn, symptoms occur in the lower respiratory
what is the most important question to ask airways as it is caused by chemical toxin
the client on admission? exposure, which may bypass the upper
airways.
A. “Are you a smoker?” Option C: Short-term complications are
B. “When was your last chest x-ray?” seen in more severe injuries within 4 to 5
C. “Have you ever had asthma or any other days, and the most common issue is
lung problem?” pneumonia. Acute respiratory distress
D. “In what exact place or space were you syndrome and pulmonary edema are also
when you were burned?” seen in the short term.
Correct 2. Question
Which information obtained by assessment a pigeon chest (pectus carinatum) or funnel
ensures that the client’s respiratory efforts chest (pectus excavatum).
are currently adequate? 3. Question
The nursing instructor is going over burn
A. The client is able to talk. injuries. The instructor tells the students that
B. The client is alert and oriented. the nursing care priorities for a patient with
C. The client’s oxygen saturation is 97%. a burn injury include wound care, nutritional
D. The client’s chest movements are support, and prevention of complications
uninhibited. such as infection. Based upon these care
Correct priorities, the instructor is most likely
Correct Answer: C. The client’s oxygen discussing a patient in what phase of burn
saturation is 97%. care?

Clients may have ineffective respiratory A. Emergent Phase


efforts and gas exchange even though they B. Immediate Resuscitative Phase
are able to talk, have good respiratory C. Acute Phase
movement, and are alert. The best indicator D. Rehabilitation Phase
for respiratory effectiveness is the Correct
maintenance of oxygen saturation within the Correct Answer: C. Acute Phase
normal range.
The acute or intermediate phase of burn
Option A: A thorough respiratory care follows the emergent/resuscitative
assessment consists of inspection, phase and begins 48 to 72 hours after the
palpation, percussion, and auscultation in burn injury. During this phase, attention is
conjunction with a comprehensive health directed toward continued assessment and
history. Use a systematic approach and maintenance of respiratory and circulatory
compare findings between left and right so status, fluid and electrolyte balance, and
the patient serves as his own control. gastrointestinal function. Infection
Option B: Respirations should be even, prevention, burn wound care that includes
unlabored, and regular at a rate of 12 to 20 wound cleaning, topical antibacterial
breaths per minute. Normally, inspiration is therapy, wound dressing, dressing changes,
half as long as expiration, and chest wound debridement, and wound grafting,
expansion is symmetrical. If the client pain management, and nutritional support
appears anxious or exhibits nasal flaring, are priorities at this stage and are discussed
cyanosis of the lips and mouth, intercostal in detail in the following sections.
retraction, or use of accessory muscles of
respiration, he may be in respiratory Option A: The emergent phase begins with
distress. the onset of burn injury and lasts until the
Option D: Normally, the thorax is completion of fluid resuscitation or a period
symmetrical and the anterior-posterior of about the first 24 hours. During the
diameter is less than the transverse emergent phase, the priority of client care
diameter. (Equal diameters may signal involves maintaining an adequate airway
chronic obstructive pulmonary disease in an and treating the client for burn shock.
adult.) Note any structural deformity such as
Option B: Priorities during the immediate be monitored and fluid rates adjusted
resuscitative phase include first aid, accordingly. Urine output of 0.5 mL/kg or
prevention of shock and respiratory distress, about 30 – 50 mL/hr in adults and 0.5-1.0
detection and treatment of concomitant mL/kg/hr in children less than 30kg is a
injuries, and initial wound assessment and good target for adequate fluid resuscitation.
care. Recent literature has raised concerns about
Option D: The priorities during the complications from over-resuscitation
rehabilitation phase include prevention of described as “fluid creep.” Again, adequate
scars and contractures, rehabilitation, fluid resuscitation is the goal.
functional and cosmetic reconstruction, and Option D: Severe burn is associated with
psychosocial counseling. significant changes in body weight due to
4. Question large resuscitation volumes, fluid shifts, a
The burned client’s family asks at what point hypermetabolic state, prolonged bed rest,
the client will no longer be at increased risk and caloric intake. Weight gain in the
for infection. What is the nurse’s best severely burned patient often follows initial
response? fluid resuscitation, which can increase
weight by up to 10–20 kg.
A. “When fluid remobilization has started.” 5. Question
B. “When the burn wounds are closed.” The burned client relates the following
C. “When IV fluids are discontinued.” history of previous health problems. Which
D. “When body weight is normal.” one should alert the nurse to the need for
Correct alteration of the fluid resuscitation plan?
Correct Answer: B. “When the burn wounds
are closed.” A. Seasonal asthma
B. Hepatitis B 10 years ago
Intact skin is a major barrier to infection and C. Myocardial infarction 1 year ago
other disruptions in homeostasis. No matter D. Kidney stones within the last 6 month
how much time has passed since the burn Correct
injury, the client remains at great risk for Correct Answer: C. Myocardial infarction 1
infection as long as any area of skin is year ago
open.
It is likely the client has a diminished cardiac
Option A: Fluid resuscitation replaces lost output as a result of the old MI and would
fluids and electrolytes and helps prevent be at greater risk for the development of
complications (shock, acute tubular congestive heart failure and pulmonary
necrosis). Once initial fluid resuscitation has edema during fluid resuscitation. The initial
been accomplished, a steady rate of fluid response to severe burn injury or early
administration is preferred to boluses, which shock state is characterized by a decrease
may increase interstitial fluid shifts and in cardiac output and metabolic rate. The
cardiopulmonary congestion. reduction in cardiac output is partially due to
Option C: The important point to remember hypovolemia and reduced venous return.
is the fluid amount calculated is just a
guideline. Patient’s vital signs, mental Option A: Systemic effects may occur
status, capillary refill, and urine output must especially after inhalation injury. Systemic
effects of inhalation injury occur both airway is in severe jeopardy and intubation
indirectly from hypoxia or hypercapnia is highly likely to be needed shortly. Close
resulting from loss of pulmonary function physical examination of patients with
and systemic effects of pro-inflammatory inhalation injury can reveal signs of smoke
cytokines, as well as direct effects from inhalation, including facial burns, perioral
metabolic poisons such as carbon burns, and singed nasal hairs. This warrants
monoxide and cyanide. laryngoscopy and evidence of significant
Option B: A history of hepatitis B does not edema, blisters, or ulcerations should lead
affect the fluid resuscitation plan. Fluid to consideration for intubation to stabilize
creep in patients recovering from acute the airway.
burns still exists, despite the use of a more
treatment conservative approach. Most Option A: Neurovascular assessment may
severe burn patients develop fluid overload be done after establishing a patent airway.
and body weight increase after acute fluid Inhalation of smoke also leads to the
resuscitation. How to quickly return patients absorption of many toxins in the blood,
to their pre-injury body weight is an including carbon monoxide and cyanide,
important issue thereby causing the entire body to be
Option D: Acute renal failure is one of the affected, and making inhalational injury a
major complications of burns and it is systemic insult.
accompanied by a high mortality rate. Most Option B: History taking can be done after
renal failures occur either immediately after the patient has been deemed stable. It is
the injury or at a later period when sepsis well known that rapid diagnosis and
develops. treatment are key when it comes to
6. Question inhalational burns, as acute complications,
The burned client on admission is drooling which sometimes go unnoticed, are the
and having difficulty swallowing. What is the reason behind long-term sequels and the
nurse’s best first action? high mortality rate seen with this type of
injury.
A. Assess level of consciousness and Option D: GI assessment is not a priority.
pupillary reactions. Edema of the oral mucosa and/or the
B. Ask the client at what time food or liquid trachea can develop within 0.5 hours of the
was last consumed. time of injury and can progress to mucosal
C. Auscultate breath sounds over the necrosis within 12-24 hours. Supraglottic
trachea and mainstem bronchi. injury, swelling, and resulting obstruction of
D. Measure abdominal girth and auscultate the airway occur more commonly in children
bowel sounds in all four quadrants. due to the smaller size of the trachea, and
Correct relatively large epiglottis.
Correct Answer: C. Auscultate breath 7. Question
sounds over the trachea and mainstem Which intervention is most important for the
bronchi. nurse to use to prevent infection by cross-
contamination in the client who has open
Difficulty swallowing and drooling are burn wounds?
indications of oropharyngeal edema and
can precede pulmonary failure. The client’s
A. Handwashing on entering the client’s listed. Which laboratory finding indicates the
room possibility of sepsis?
B. Encouraging the client to cough and
deep breathe A. The total white blood cell count is
C. Administering the prescribed tetanus 9000/mm3.
toxoid vaccine B. The lymphocytes outnumber the
D. Changing gloves between cleansing basophils.
different burn areas C. The “bands” outnumber the “segs.”
Incorrect D. The monocyte count is 1,800/mm3.
Correct Answer: A. Handwashing on Incorrect
entering the client’s room Correct Answer: C. The “bands” outnumber
the “segs.”
Cross-contamination occurs when
microorganisms from another person or the Normally, the mature segmented
environment are transferred to the client. neutrophils (“segs”) are the major
Handwashing with soap and water is the population of circulating leukocytes,
best way to get rid of germs in most constituting 55% to 70% of the total white
situations. Emphasize and model good blood count. Fewer than 3% to 5% of the
handwashing techniques for all individuals circulating white blood cells should be the
coming in contact with the patient. less mature “band” neutrophils. A left shift
occurs when the bone marrow releases
Option B: Although all the interventions more immature neutrophils than mature
listed above can help reduce the risk of neutrophils. Such a shift indicates severe
infection, only handwashing can prevent infection or sepsis, in which the client’s
cross-contamination. Airway obstruction immune system cannot keep pace with the
and/or respiratory distress can occur very infectious process.
quickly or may be delayed, e.g., up to 48 hr
after a burn. Option A: The normal WBC count is 4,500
Option C: Tissue destruction and altered to 11,000/mm3. Burn injury causes systemic
defense mechanisms increase the risk of inflammatory response. The magnitude of
developing tetanus or gas gangrene, the changes is roughly a function of burn
especially in deep burns such as those size that is manifested by increased body
caused by electricity. temperature, increased WBC count, and
Option D: Serious complications often can increased metabolic rate, which makes
be avoided by following strict aseptic diagnosis of infection in the burned patient
techniques. Use of a mask, hat, gown, and more difficult.
sterile gloves, and drapes during placement Option B: Peripheral blood lymphocytes
of central venous catheters (CVCs) should represent the most important line of host
be strictly implemented. defense against pathogenic microorganisms
8. Question in humans. Researchers found a reduction
In reviewing the burned client’s laboratory in the number of lymphocytes as well as
report of white blood cell count with WBC, which may contribute to the
differential, all the following results are impairment of general mechanisms for
immune regulation during burn shock and
transition of blood to the level of self- then, the area must be kept in an
regulation. antideformity position.
Option D: The normal monocyte count Option B: The client should not only be in a
ranges from 100-700 per mm3 (2–8%). supine position but there should be a
Severe burn and sepsis profoundly inhibit movement to avoid contractures. Splinting
the functions of DC, monocyte, and and proper positioning will also help achieve
macrophage. These phagocytes are the first the prevention of contractures. As a matter
cellular responders to severe burn injury of importance, movement should be
after acute disruption of the skin barrier. incorporated into the patient’s daily routine
9. Question from their inception to the hospital.
The client has a deep partial-thickness Option D: The burns are in the client’s
injury to the posterior neck. Which posterior neck. Performing active or passive
intervention is most important to use during range of motion (ROM) exercises,
the acute phase to prevent contractures depending on the patient’s level of
associated with this injury? consciousness is crucial in the prevention of
these complications.
A. Place a towel roll under the client’s neck 10. Question
or shoulder. The client has severe burns around the right
B. Keep the client in a supine position hip. Which position is most important to be
without the use of pillows. emphasized by the nurse that the client
C. Have the client turn the head from side maintains to retain maximum function of this
to side 90 degrees every hour while awake. joint?
D. Keep the client in a semi-Fowler’s
position and actively raise the arms above A. Hip maintained in 30-degree flexion, no
the head every hour while awake. knee flexion
Correct B. Hip flexed 90 degrees and knee flexed
Correct Answer: C. Have the client turn the 90 degrees
head from side to side 90 degrees every C. Hip, knee, and ankle all at maximum
hour while awake. flexion
D. Hip at zero flexion with leg flat
The function that would be disrupted by a Incorrect
contracture to the posterior neck is flexion. Correct Answer: D. Hip at zero flexion with
Moving the head from side to side prevents leg flat
such a loss of flexion. Deformities and
contractures can often be prevented by Maximum function for ambulation occurs
proper positioning. Maintaining proper body when the hip and leg are maintained at full
alignment when the patient is in bed is vital. extension with neutral rotation. Although the
This movement is what would prevent client does not have to spend 24 hours at a
contractures from occurring. time in this position, he or she should be in
this position (in bed or standing) more of the
Option A: Placing a towel roll under the time than with the hip in any degree of
neck might not help prevent contractures. flexion.
Immobilization is only allowed when a part
of the body has just been grafted. Even
Option A: Anti-contracture positioning and
splinting must start from day one and may Gentamicin does not stimulate pain in the
continue for many months post-injury. Legs wound. The small, pale pink bumps in the
should be positioned in a neutral position wound bed are areas of re-epithelialization
ensuring that the patient is not externally and not an adverse reaction. Gentamicin is
rotating at the hips. nephrotoxic and sufficient amounts can be
Option B: When burns occur to the flexor absorbed through burn wounds to affect
aspect of a joint or limb the risk of kidney function. Any client receiving
contracture is greater. This is due to the gentamicin by any route should have kidney
position of comfort being a flexed position; function monitored.
also the flexor muscles are generally
stronger than the extensors so should a Option A: Gentamicin does not stimulate
burn occur to the extensor aspect, patients pain in the wound. The gentamicin is prone
can use the strength of the flexors to stretch to accumulate in the renal proximal tubular
the particular area. cells and can cause damage. Hence, mild
Option C: Patients rest in a position of proteinuria and reduction of the glomerular
comfort; this is generally a position of flexion filtration rate are potential consequences of
and also the position of contracture. Without gentamicin use, achieving 14% of
ongoing advice and help with positioning, gentamicin users in a review.
the patient will continue to take the position Option B: The small, pale pink bumps in the
of contracture and can quickly lose ROM in wound bed are areas of re-epithelialization
multiple joints. Once contracture starts to and not an adverse reaction. Renal function
develop it can be a constant battle to should be evaluated twice-weekly in
achieve full movement, so preventative patients without previous renal disease
measures to minimize contracture through serum creatinine and blood urea
development are necessary. nitrogen. Periodic microscopic urinalysis is
11. Question also vital to detect proteinuria and casts,
During the acute phase, the nurse applied which may indicate kidney injury.
gentamicin sulfate (topical antibiotic) to the Option C: The possible hypersensitivity
burn before dressing the wound. The client manifestations of gentamicin are urticaria,
has all the following manifestations. Which eosinophilia, delayed-type hypersensitivity
manifestation indicates that the client is reaction (Stevens-Johnson syndrome and
having an adverse reaction to this topical toxic epidermal necrolysis), angioedema,
agent? and anaphylactic shock. The clinical
manifestations should guide the treatment
A. Increased wound pain 30 to 40 minutes strategy.
after drug application 12. Question
B. Presence of small, pale pink bumps in The client, who is 2 weeks postburn with a
the wound beds 40% deep partial-thickness injury, still has
C. Decreased white blood cell count open wounds. On taking the morning vital
D. Increased serum creatinine level signs, the client is found to have a below-
Correct normal temperature, is hypotensive, and
Correct Answer: D. Increased serum has diarrhea. What is the nurse’s best
creatinine level action?
bacteria are present at low concentrations
A. Nothing, because the findings are (<105 colony-forming units [CFU]) on the
normal for clients during the acute phase of wound’s surface. This situation often is
recovery. accompanied by signs of sepsis and
B. Increase the temperature in the room changes in the burn wound such as black,
and increase the IV infusion rate. blue, or brown discoloration of the eschar.
C. Assess the client’s airway and oxygen 13. Question
saturation. Which intervention is most important to use
D. Notify the burn emergency team. to prevent infection by autocontamination in
Correct the burned client during the acute phase of
Correct Answer: D. Notify the burn recovery?
emergency team.
A. Changing gloves between wound care
These findings are associated with systemic on different parts of the client’s body.
gram-negative infection and sepsis. This is B. Avoiding sharing equipment such as
a medical emergency and requires prompt blood pressure cuffs between clients.
attention. Invasive infection of burn wounds C. Using the closed method of burn wound
is a surgical emergency because of the high management.
concentrations of bacteria (>105 CFU) in D. Using proper and consistent
the wound and surrounding area, together handwashing.
with new areas of necrosis in unburned Correct
tissues. Correct Answer: A. Changing gloves
between wound care on different parts of
Option A: Invasive infection is now the chief the client’s body.
reason for death and morbidity after burn
injury, with it being responsible for 51% of Autocontamination is the transfer of
the deaths. The importance of prevention, microorganisms from one area to another
surveillance, and sampling for infections in area of the same client’s body, causing
this immunocompromised group has been infection of a previously uninfected area.
well established; however, there is a dearth Use gowns, gloves, masks, and strict
of standard-of-care guidelines and novel aseptic techniques during direct wound care
approaches. and provide sterile or freshly laundered bed
Option B: Urgent resuscitation measures linens or gowns.
are required, along with broad-spectrum
antimicrobial agents, antifungals, and Option B: Although all techniques listed can
surgical debridement of the affected area. help reduce the risk of infection, only
Specimens of this tissue must undergo changing gloves between carrying out
histopathologic and microbiologic analysis wound care on different parts of the client’s
to assist in the identification of the causative body can prevent autocontamination.
organism(s). Depending on the type or extent of wounds
Option C: Assessment of the airway and and the choice of wound treatment (open
oxygen saturation would not help in versus closed), isolation may range from a
diagnosing a burn infection. Burn wound simple wound and/or skin to complete or
colonization may be diagnosed when reverse to reduce the risk of cross-
contamination and exposure to multiple grafting without compromising graft take if
bacterial flora. external compression is applied.
Option C: Prevent skin-to-skin surface Option B: Following thermal injury, the
contact (wrap each burned finger or toe innate immune system responds
separately; do not allow burned ear to touch immediately by stimulating localized and
scalp). This identifies the presence of systemic inflammatory reactions. The innate
healing (granulation tissue) and provides for immune response participates in activating
early detection of burn-wound infection. the adaptive immune response; however, in
Infection in a partial-thickness burn may so doing it has an adverse effect on the
cause conversion of burn to full-thickness burn victim’s ability to mount a vigorous
injury. immune response to invading
Option D: Emphasize and model good microorganisms and, therefore, predisposes
handwashing techniques for all individuals the burn victim to infectious complications.
coming in contact with the patient. This Option C: Pain control is obtainable by
prevents cross-contamination and reduces performing therapies during wound dressing
the risk of acquired infection. and debridement, if possible. Analgesics
14. Question should also be administered prior to therapy
When should ambulation be initiated in the sessions to encourage participation in
client who has sustained a major burn? movement activities.
15. Question
A. When all full-thickness areas have been What statement by the client indicates the
closed with skin grafts need for further discussion regarding the
B. When the client’s temperature has outcome of skin grafting (allografting)
remained normal for 24 hours procedures?
C. As soon as possible after wound
debridement is complete A. “For the first few days after surgery, the
D. As soon as possible after the resolution donor sites will be painful.”
of the fluid shift B. “Because the graft is my own skin, there
Incorrect is no chance it won’t ‘take’.”
Correct Answer: D. As soon as possible C. “I will have some scarring in the area
after the resolution of the fluid shift when the skin is removed for grafting.”
D. “Once all grafting is completed, my risk
Regular, progressive ambulation is initiated for infection is the same as it was before I
for all burn clients who do not have was burned.”
contraindicating concomitant injuries as Correct
soon as the fluid shift resolves. Clients can Correct Answer: B. “Because the graft is my
be ambulated with extensive dressings, own skin, there is no chance it won’t ‘take’.”
open wounds, and nearly any type of
attached lines, tubing, and other equipment. Factors other than tissue type, such as
circulation and infection, influence whether
Option A: The consistent finding in the and how well a graft “takes.” The client
literature is that early ambulation can be should be prepared for the possibility that
safely initiated after lower extremity skin not all grafting procedures will be
successful. Graft survival depends on the
diffusion of nutrients and oxygen from the C. “I will be fully recovered when I am able
wound bed known as imbibition. to perform all the activities I did before my
Inosculation then follows when the blood injury.”
vessels of the graft and from the wound bed D. “I will be fully recovered when I achieve
grow together to make end-to-end contact. the highest possible level of functioning that
Lastly, neovascularization occurs when new I can.”
blood vessels grow from the wound bed into Correct
the graft. Correct Answer: D. “I will be fully recovered
when I achieve the highest possible level of
Option A: The donor sites will be painful functioning that I can.”
after the surgery. Silicone gel sheets, along
with pressure dressings, have shown a Although a return to pre-burn functional
dramatic decrease in pain, pruritus, and levels is rarely possible, burned clients are
scar thickness six months after burn injury. considered fully recovered or rehabilitated
Option C: There can be scarring in the area when they have achieved their highest
where the skin is removed for grafting. Burn possible level of physical, social, and
scars are a common occurrence after skin emotional functioning. The technical
grafting and can cause anxiety, depression, rehabilitative phase of rehabilitation begins
pain, itching, altered pigmentation, with wound closure and ends when the
temperature intolerance, and decreased client returns to her or his highest possible
range of motion secondary to scar level of functioning.
contracture. Scar formation is propagated
by deficiencies in the biosynthetic and Option A: Rehabilitation of burns patients is
tissue degradation pathway during wound a continuum of active therapy starting from
healing. admission. There should be no delineation
Option D: The client is still at risk for between an ‘acute phase’ and a
infection. Early failure of graft survival is ‘rehabilitation phase’ as this idea can
attributable to seroma and hematoma promote the inequality of secondary
formation, which lifts the graft off the wound disjointed scar management and/or
bed, preventing imbibition. Other factors functional rehabilitation teams.
that lead to graft failure include shearing Option B: The final stage in caring for a
forces, edematous tissue, and infected patient with a burn injury is the rehabilitative
tissue. stage. This stage starts with the closure of
16. Question the burn and ends when the patient has
Which statement by the client indicates a reached the optimal level of functioning. The
correct understanding of rehabilitation after focus is on helping the patient return to a
burn injury? normal injury-free life. Helping the patient
adjust to the changes the injury has
A. “I will never be fully recovered from the imposed is also a priority.
burn.” Option C: Early compliance is essential to
B. “I am considered fully recovered when ensure the best possible long-term outcome
all the wounds are closed.” and also to ease pain and assist with
exercise regimes. Patients need to adhere
to a positioning regime in the early stages of
healing and this takes teamwork and purpose of wearing the mask. A widespread
dedication. modality of prevention and treatment of
17. Question hypertrophic scarring is the utilization of
Which statement made by the client with pressure garment therapy (PGT).
facial burns who has been prescribed to Option C: Scars will still be present. This
wear a facial mask pressure garment treatment modality continues to be a
indicates a correct understanding of the clinically accepted practice. It is the most
purpose of this treatment? common therapy used for the treatment and
prevention of abnormal scars after burn
A. “After this treatment, my ears will not injury particularly in North America, Europe,
stick out.” and Scandinavia where it is considered
B. “The mask will help protect my skin from routine practice and regarded as the
sun damage.” preferred conservative management with
C. “Using this mask will prevent scars from reported thinning and better pliability
being permanent.” ranging from 60% to 85%.
D. “My facial scars should be less severe 18. Question
with the use of this mask.” What is the priority nursing diagnosis for a
Correct client in the rehabilitative phase of recovery
Correct Answer: D. “My facial scars should from a burn injury?
be less severe with the use of this mask.”
A. Acute Pain
The purpose of wearing the pressure B. Impaired Adjustment
garment over burn injuries for up to 1 year is C. Deficient Diversional Activity
to prevent hypertrophic scarring and D. Imbalanced Nutrition: Less than Body
contractures from forming. Hypertrophic Requirements
burn scars pose a challenge for burn Correct
survivors and providers. In many cases, Correct Answer: B. Impaired Adjustment
they can severely limit a burn survivor’s
level of function, including work and Recovery from a burn injury requires a lot of
recreational activities. work on the part of the client and significant
others. Seldom is the client restored to the
Option A: The pressure garment will not preburn level of functioning. Adjustments to
change the angle of the ear attachment to changes in appearance, family structure,
the head. By applying pressure to the burn employment opportunities, role, and
or scar, the face mask keeps the skin soft functional limitations are only a few of the
and flat during the scar-forming phase of numerous life-changing alterations that
healing. It helps the face heal with the least must be made or overcome by the client.
amount of scarring. The transparent face
mask is worn 18-20 hours every day for 8 Option A: By the rehabilitation phase, acute
months to 2 years until the skin graft is pain from the injury or its treatment is no
mature. longer a problem. This stage starts with the
Option B: Although the mask does provide closure of the burn and ends when the
protection of sensitive newly healed skin patient has reached the optimal level of
and grafts from sun exposure, this is not the functioning. The focus is on helping the
patient return to a normal injury-free life. cardiac output is a hallmark in this early
Helping the patient adjust to the changes post-injury phase.
the injury has imposed is also a priority. Option B: Increase in transcapillary
Option C: Diversional activity for pain is permeability results in a rapid transfer of
applicable during the intermediate phase of water, inorganic solutes, and plasma
the burn injury. Provide diversional activities proteins between the intravascular and
appropriate for age and condition. This interstitial spaces. Subsequently,
helps lessen concentration on pain intravascular hypovolemia and
experience and refocus attention. haemoconcentration develop and maximum
Option D: Imbalanced nutrition is more levels are reached within 12 hours after
appropriate during the emergent and injury.
intermediate phases of the burn injury. Option D: Disruption of sodium-ATPase
Appropriate guides to proper caloric intake activity presumably causes an intracellular
include 25 kcal/kg body weight, plus 40 kcal sodium shift which contributes to
per percentage of TBSA burn in the adult. hypovolemia and cellular edema. Heat
As the burn wound heals, the percentage of injury also initiates the release of
burned areas is reevaluated to calculate inflammatory and vasoactive mediators.
prescribed dietary formulas, and appropriate These mediators are responsible for local
adjustments are made. vasoconstriction, systemic vasodilation, and
19. Question increased transcapillary permeability.
Nurse Faith should recognize that fluid shift 20. Question
in a client with burn injury results from an Louie, with burns over 35% of the body,
increase in the: complains of chilling. In promoting the
client’s comfort, the nurse should:
A. Total volume of circulating whole blood
B. Total volume of intravascular plasma A. Maintain room humidity below 40%
C. Permeability of capillary walls B. Place top sheet on the client
D. Permeability of kidney tubules C. Limit the occurrence of drafts
Correct D. Keep room temperature at 80 degrees
Correct Answer: C. Permeability of capillary Incorrect
walls Correct Answer: C. Limit the occurrence of
drafts
In burn, the capillaries and small vessels
dilate, and cell damage causes the release A client with burns is very sensitive to
of a histamine-like substance. The temperature changes because heat is lost in
substance causes the capillary walls to the burn areas. Changes in location,
become more permeable and significant character, intensity of pain may indicate
quantities of fluid are lost. developing complications (limb ischemia) or
herald improvement and/or return of nerve
Option A: The steady intravascular fluid loss function and sensation.
due to these sequences of events requires
sustained replacement of intravascular Option A: Maintain comfortable
volume in order to prevent end-organ environmental temperature, provide heat
hypoperfusion and ischemia. Reduced lamps, heat-retaining body coverings.
Temperature regulation may be lost with
major burns. External heat sources may be A patient is admitted to the emergency
necessary to prevent chilling. department after sustaining abdominal
Option B: Cover wounds as soon as injuries and a broken femur from a motor
possible unless open-air exposure burn vehicle accident. The patient is pale,
care method is required. Temperature diaphoretic, and is not talking coherently.
changes and air movement can cause great Vital signs upon admission are temperature
pain to exposed nerve endings. 98, heart rate 130 beats/minute, respiratory
Option D: The major burn patient needs a rate 34 breaths/minute, blood pressure
body temperature greater than 37 – 37.5ºC 50/40. The healthcare provider suspects
to reach 38.5ºC, to avoid critical which type of shock?
temperature and decrease energy
expenditure, controlling hypercatabolic A. Cardiogenic
state. The recommended ambient B. Hypovolemic
temperature in large burn units is between C. Neurogenic
28 and 33ºC. D. Distributive
B. Hypovolemic
After falling from a 10' ladder, a patient is
brought to the emergency department. The The healthcare provider is caring for a
patient is alert, reports back pain, and patient with a diagnosis of hemorrhagic
difficulty moving the lower extremities. pancreatitis. The patient's central venous
Which additional observation is an pressure (CVP) reading is 2, blood pressure
indication the patient may be experiencing is 90/50, lung sounds are clear, and jugular
neurogenic shock? veins are flat. Which of these actions is
most appropriate for the nurse to take?
A. Cool and pale skin
B. Increased systolic blood pressure A. Slow the IV infusion rate
C. Poor skin turgor B. Administer dopamine
D. Bradycardia C. No interventions are needed at this time
D. Bradycardia D. Increase the IV infusion rate
D. Increase the IV infusion rate
A patient who is in hypovolemic shock has
the following clinical signs: Heart rate 120 A child is brought to the emergency room by
beats/minute, blood pressure 80/55 and his mother who reports the child was stung
urine output 20ml/hr. After administering an by a bee while playing in the back yard. The
IV fluid bolus, which of these signs if noted child has an itchy rash on the face, neck,
by the healthcare provider is the best and chest. Breathing is labored with audible
indication of improved perfusion? wheezing. Which of these medications
should the healthcare provider administer
A. Right atrial pressure increases first?
B. Systolic blood pressure increases to 85
C. Urine output increases to 30ml/hr A. Diphenhydramine
D. Heart rate drops to 100 beats/minute B. Albuterol
C. Urine output increases to 30ml/hr C. Epinephrine
D. Dopamine
C. Epinephrine The healthcare provider is caring for a
patient who has septic shock. Which of
A patient who has pericarditis related to these should the healthcare provider
radiation therapy, becomes dyspneic, and administer to the patient first?
has a rapid, weak pulse. Heart sounds are
muffled, and a 12mmHg drop in blood A. Antibiotics to treat the underlying
pressure is noted on inspiration. The infection.
healthcare provider's interventions are B. Corticosteroids to reduce inflammation.
aimed at preventing which type of shock? C. IV fluids to increase intravascular
volume.
A. Distributive D. Vasopressors to increase blood
B. Neurogenic pressure.
C. Obstructive C. IV fluids to increase intravascular volume
D. Cardiogenic
C. Obstructive A patient who is in cardiogenic shock has a
urine output of 20mL/hr. When further
An infant was delivered to a mother with a assessing the patient's renal function, what
diagnosis of chorioamnionitis. The infant is additional findings are anticipated? Select
lethargic, tachypneic, and has an axillary all that apply.
temperature of 96.8. The healthcare
provider suspects septic shock. Which of A. Decreased urine specific gravity
these assessments is an indication that this B. Increased blood urea nitrogen (BUN)
infant is compensating by increasing cardiac C. Decreased urine sodium
output? D. Decreased serum creatinine
B Increased BUN
A. Active precordium C. Decreased urine sodium
B. Warm, flushed skin
C. Bounding pulses Which stage of shock is associated with the
D. Tachycardia worsening of tissue hypoperfusion and
D. Tachycardia onset of worsening circulatory and
metabolic imbalances, including acidosis?
A patient is being treated for hemorrhagic A. Initial nonprogressive phase
shock secondary to multiple rib fractures B. Developing phase
and a lacerated liver. Two units of packed C. Progressive stage
red blood cells have been administered. D. Irreversible stage
Which of these measurements is an C. Progressive Stage
indication the patient has received adequate
volume replacement? What are characteristics of the irreversible
A. Oxygen saturation 90% stage of shock?
B. Increased serum creatinine
C. Decreased right atrial pressure A. The worsening of tissue hypoperfusion
D. Decreased serum lactate and the onset of worsening circulatory and
D. Decreased serum lactate metabolic imbalances, including acidosis
B. The body tries to initiate compensatory A. An advanced practice nurse and an
mechanisms experienced LPN/LVN
C. Nothing can correct the hemodynamic B. An experienced LPN/LVN and an
defect inexperienced RN
D. Tissue and cell damage is too great C. An experienced RN and an
tissue and necrosis of the tissue will occur inexperienced RN
even if the underlying hemodynamic defect D. An experienced RN and a nursing
is corrected assistant
D. Tissue and cell damage is too great; C. Triage requires at least one experienced
tissue and necrosis of the tissue will occur RN. Pairing an experienced RN with
even if the underlying hemodynamic defect inexperienced RN provides opportunities for
is corrected mentoring. Advanced practice nurses are
qualified to perform triage; however, their
Your patient is in the progressive stage of services are usually required in other areas
shock. If the shock is not corrected and of the ED. An LPN/LVN is not qualified to
tissue hypoxia occurs, what would happen perform the initial patient assessment or
with the patients metabolism? decision making. Pairing an experienced
RN with a nursing assistant is the second
A. The tissues will undergo anaerobic best option, because the assistant can
metabolism, creating lactic acid and obtain vital signs and assist in transporting.
lowering the tissue pH
B. Vasomotor reflex of arteriolar constriction You are working in the triage area of an ED,
that reduces pooling of blood in the and four patients approach the triage desk
microcirculation at the same time. List the order in which you
C. Metabolism would be unaffected by the will assess these patients. a. An
lower oxygen level for the 2-3 hours ambulatory, dazed 25-year-old male with a
A. The tissues will undergo anaerobic bandaged head wound b. An irritable infant
metabolism, creating lactic acid and with a fever, petechiae, and nuchal rigidity
lowering the tissue pH c. A 35-year-old jogger with a twisted ankle,
having pedal pulse and no deformity d. A
Which type of shock is associated with low 50-year-old female with moderate
blood levels? abdominal pain and occasional vomiting
1. A B D C
A. Hypovolemic shock 2. B A D C
B. Septic shock 3. C D B A
C. Anaphylactic shock 4. C B A D
D. Cardiogenic shock 2. An irritable infant with fever and
A. Hypovolemic Shock petechiae should be further assessed for
other meningeal signs. The patient with the
You are the charge nurse in an emergency head wound needs additional history and
department (ED) and must assign two staff assessment for intracranial pressure. The
members to cover the triage area. Which patient with moderate abdominal pain is
team is the most appropriate for this uncomfortable, but not unstable at this
assignment? point. For the ankle injury, medical
evaluation can be delayed 24 - 48 hours if comes down. Which of the following would
necessary. be appropriate to delegate to the nursing
assistant?
In conducting a primary survey on a trauma
patient, which of the following is considered A. Assist the child to remove outer clothing.
one of the priority elements of the primary B. Advise the parent to use acetaminophen
survey? instead of aspirin.
C. Explain the need for cool fluids.
A. Complete set of vital signs D. Prepare and administer a tepid bath.
B. Palpation and auscultation of the A. The nursing assistant can assist with the
abdomen removal of the outer clothing, which allows
C. Brief neurologic assessment the heat to dissipate from the child's skin.
D. Initiation of pulse oximetry Advising and explaining are teaching
C. A brief neurologic assessment to functions that are the responsibility of the
determine level of consciousness and pupil RN. Tepid baths are not usually performed
reaction is part of the primary survey. Vital because of potential for rebound and
signs, assessment of the abdomen, and shivering.
initiation of pulse oximetry are considered
part of the secondary survey. It is the summer season, and patients with
Question 4 CORRECT signs and symptoms of heat-related illness
present in the ED. Which patient needs
A 56-year-old patient presents in triage with attention first?
left-sided chest pain, diaphoresis, and
dizziness. This patient should be prioritized A. An elderly person complains of dizziness
into which category? and syncope after standing in the sun for
several hours to view a parade
A. High urgent
B. Urgent B. A marathon runner complains of severe
C. Non-urgent leg cramps and nausea. Tachycardia,
D. Emergent diaphoresis, pallor, and weakness are
D. Chest pain is considered an emergent observed.
priority, which is defined as potentially life-
threatening. Patients with urgent priority C. A previously healthy homemaker reports
need treatment within 2 hours of triage (e.g. broken air conditioner for days. Tachypnea,
kidney stones). Non-urgent conditions can hypotension, fatigue, and profuse
wait for hours or even days. (High urgent is diaphoresis are observed.
not commonly used; however, in 5-tier
triage systems, High urgent patients fall D. A homeless person, poor historian,
between emergent and urgent in terms of presents with altered mental status, poor
the time lapsing prior to treatment). muscle coordination, and hot, dry, ashen
skin. Duration of exposure is unknown.
The physician has ordered cooling D. The homeless person has symptoms of
measures for a child with fever who is likely heat stroke, a medical emergency, which
to be discharged when the temperature increases risk for brain damage. Elderly
patients are at risk for heat syncope and and no palpable pulse. Which task is
should be educated to rest in cool area and appropriate to delegate to the nursing
avoid future similar situations. The runner is assistant?
having heat cramps, which can be managed A. Chest compressions
with rest and fluids. The housewife is B. Bag-valve mask ventilation
experiencing heat exhaustion, and C. Assisting with oral intubation
management includes fluids (IV or D. Placing the defibrillator pads
parenteral) and cooling measures. The A. Nursing assistants are trained in basic
prognosis for recovery is good. cardiac life support and can perform chest
compressions. The use of the bag-valve
You respond to a call for help from the ED mask requires practice and usually a
waiting room. There is an elderly patient respiratory therapist will perform this
lying on the floor. List the order for the function. The nurse or the respiratory
actions that you must perform. therapist should provide PRN assistance
during intubation. The defibrillator pads are
a. Perform the chin lift or jaw thrust clearly marked; however, placement should
maneuver. be done by the RN or physician because of
b. Establish unresponsiveness. the potential for skin damage and electrical
c. Initiate cardiopulmonary resuscitation arcing.
(CPR).
d. Call for help and activate the code team. An anxious 24-year-old college student
e. Instruct a nursing assistant to get the complains of tingling sensations,
crash cart. palpitations, and chest tightness. Deep,
BDACE rapid breathing and carpal spasms are
noted. What priority nursing action should
Establish unresponsiveness first. (The you take?
patient may have fallen and sustained a
minor injury.) If the patient is unresponsive, A. Notify the physician immediately.
get help and have someone initiate the B. Administer supplemental oxygen.
code. Performing the chin lift or jaw thrust C. Have the student breathe into a paper
maneuver opens the airway. The nurse is bag.
then responsible for starting CPR. CPR D. Obtain an order for an anxiolytic
should not be interrupted until the patient medication
recovers or it is determined that heroic C. The patient is hyperventilating secondary
efforts have been exhausted. A crash cart to anxiety, and breathing into a paper bag
should be at the site when the code team will allow rebreathing of carbon dioxide.
arrives; however, basic CPR can be Also, encouraging slow breathing will help.
effectively performed until the team arrives. Other treatments such as oxygen and
medication may be needed if other causes
The emergency medical service (EMS) has are identified.
transported a patient with severe chest pain.
As the patient is being transferred to the An experienced traveling nurse has been
emergency stretcher, you note assigned to work in the ED; however, this is
unresponsiveness, cessation of breathing, the nurse's first week on the job. Which
area of the ED is the most appropriate B. Collect hair samples, saliva swabs, and
assignment for the nurse? scrapings beneath fingernails.
C. Provide emotional support and
A. Trauma team supportive communication.
B. Triage D. Ensure that the "chain of custody" is
C. Ambulatory or fast track clinic maintained
D. Pediatric medicine team C. The LPN/LVN is able to listen and
C. The fast track clinic will deal with provide emotional support for her patients.
relatively stable patients. Triage, trauma, The other tasks are the responsibility of an
and pediatric medicine should be staffed RN or, if available, a SANE (sexual assault
with experienced nurses who know the nurse examiner) who has received training
hospital routines and policies and can to assess, collect and safeguard evidence,
rapidly locate equipment. and care for these victims.

A tearful parent brings a child to the ED for You are caring for a victim of frostbite to the
taking an unknown amount of children's feet. Place the following interventions in the
chewable vitamins at an unknown time. The correct order.
child is currently alert and asymptomatic.
What information should be immediately a. Apply a loose, sterile, bulky dressing.
reported to the physician? b. Give pain medication.
c. Remove the victim from the cold
A. The ingested children's chewable environment.
vitamins contain iron. d. Immerse the feet in warm water 100o F to
B. The child has been treated several times 105o F (40.6o C to 46.1o C)
for ingestion of toxic substances. CBDA
C. The child has been treated several times
for accidental injuries. The victim should be removed from the cold
D. The child was nauseated and vomited environment first, and then the rewarming
once at home process can be initiated. It will be painful, so
A. Iron is a toxic substance that can lead to give pain medication prior to immersing the
massive hemorrhage, coma, shock, and feet in warmed water.
hepatic failure. Deferoxamine is an antidote
that can be used for severe cases of iron A patient sustains an amputation of the first
poisoning. Other information needs and second digits in a chainsaw accident.
additional investigation, but will not change Which task should be delegated to the
the immediate diagnostic testing or LPN/LVN?
treatment plan.
A. Gently cleanse the amputated digits with
In caring for a victim of sexual assault, Betadine solution.
which task is most appropriate for an B. Place the amputated digits directly into
LPN/LVN? ice slurry.
A. Assess immediate emotional state and C. Wrap the amputated digits in sterile
physical injuries gauze moistened with saline.
D. Store the amputated digits in a solution C. Parent refusal is an absolute
of sterile normal saline contraindication; therefore, the physician
C. The only correct intervention is C. the must be notified. Tetanus status can be
digits should be gently cleansed with normal addressed later. The RN can restart the IV
saline, wrapped in sterile gauze moistened and provide information about conscious
with saline, and placed in a plastic bag or sedation; if the parent still not satisfied, the
container. The container is then placed on physician can give more information.
ice.
An intoxicated patient presents with slurred
A 36-year-old patient with a history of speech, mild confusion, and uncooperative
seizures and medication compliance of behavior. The patient is a poor historian but
phenytoin (Dilantin) and carbamazepine admits to "drinking a few on the weekend."
(Tegretol) is brought to the ED by the MS What is the priority nursing action for this
personnel for repetitive seizure activity that patient?
started 45 minutes prior to arrival. You
anticipate that the physician will order which A. Obtain an order for a blood alcohol level.
drug for status epilepticus? B. Contact the family to obtain additional
history and baseline information.
A. PO phenytoin and carbamazepine C. Administer naloxone (Narcan) 2 - 4 mg
B. IV lorazepam (Ativan) as ordered.
C. IV carbamazepine D. Administer IV fluid support with
D. IV magnesium sulfate supplemental thiamine as ordered
B. IV Lorazepam (Ativan) is the drug of D. The patient presents with symptoms of
choice for status epilepticus. Tegretol is alcohol abuse and there is a risk for
used in the management of generalized Wernicke's syndrome, which is caused by a
tonic-clonic, absence or mixed type thiamine deficiency. Multiples drug abuse is
seizures, but it does not come in an IV form. not uncommon; however, there is nothing in
PO (per os) medications are inappropriate the question that suggests an opiate
for this emergency situation. Magnesium overdose that requires naloxone. Additional
sulfate is given to control seizures in information or the results of the blood
toxemia of pregnancy. alcohol level are part of the total treatment
plan but should not delay the immediate
You are preparing a child for IV conscious treatment.
sedation prior to repair of a facial laceration.
What information should you immediately When an unexpected death occurs in the
report to the physician? ED, which of the following tasks is most
appropriate to delegate to the nursing
A. The parent is unsure about the child's assistant?
tetanus immunization status.
B. The child is upset and pulls out the IV. A. Escort the family to a place of privacy.
C. The parent declines the IV conscious B. Go with the organ donor specialist to talk
sedation. to the family.
D. The parent wants information about the C. Assist with postmortem care.
IV conscious sedation. D. Assist the family to collect belongings
C. Postmortem care requires some turning, C. "The stick was really dirty and covered
cleaning, lifting, etc., and the nursing with mud."
assistant is able to assist with these duties. D. "He's a diabetic, so he needs attention
The RN should take responsibility for the right away
other tasks to help the family begin the B. An impaled object may be providing a
grieving process. In cases of questionable tamponade effect, and removal can
death, belongings may be retained for precipitate sudden hemodynamic
evidence, so the chain of custody would decompensation. Additional history
have to be maintained. including a more definitive description of the
blood loss, depth of penetration, and
Following emergency endotracheal medical history should be obtained. Other
intubation, you must verify tube placement information, such as the dirt on the stick or
and secure the tube. List in order the steps history of diabetes, is important in the
that are required to perform this function? overall treatment plan, but can be
addressed later.
a. Obtain an order for a chest x-ray to
document tube placement. You are assigned to telephone triage. A
b. Secure the tube in place. patient who was stung by a common honey
c. Auscultate the chest during assisted bee calls for advice, reports pain and
ventilation. localized swelling, but denies any
d. Confirm that the breath sounds are equal respiratory distress or other systemic signs
and bilateral. of anaphylaxis. What is the action that you
CDBA should direct the caller to perform?

Auscultating and confirming equal bilateral A. Call 911.


breath sounds should be performed in rapid B. Remove the stinger by scraping.
succession. If the sounds are not equal or if C. Apply a cool compress.
the sounds are heard over the mid- D. Take an oral antihistamine
epigastric area, tube placement must be B. The stinger will continue to release
corrected immediately. Securing the tube is venom into the skin, so prompt removal of
appropriate while waiting for the x-ray study. the stinger is advised. Cool compresses and
antihistamines can follow. The caller should
A teenager arrives by private car. He is alert be further advised about symptoms that
and ambulatory, but this shirt and pants are require 911 assistance.
covered with blood. He and his hysterical
friends are yelling and trying to explain that These patients present to the ED
that they were goofing around and he got complaining of acute abdominal pain.
poked in the abdomen with a stick. Which of Prioritize them in order of severity.
the following comments should be given first
consideration? a. A 35-year-old male complaining of
A. "There was a lot of blood and we used severe, intermittent cramps with three
three bandages." episodes of watery diarrhea, 2 hours after
B. "He pulled the stick out, just now, eating
because it was hurting him."
b. A 11-year-old boy with a low-grade fever, B. Gross deformity in a lower extremity
left lower quadrant tenderness, nausea, and C. Decreased bowel sounds
anorexia for the past 2 days D. Hematuria
c. A 40-year-old female with moderate left A. A deviated trachea is a symptoms of
upper quadrant pain, vomiting small tension pneumothorax. All of the other
amounts of yellow bile, and worsening symptoms need to be addressed, but are of
symptoms over the past week lesser priority.
d. A 56-year-old male with a pulsating
abdominal mass and sudden onset of A patient in a one-car rollover presents with
pressure-like pain in the abdomen and flank multiple injuries. Prioritize the interventions
within the past hour that must be initiated for this patient.
DBCA
a. Secure/start two large-bore IVs with
The patient with a pulsating mass has an normal saline
abdominal aneurysm that may rupture and b. Use the chin lift or jaw thrust method to
he may decompensate suddenly. The 11- open the airway.
year-old boy needs evaluation to rule out c. Assess for spontaneous respirations
appendicitis. The woman needs evaluation d. Give supplemental oxygen per mask.
for gallbladder problems that appear to be e. Obtain a full set of vital signs.
worsening. The 35-year-old man has food f. Remove patient's clothing.
poisoning, which is usually self-limiting g. Insert a Foley catheter if not
contraindicated.
The nursing manager decides to form a CBDAEFG
committee to address the issue of violence
against ED personnel. Which combination For a multiple trauma victim, many
of employees is best suited to fulfill this interventions will occur simultaneously as
assignment? team members assist in the resuscitation.
Methods to open the airway such as the
A. ED physicians and charge nurses chin lift or jaw thrust can be used
B. Experienced RNs and experienced simultaneously while assessing for
paramedics spontaneous respirations. However, airway
C. RNs, LPN/LVNs, and nursing assistants and oxygenation are priority. Starting IVs for
D. At least one representative from each fluid resuscitation is part of supporting
group of ED personnel circulation. (EMS will usually establish at
D. At least one representative from each least one IV in the field.) Nursing assistants
group should be included because all can be directed to take vitals and remove
employees are potential targets fro violence clothing. Foley catheter is necessary to
in the ED. closely monitor output.

In a multiple-trauma victim, which You are giving discharge instructions to a


assessment finding signals the most serious woman who has been treated for contusions
and life-threatening condition? and bruises sustained during an episode of
domestic violence. What is your priority
A. A deviated trachea intervention for this patient?
A. Transportation arrangements to a safe A. A patient with low-grade fever, headache,
house and myalgias for the past 72 hours.
B. Referral to a counselor B. A patient who is unable to bear weight on
C. Advice about contacting the police the left foot, with swelling and bruising
D. Follow-up appointment for injuries following a running accident.
A. Safety is a priority for this patient, and C. A patient with abdominal and chest pain
she should not return to a place where following a large, spicy meal.
violence could occur. The other options are D. A child with a one-inch bleeding
important for the long term management of laceration on the chin but otherwise well
this care. after falling while jumping on his bed.
Answer: C
A patient arrives at the emergency
department complaining of mid-sternal Emergency triage involves quick patient
chest pain. Which of the following nursing assessment to prioritize the need for further
action should take priority? evaluation and care. Patients with trauma,
chest pain, respiratory distress, or acute
A. A complete history with emphasis on neurological changes are always classified
preceding events. number one priority. Though the patient with
B. An electrocardiogram. chest pain presented in the question
C. Careful assessment of vital signs. recently ate a spicy meal and may be
D. Chest exam with auscultation. suffering from heartburn, he also may be
Answer: C having an acute myocardial infarction and
require urgent attention. The patient with
The priority nursing action for a patient fever, headache and muscle aches (classic
arriving at the ED in distress is always flu symptoms) should be classified as non-
assessment of vital signs. This indicates the urgent. The patient with the foot injury may
extent of physical compromise and provides have sustained a sprain or fracture, and the
a baseline by which to plan further limb should be x-rayed as soon as is
assessment and treatment. A thorough practical, but the damage is unlikely to
medical history, including onset of worsen if there is a delay. The child's chin
symptoms, will be necessary and it is likely laceration may need to be sutured but is
that an electrocardiogram will be performed also non-urgent.
as well, but these are not the first priority.
Similarly, chest exam with auscultation may A nurse employed in an emergency
offer useful information after vital signs are department is assigned to triage clients
assessed. arriving to the emergency room for
treatment on the evening shift. The nurse
Emergency department triage is an should assign highest priority to which of the
important nursing function. A nurse working following clients?
the evening shift is presented with four
patients at the same time. Which of the a) a client complaining of muscle aches, a
following patients should be assigned the headache, and malaise
highest priority?
b) a client who twisted her ankle when she Cardiogenic shock severely impairs the
fell while rollerblading pumping function of the heart muscle,
c) a client with a minor laceration on the causing diminished blood flow to the organs
index finger sustained while cutting an of the body. This results in diminished brain
eggplant function and confusion, as well as
d) a client with chest pain who states that he hypotension, tachycardia, and weak pulse.
just ate pizza that was made with a very Cardiogenic shock is a serious complication
spicy sauce of myocardial infarction with a high mortality
D rate
- In an emergency department, triage
involves brief client assessment to classify What is the treatment of choice to correct
clients according to their need for care and Hypovolemic Shock?
includes establishing priorities of care. The
type of illness or injury, the severity of the A. Have the patient drink a large bottle of
problem, and the resources available Gatorade.
govern the process. Clients with trauma, B. Replace fluids intravenously as quickly
chest pain, severe respiratory distress or as possible.
cardiac arrest, limb amputation, acute C. Administer a vasopressor
neurological deficits, and those who have D. All of the above.
sustained chemical splashes to the eyes are B. Replace fluids intravenously as quickly
classified as emergent and are the number as possible
1 priority. Clients with conditions such as a
simple fracture, asthma without respiratory What causes decreased BP in neurogenic
distress, fever, hypertension, abdominal shock?
pain, or a renal stone have urgent needs
and are classified as number 2 priority. A. Movement of fluid into the cells
Clients with conditions such as a minor B. Movement of fluid into the vasculature
laceration, sprain, or cold symptoms are C. Disrupted SNS communication
classified as nonurgent and are the number D. Polyuria
3 priority. C. Disrupted SNS communication

A patient arrives in the emergency What class of drug is typically used to treat
department with symptoms of myocardial neurogenic shock?
infarction, progressing to cardiogenic shock.
Which of the following symptoms should the A. Beta-blocker
nurse expect the patient to exhibit with B. Calcium-channel blocker
cardiogenic shock? C. Loop diuretic
D. Vasopressor
A. Hypertension. D. Vasporessor
B. Bradycardia.
C. Bounding pulse. The newly admitted client has burns on both
D. Confusion. legs. The burned areas appear white and
D. Confusion leather-like. No blisters or bleeding are
present, and the client states that he or she
has little pain. How should this injury be C. The danger of an overdose during fluid
categorized? remobilization is reduced.
D. The client delayed gastric emptying
A. Superficial C. The danger of an overdose during fluid
B. Partial-thickness superficial remobilization is reduced
C. Partial-thickness deep
D. Full thickness Although providing some pain relief has a
D. Full thickness high priority, and giving the drug by the IV
route instead of IM, SC, or orally does
The characteristics of the wound meet the increase the rate of effect, the most
criteria for a full-thickness injury (color that important reason is to prevent an overdose
is black, brown, yellow, white or red; no from accumulation of drug in the interstitial
blisters; pain minimal; outer layer firm and space during the fluid shift of the emergent
inelastic). phase. When edema is present, cumulative
doses are rapidly absorbed when the fluid
The newly admitted client has a large shift is resolving. This delayed absorption
burned area on the right arm. The burned can result in lethal blood levels of
area appears red, has blisters, and is very analgesics.
painful. How should this injury be
categorized? Which vitamin deficiency is most likely to be
a long-term consequence of a full-thickness
A. Superficial burn injury?
B. Partial-thickness superficial
C. Partial-thickness deep A. Vitamin A
D. Full thickness B. Vitamin B
B. Partial-thickness superficial C. Vitamin C
D. Vitamin D
The characteristics of the wound meet the D. Vitamin D
criteria for a superficial partial thickness
injury (color that is pink or red; blisters; pain Skin exposed to sunlight activates vitamin
present and high). D. Partial-thickness burns reduce the
activation of vitamin D. Activation of vitamin
The burned client newly arrived from an D is lost completely in full thickness burns.
accident scene is prescribed to receive 4
mg of morphine sulfate by IV push. What is Which client factors should alert the nurse
the most important reason to administer the to potential increased complications with a
opioid analgesic to this client by the burn injury?
intravenous route?
A. The client is a 26-year-old male.
A. The medication will be effective more B. The client has had a burn injury in the
quickly than if given intramuscularly. past.
B. It is less likely to interfere with the client's C. The burned areas include the hands and
breathing and oxygenation. perineum.
D. The burn took place in an open field and C. During fluid remobilization
ignited the client's clothing.
C. The burned areas include the hands and Hypokalemia is most likely to occur during
perineum the fluid remobilization period as a result of
dilution, potassium movement back into the
Burns of the perineum increase the risk for cells, and increased potassium excreted
sepsis. Burns of the hands require special into the urine with the greatly increased
attention to ensure the best functional urine output.
outcome.
What clinical manifestation should alert the
The burned client is ordered to receive nurse to possible carbon monoxide
intravenous cimetidine, an H2 histamine poisoning in a client who experienced a
blocking agent, during the emergent phase. burn injury during a house fire?
When the client's family asks why this drug
is being given, what is the nurse's best A. Pulse oximetry reading of 80%
response? B. Expiratory stridor and nasal flaring
C. Cherry red color to the mucous
A. "To increase the urine output and prevent membranes
kidney damage." D. Presence of carbonaceous particles in
B. "To stimulate intestinal movement and the sputum
prevent abdominal bloating." C. Cherry red color to mucous membranes
C. "To decrease hydrochloric acid
production in the stomach and prevent The saturation of hemoglobin molecules
ulcers." with carbon monoxide and the subsequent
D. "To inhibit loss of fluid from the vasodilation induces a "cherry red" color of
circulatory system and prevent hypovolemic the mucous membranes in these clients.
shock." The other manifestations are associated
C. Decrease hydrochloric acid production with inhalation injury, but not specifically
carbon monoxide poisoning.
Ulcerative gastrointestinal disease may
develop within 24 hours after a severe burn What clinical manifestation indicates that an
as a result of increased hydrochloric acid escharotomy is needed on a circumferential
production and decreased mucosal barrier. extremity burn?
Cimetidine inhibits the production and
release of hydrochloric acid. A. The burn is full thickness rather than
partial thickness.
At what point after a burn injury should the B. The client is unable to fully pronate and
nurse be most alert for the complication of supinate the extremity.
hypokalemia? C. Capillary refill is slow in the digits and the
distal pulse is absent.
A. Immediately following the injury D. The client cannot distinguish the
B. During the fluid shift sensation of sharp versus dull in the
C. During fluid remobilization extremity.
D. During the late acute phase
C. Capillary refill is slow in the digits and the B. Crystalloids
distal pulse is absent C. Fresh-frozen plasma
D. Packed red blood cells
Circumferential eschar can act as a B. Crystalloids
tourniquet when edema forms from the fluid
shift, increasing tissue pressure and Although not universally true, most fluid
preventing blood flow to the distal resuscitation for burn injuries starts with
extremities and increasing the risk for tissue crystalloid solutions, such as normal saline
necrosis. This problem is an emergency and Ringer's lactate. The burn client rarely
and, without intervention, can lead to loss of requires blood during the emergent phase
the distal limb. This problem can be reduced unless the burn is complicated by another
or corrected with an escharotomy. injury that involved hemorrhage. Colloids
and plasma are not generally used during
What additional laboratory test should be the fluid shift phase because these large
performed on any African American client particles pass through the leaky capillaries
who sustains a serious burn injury? into the interstitial fluid, where they increase
the osmotic pressure. Increased osmotic
A. Total protein pressure in the interstitial fluid can worsen
B. Tissue type antigens the capillary leak syndrome and make
C. Prostate specific antigen maintaining the circulating fluid volume even
D. Hemoglobin S electrophoresis more difficult.
D. Hemoglobin S electrophoresis
The client with a dressing covering the neck
Sickle cell disease and sickle cell trait are is experiencing some respiratory difficulty.
more common among African Americans. What is the nurse's best first action?
Although clients with sickle cell disease
usually know their status, the client with A. Administer oxygen.
sickle cell trait may not. The fluid, B. Loosen the dressing.
circulatory, and respiratory alterations that C. Notify the emergency team.
occur in the emergent phase of a burn injury D. Document the observation as the only
could result in decreased tissue perfusion action
that is sufficient to cause sickling of cells, B. Loosen dressing
even in a person who only has the trait.
Determining the client's sickle cell status by Respiratory difficulty can arise from external
checking the percentage of hemoglobin S is pressure. The first action in this situation
essential for any African American client would be to loosen the dressing and then
who has a burn injury. reassess the client's respiratory status.

Which type of fluid should the nurse expect The client who experienced an inhalation
to prepare and administer as fluid injury 6 hours ago has been wheezing.
resuscitation during the emergent phase of When the client is assessed, wheezes are
burn recovery? no longer heard. What is the nurse's best
action?
A. Colloids
A. Raise the head of the bed. C. Hemorrhage
B. Notify the emergency team. D. Carbon monoxide poisoning
C. Loosen the dressings on the chest. A. Fluid shift
D. Document the findings as the only action
B. Notify the emergency team Intense pain and carbon monoxide
poisoning increase blood pressure.
Clients with severe inhalation injuries may Hemorrhage is unusual in a burn injury. The
sustain such progressive obstruction that physiologic effect of histamine release in
they may lose effective movement of air. injured tissues is a loss of vascular volume
When this occurs, wheezing is no longer to the interstitial space, with a resulting
heard and neither are breath sounds. The decrease in blood pressure.
client requires the establishment of an
emergency airway and the swelling usually Twelve hours after the client was initially
precludes intubation. burned, bowel sounds are absent in all four
abdominal quadrants. What is the nurse's
Ten hours after the client with 50% burns is best action?
admitted, her blood glucose level is 90
mg/dL. What is the nurse's best action? A. Reposition the client onto the right side.
B. Document the finding as the only action.
A. Notify the emergency team. C. Notify the emergency team.
B. Document the finding as the only action. D. Increase the IV flow rate
C. Ask the client if anyone in her family has B. Document
diabetes mellitus.
D. Slow the intravenous infusion of dextrose Decreased or absent peristalsis is an
5% in Ringer's lactate expected response during the emergent
B. Document phase of burn injury as a result of neural
and hormonal compensation to the stress of
Neural and hormonal compensation to the injury. No currently accepted intervention
stress of the burn injury in the emergent changes this response, and it is not the
phase increases liver glucose production highest priority of care at this time.
and release. An acute rise in the blood
glucose level is an expected client response Which clinical manifestation indicates that
and is helpful in the generation of energy the burned client is moving into the fluid
needed for the increased metabolism that remobilization phase of recovery?
accompanies this trauma.
A. Increased urine output, decreased urine
On admission to the emergency department specific gravity
the burned client's blood pressure is 90/60, B. Increased peripheral edema, decreased
with an apical pulse rate of 122. These blood pressure
findings are an expected result of what C. Decreased peripheral pulses, slow
thermal injury-related response? capillary refill
D. Decreased serum sodium level,
A. Fluid shift increased hematocrit
B. Intense pain
A. Increased urine and decreased urine responses could result. With such a rapid
specific gravity rise in the potassium level, the client is at
high risk for experiencing severe cardiac
The "fluid remobilization" phase improves dysrhythmias and death.
renal blood flow, increasing diuresis and
restoring fluid and electrolyte levels. The The client has experienced an electrical
increased water content of the urine injury, with the entrance site on the left hand
reduces its specific gravity. and the exit site on the left foot. What are
the priority assessment data to obtain from
What is the priority nursing diagnosis during this client on admission?
the first 24 hours for a client with full-
thickness chemical burns on the anterior A. Airway patency
neck, chest, and all surfaces of the left arm? B. Heart rate and rhythm
C. Orientation to time, place, and person
A. Risk for Ineffective Breathing Pattern D. Current range of motion in all extremities
B. Decreased Tissue Perfusion B. Heart rate and rhythm
C. Risk for Disuse Syndrome The airway is not at any particular risk with
D. Disturbed Body Image this injury. Electric current travels through
C. Risk for disuse syndrome the body from the entrance site to the exit
site and can seriously damage all tissues
During the emergent phase, fluid shifts into between the two sites. Early cardiac
interstitial tissue in burned areas. When the damage from electrical injury includes
burn is circumferential on an extremity, the irregular heart rate, rhythm, and ECG
swelling can compress blood vessels to changes.
such an extent that circulation is impaired
distal to the injury, necessitating the In assessing the client's potential for an
intervention of an escharotomy. Chemical inhalation injury as a result of a flame burn,
burns do not cause inhalation injury what is the most important question to ask
the client on admission?
All of the following laboratory test results on
a burned client's blood are present during A. "Are you a smoker?"
the emergent phase. Which result should B. "When was your last chest x-ray?"
the nurse report to the physician C. "Have you ever had asthma or any other
immediately? lung problem?"
D. "In what exact place or space were you
A. Serum sodium elevated to 131 mmol/L when you were burned?"
(mEq/L) D. Place and space
B. Serum potassium 7.5 mmol/L (mEq/L)
C. Arterial pH is 7.32 The risk for inhalation injury is greatest
D. Hematocrit is 52% when flame burns occur indoors in small,
B. Potassium 7.5 poorly ventilated rooms. although smoking
All these findings are abnormal; however, increases the risk for some problems, it
only the serum potassium level is changed does not predispose the client for an
to the degree that serious, life-threatening inhalation injury.
C. Myocardial infarction 1 year ago
Which information obtained by assessment D. Kidney stones within the last 6 month
ensures that the client's respiratory efforts C. MI 1 year ago
are currently adequate?
It is likely the client has a diminished cardiac
A. The client is able to talk. output as a result of the old MI and would
B The client is alert and oriented. be at greater risk for the development of
C. The client's oxygen saturation is 97%. congestive heart failure and pulmonary
D. The client's chest movements are edema during fluid resuscitation
uninhibited
C. oxygen saturation is 97% The burned client on admission is drooling
and having difficulty swallowing. What is the
Clients may have ineffective respiratory nurse's best first action?
efforts and gas exchange even though they
are able to talk, have good respiratory A. Assess level of consciousness and
movement, and are alert. The best indicator pupillary reactions.
for respiratory effectiveness is the B. Ask the client at what time food or liquid
maintenance of oxygen saturation within the was last consumed
normal range. C. Auscultate breath sounds over the
trachea and mainstem bronchi
The burned client's family ask at what point D. Measure abdominal girth and auscultate
the client will no longer be at increased risk bowel sounds in all four quadrants
for infection. What is the nurse's best C. Auscultate breath sounds
response?
Difficulty swallowing and drooling are
A. "When fluid remobilization has started." indications of oropharyngeal edema and
B. "When the burn wounds are closed." can precede pulmonary failure. The client's
C. "When IV fluids are discontinued." airway is in severe jeopardy and intubation
D. "When body weight is normal." is highly likely to be needed shortly.
B. When burn wounds are closed
Intact skin is a major barrier to infection and Which intervention is most important for the
other disruptions in homeostasis. No matter nurse to use to prevent infection by cross-
how much time has passed since the burn contamination in the client who has open
injury, the client remains at great risk for burn wounds?
infection as long as any area of skin is
open. A. Handwashing on entering the client's
room
The burned client relates the following B. Encouraging the client to cough and
history of previous health problems. Which deep breathe
one should alert the nurse to the need for C. Administering the prescribed tetanus
alteration of the fluid resuscitation plan? toxoid vaccine
D. Changing gloves between cleansing
A. Seasonal asthma different burn areas
B. Hepatitis B 10 years ago A. Handwashing upon entering room
D. Hip at zero flexion with leg flat
Cross-contamination occurs when
microorganisms from another person or the Maximum function for ambulation occurs
environment are transferred to the client. when the hip and leg are maintained at full
Although all the interventions listed above extension with neutral rotation. Although the
can help reduce the risk for infection, only client does not have to spend 24 hours at a
hand washing can prevent cross time in this position, he or she should be in
contamination. this position (in bed or standing) more of the
time than with the hip in any degree of
The client has a deep partial-thickness flexion
injury to the posterior neck. Which
intervention is most important to use during During the acute phase, the nurse applied
the acute phase to prevent contractures gentamicin sulfate (topical antibiotic) to the
associated with this injury? burn before dressing the wound. The client
has all the following manifestations. Which
A. Place a towel roll under the client's neck manifestation indicates that the client is
or shoulder. having an adverse reaction to this topical
B. Keep the client in a supine position agent?
without the use of pillows.
C. Have the client turn the head from side to A. Increased wound pain 30 to 40 minutes
side 90 degrees every hour while awake. after drug application
D. Keep the client in a semi-Fowler's B. Presence of small, pale pink bumps in
position and actively raise the arms above the wound beds
the head every hour while awake. C. Decreased white blood cell count
C. Turn head side to side D. Increased serum creatinine level
D. Increased creatinine level
The function that would be disrupted by a
contracture to the posterior neck is flexion. Gentamicin does not stimulate pain in the
Moving the head from side to side prevents wound. The small, pale pink bumps in the
such a loss of flexion. wound bed are areas of re-epithelialization
and not an adverse reaction. Gentamicin is
The client has severe burns around the right nephrotoxic and sufficient amounts can be
hip. Which position is most important to be absorbed through burn wounds to affect
emphasized by the nurse that the client kidney function. Any client receiving
maintain to retain maximum function of this gentamicin by any route should have kidney
joint? function monitored

A. Hip maintained in 30-degree flexion, no The client, who is 2 weeks postburn with a
knee flexion 40% deep partial-thickness injury, still has
B. Hip flexed 90 degrees and knee flexed open wounds. On taking the morning vital
90 degrees signs, the client is found to have a below-
C. Hip, knee, and ankle all at maximum normal temperature, is hypotensive, and
flexion has diarrhea. What is the nurse's best
D. Hip at zero flexion with leg flat action?
A. Nothing, because the findings are normal A. When all full-thickness areas have been
for clients during the acute phase of closed with skin grafts
recovery. B. When the client's temperature has
B. Increase the temperature in the room and remained normal for 24 hours
increase the IV infusion rate. C. As soon as possible after wound
C. Assess the client's airway and oxygen debridement is complete
saturation. D. As soon as possible after resolution of
D. Notify the burn emergency team. the fluid shift
D. Notify burn emergency team D. As soon as possible after fluid shift

These findings are associated with systemic Regular, progressive ambulation is initiated
gram-negative infection and sepsis. This is for all burn clients who do not have
a medical emergency and requires prompt contraindications concomitant injuries as
attention soon as the fluid shift resolves. Clients can
be ambulated with extensive dressings,
Which intervention is most important to use open wounds, and nearly any type of
to prevent infection by autocontamination in attached lines, tubing, and other equipment.
the burned client during the acute phase of
recovery? What statement by the client indicates the
need for further discussion regarding the
A. Changing gloves between wound care on outcome of skin grafting (allografting)
different parts of the client's body. procedures?
B. Avoiding sharing equipment such as
blood pressure cuffs between clients. A. "For the first few days after surgery, the
C. Using the closed method of burn wound donor sites will be painful."
management. B. "Because the graft is my own skin, there
D. Using proper and consistent is no chance it won't 'take'."
handwashing. C. "I will have some scarring in the area
A. Changing gloves between wound care on when the skin is removed for grafting."
different parts of the body D. "Once all grafting is completed, my risk
for infection is the same as it was before I
Autocontamination is the transfer of was burned."
microorganisms from one area to another B. Because it is my own skin
area of the same client's body, causing
infection of a previously uninfected area. Factors other than tissue type, such as
Although all techniques listed can help circulation and infection, influence whether
reduce the risk for infection, only changing and how well a graft "takes." The client
gloves between carrying out wound care on should be prepared for the possibility that
difference parts of the client's body can not all grafting procedures will be
prevent autocontamination. successful.

When should ambulation be initiated in the


client who has sustained a major burn?
Which statement by the client indicates protection of sensitive newly healed skin
correct understanding of rehabilitation after and grafts from sun exposure, this is not the
burn injury? purpose of wearing the mask. The pressure
garment will not change the angle of ear
A. "I will never be fully recovered from the attachment to the head.
burn."
B. "I am considered fully recovered when all What is the priority nursing diagnosis for a
the wounds are closed." client in the rehabilitative phase of recovery
C. "I will be fully recovered when I am able from a burn injury?
to perform all the activities I did before my
injury." A. Acute Pain
D. "I will be fully recovered when I achieve B. Impaired Adjustment
the highest possible level of functioning that C. Deficient Diversional Activity
I can." D. Imbalanced Nutrition: Less than Body
D. I will achieve highest level of functioning Requirements
B. Impaired adjustment
Although a return to pre burn functional
levels is rarely possible, burned clients are Recovery from a burn injury requires a lot of
considered fully recovered or rehabilitated work on the part of the client and significant
when they have achieved their highest others. Seldom is the client restored to the
possible level of physical, social, and preburn level of functioning. Adjustments to
emotional functioning. changes in appearance, family structure,
employment opportunities, role, and
Which statement made by the client with functional limitations are only a few of the
facial burns who has been prescribed to numerous life-changing alterations that
wear a facial mask pressure garment must be made or overcome by the client. By
indicates correct understanding of the the rehabilitation phase, acute pain from the
purpose of this treatment? injury or its treatment is no longer a
problem.
A. "After this treatment, my ears will not
stick out." Nurse Faith should recognize that fluid shift
B. "The mask will help protect my skin from in an client with burn injury results from
sun damage." increase in the:
C. "Using this mask will prevent scars from
being permanent." A. Total volume of circulating whole blood
D. "My facial scars should be less severe B. Total volume of intravascular plasma
with the use of this mask." C. Permeability of capillary walls
D. Facial scars should be less severe D. Permeability of kidney tubules
C. Permeability of capillary walls
The purpose of wearing the pressure
garment over burn injuries for up to 1 year is In burn, the capillaries and small vessels
to prevent hypertrophic scarring and dilate, and cell damage cause the release of
contractures from forming. Scars will still be a histamine-like substance. The substance
present. Although the mask does provide causes the capillary walls to become more
permeable and significant quantities of fluid A. Begin intravenous fluids
are lost. B. Check the pulses with a Doppler device
C. Obtain a complete blood count (CBC)
Louie, with burns over 35% of the body, D. Obtain an electrocardiogram (ECG)
complains of chilling. In promoting the A. Begin IV fluids
client's comfort, the nurse should:
Hypovolemic shock is a common cause of
A. Maintain room humidity below 40% death in the emergent phase of clients with
B. Place top sheet on the client serious injuries. Fluids can treat this
C. Limit the occurrence of drafts problem. An ECG and CBC will be taken to
D. Keep room temperature at 80 degrees ascertain if a cardiac or bleeding problem is
C. Limit drafts causing these vital signs. However these
are not actions that the nurse would take
A Client with burns is very sensitive to immediately. Checking pulses would
temperature changes because heat is loss indicate perfusion to the periphery but this is
in the burn areas. not an immediate nursing action.

A client who has had a full-thickness burn is A client who was burned has crackles and a
being discharged from the hospital. Which respiratory rate of 40/min, and is coughing
information is most important for the nurse up blood-tinged sputum. What action will the
to provide prior to discharge? nurse take first?

A. How to maintain home smoke detectors A. Administer digoxin


B. Joining a community reintegration B. Perform chest physiotherapy
program C. Monitor urine output
C. Learning to perform dressing changes D. Place the client in an upright position
D. Options available for scar removal D. Upright position
C. Dressing changes
Pulmonary edema can result from fluid
Critical for the goal of progression toward resuscitation given for burn treatment. This
independence for the client is teaching can occur even in a young healthy person.
clients and family members to perform care Placing the client in the upright position can
tasks such as dressing changes. All the relieve the lung congestion immediately
other distractors are important in the before other measures can be carried out.
rehabilitation stage. However, dressing Digoxin may be given later to increase
changes have priority. cardiac contractility to prevent backup of
fluid into the lungs. Chest physiotherapy will
A client who is admitted after a thermal burn not get rid of fluid. Monitoring urine output is
injury has the following vital signs: blood important. However it is not an immediate
pressure, 70/40; heart rate, 140 beats/min; intervention.
respiratory rate, 25/min. He is pale in color
and it is difficult to find pedal pulses. Which On assessment, the nurse notes that the
action will the nurse take first? client has burns inside the mouth and is
wheezing. Several hours later, the wheezing
is no longer heard. What is the nurse's next The client with facial burns asks the nurse if
action? he will ever look the same. Which response
is best for the nurse to provide?
A. Documenting the findings
B. Loosening any dressings on the chest A. "With reconstructive surgery, you can
C. Raising the head of the bed look the same."
D. Preparing for intubation B. "We can remove the scars with the use
D. Preparing for intubation of a pressure dressing."
C. "You will not look exactly the same."
Clients with severe inhalation injuries may D. "You shouldn't start worrying about your
sustain such progressive obstruction that appearance right now."
they may lose effective movement of air. C. You will not look exactly the same
When this occurs, wheezing is no longer
heard and neither are breath sounds. The Many clients have unrealistic expectations
client requires the establishment of an of reconstructive surgery and envision an
emergency airway. The swelling usually appearance identical or equal in quality to
precludes intubation. the preburn state. Pressure dressings
prevent further scarring. They cannot
The client with a full-thickness burn is being remove scars. The client and family should
discharged to home after a month in the be taught the expected cosmetic outcomes.
hospital. His wounds are minimally opened
and he will be receiving home care. Which The client with open burn wounds begins to
nursing diagnosis has the highest priority? have diarrhea. The client is found to have a
below-normal temperature, with a white
A. Acute Pain blood cell count of 4000/mm3. Which is the
B. Deficient Diversional Activity nurse's best action?
C. Impaired Adjustment
D. Imbalanced Nutrition: Less than Body A. Continuing to monitor the client
Requirements B. Increasing the temperature in the room
C. Impaired adjustment C. Increasing the rate of the intravenous
fluids
Recovery from a burn injury requires a lot of D. Preparing to do a workup for sepsis
work on the part of the client and significant D. Sepsis
others. The client is seldom restored to his
or her preburn level of functioning. These findings are associated with systemic
Adjustments to changes in appearance, gram-negative infection and sepsis. To
family structure, employment opportunities, verify that sepsis is occurring, cultures of
role, and functional limitations are only a the wound and blood must be taken to
few of the numerous life-changing determine the appropriate antibiotic to be
alterations that must be made or overcome started. Continuing just to monitor the
by the client. By the rehabilitation phase, situation can lead to septic shock.
acute pain from the injury or its treatment is Increasing the temperature in the room may
no longer a problem. make the client more comfortable, but the
priority is finding out if the client has sepsis
and treating it before it becomes a shock A. Patient in room 2101 with an electrical
situation. Increasing the rate of intravenous burn on torso.
fluids may be done to replace fluid losses B. Patient in room 2106 with a radiation
with diarrhea, but is not the priority action. burn on the abdomen.
C. Patient in room 2103 with a thermal burn
The nurse provides wound care for a client to peritoneal area.
48 hours after a burn injury. To achieve the D. Patient in room 2101 with a chemical
desired outcome of the procedure, which burn to face.
nursing action will be carried out first? A. electrical burn

A. Applies silver sulfadiazine (Silvadene) Electrical burns are caused by heat


ointment generated by electrical current which is
B. Covers the area with an elastic wrap transferred through the body. This current
C. Places a synthetic dressing over the area burns the skin but also affects internal
D. Removes loose nonviable tissue tissue as well.
D. Remove nonviable tissue
A preceptor is observing a nursing student
All steps are part of the nonsurgical wound provide care to a patient with major burns to
care for clients with burn injuries. The first the face and head. What nursing
step in this process is removing exudates intervention does the student perform
and necrotic tissue. correctly?

Which patient is at risk for compartment A. Elevates the head of the bead at 30'.
syndrome due to a burn? B. Uses gloves and face mask when
providing care.
A. A 25 year old with circumferential burn of C. Places the patient in trendelenburg
the anterior and posterior left arm. position.
B. A 7 year old with a burn of the left and D. Assist the patient with eating food tray
right ear. A. elevate HOB
C. A 55 year old with an electrical burn on
the neck. Due to edema and respiratory issues patient
D. A 15 year old with a chemical burn to the with facial burns should have the HOB at
right foot 30'. In additon, strict isolation protocol is
A. anterior and posterior arm implemented because they patient is at high
risk for infection ( gloves and facial mask
Circumferential burns of the extremities are not sufficent enough)
produce a tourniquet like effect and leads to
vascular problems. A patient with 55% burns is groaing out in
pain and rates pain 10 on 1-10 scale. You
As a nurse working on a burn unit, which of have PRN orders for the following
your patients are at high risk for internal medications. What is the best option for this
tissue damage? patient?

A. IV Morphine
B. Oral Lortab liquid suspension C. It is performed if tissue perfusion does
C. IM Demerol NOT return after a fasciotomy.
D. Subcutaneous Demerol D. None of the options are correct.
A. IV morphine A

IV route is the best option when a patient Escharotomy are performed at the beside
has burns. If a medication is given IM or without anesthia because the nerves are
subq, hypovolemia may disrupt absorption. already damaged. It is first performed when
In addition, oral route should be avoid due a patient has a circumferential burn and if
to potential GI dysfunction. tissue perfusion fails to return a fasciotomy
is performed in the operating room.
A patient is being discharge after having
autografting. What would you include in A patient with a spinal cord injury (SCI)
your discharge education? complains about a severe throbbing
headache that suddenly started a short time
A. Keep the site free from pressure and ago. Assessment of the patient reveals
keep the site lubricated. increased blood pressure (168/94) and
B. Encourage for the site to be exposed to decreased heart rate (48/minute),
sunlight to promoted melanin production. diaphoresis, and flushing of the face and
C. Avoid using splints or any type of support neck. What action should you take first?
garment.
D. Encourage weight-bearing exercise A. Administer the ordered acetaminophen
every 4 to 6 hours. (Tylenol).
A. Keep site from pressure and keep B. Check the Foley tubing for kinks or
lubricated obstruction.
C. Adjust the temperature in the patient's
The patient should avoid the sunlight due to room.
increase risk of sunburn to delicate skin. In D. Notify the physician about the change in
addition, the patient should avoid weight- status
bearing activites to prevent damage to the B. Foley
newly grafted skin. It is best to encourage
splints and support garments to protect the These signs and symptoms are
skin during acitiviy. characteristic of autonomic dysreflexia, a
neurologic emergency that must be
A patient is undergoing a escharotomy. promptly treated to prevent a hypertensive
Which of the following is correct about the stroke. The cause of this syndrome is
procedure? noxious stimuli, most often a distended
bladder or constipation, so checking for
A. It is performed on circumferential burns poor catheter drainage, bladder distention,
and is usually performed at bedside without or fecal impaction is the first action that
anesthesia. should be taken. Adjusting the room
B. It is performed on radiation burns and temperature may be helpful, since too cool
requires general anesthesia. a temperature in the room may contribute to
the problem. Tylenol will not decrease the
autonomic dysreflexia that is causing the B. Vital signs
patient's headache. Notification of the
physician may be necessary if nursing The nursing assistant's training and
actions do not resolve symptoms. Focus: education include taking and recording
Prioritization patient's vital signs. The nursing assistant
may assist with turning and repositioning
A patient with a spinal cord injury at level the patient and may remind the patient to
C3-4 is being cared for in the ED. What is cough and deep breathe but does not teach
the priority assessment? the patient how to perform these actions.
Assessing and monitoring patients require
A. Determine the level at which the patient additional education and are appropriate to
has intact sensation. the scope of practice for professional
B. Assess the level at which the patient has nurses. Focus: Delegation/supervision
retained mobility.
C. Check blood pressure and pulse for You are helping the patient with an SCI to
signs of spinal shock. establish a bladder-retraining program.
D. Monitor respiratory effort and oxygen What strategies may stimulate the patient to
saturation level void? (Choose all that apply).
D. Respiratory A. Stroke the patient's inner thigh.
B. Pull on the patient's pubic hair.
The first priority for the patient with an SCI C. Initiate intermittent straight
is assessing respiratory patterns and catheterization.
ensuring an adequate airway. The patient D. Pour warm water over the perineum.
with a high cervical injury is at risk for E. Tap the bladder to stimulate detrusor
respiratory compromise because the spinal muscle
nerves (C3 - 5) innervate the phrenic nerve, A, B, D, E
which controls the diaphragm. The other
assessments are also necessary, but not as All of the strategies, except straight
high priority. Focus: Prioritization catheterization, may stimulate voiding in
patients with SCI. Intermittent bladder
You are pulled from the ED to the catheterization can be used to empty the
neurologic floor. Which action should you patient's bladder, but it will not stimulate
delegate to the nursing assistant when voiding.
providing nursing care for a patient with
SCI? The patient with a cervical SCI has been
placed in fixed skeletal traction with a halo
A. Assess patient's respiratory status every fixation device. When caring for this patient
4 hours. the nurse may delegate which action (s) to
B. Take patient's vital signs and record the LPN/LVN? (Choose all that apply).
every 4 hours. A. Check the patient's skin for pressure
C. Monitor nutritional status including calorie form device.
counts. B. Assess the patient's neurologic status for
D. Have patient turn, cough, and deep changes.
breathe every 3 hours
C. Observe the halo insertion sites for signs
of infection. a) pain level
D. Clean the halo insertion sites with b) mobility level
hydrogen peroxide c) respiratory status
A, C, D d) pupillary response
C. Respiratory status
Checking and observing for signs of
pressure or infection are within the scope of All of these assessments would be
practice of the LPN/LVN. The LPN/LVN also performed on a client with a suspected
has the appropriate skills for cleaning the spinal cord injury. However, respiratory
halo insertion sites with hydrogen peroxide. status is the priority.
Neurologic examination requires additional
education and skill appropriate to the When admitting a patient who has a history
professional RN of paraplegia as a result of a spinal cord
injury, the nurse will plan to
You are preparing a nursing care plan for
the patient with SCI including the nursing a. check the patient for urinary incontinence
diagnosis Impaired Physical Mobility and every 2 hours to maintain skin integrity.
Self-Care Deficit. The patient tells you, "I b. assist the patient to the toilet on a
don't know why we're doing all this. My life's scheduled basis to help ensure bladder
over." What additional nursing diagnosis emptying.
takes priority based on this statement? c. use intermittent catheterization on a
A. Risk for Injury related to altered mobility regular schedule to avoid the risk of
B. Imbalanced Nutrition, Less Than Body infection.
Requirements d. ask the patient about the usual urinary
C. Impaired Adjustment to Spinal Cord pattern and measures used for bladder
Injury control
D. Poor Body Image related to D. ask about usual urinary patterns
immobilization
C. Impaired Adjustment A patient with a neck fracture at the C5 level
is admitted to the intensive care unit (ICU)
The patient's statement indicates following initial treatment in the emergency
impairment of adjustment to the limitations room. During initial assessment of the
of the injury and indicates the need for patient, the nurse recognizes the presence
additional counseling, teaching, and of spinal shock on finding
support. The other three nursing diagnoses
may be appropriate to the patient with SCI, a. hypotension, bradycardia, and warm
but they are not related to the patient's extremities.
statement b. involuntary, spastic movements of the
arms and legs.
A nurse is performing an assessment on a c. the presence of hyperactive reflex activity
client who has a suspected spinal cord below the level of the injury.
injury. Which of the following is the priority d. flaccid paralysis and lack of sensation
nursing assessment? below the level of the injury
D. Flaccid paralysis and lack of sensation A. teaching on self-cath measures

A patient with a T1 spinal cord injury is A patient with a history of a T2 spinal cord
admitted to the intensive care unit (ICU). tells the nurse, "I feel awful today. My head
The nurse will teach the patient and family is throbbing, and I feel sick to my stomach."
that Which action should the nurse take first?

a. use of the shoulders will be preserved. a. Notify the patient's health care provider.
b. full function of the patient's arms will be b. Check the blood pressure (BP).
retained. c. Give the ordered antiemetic.
c. total loss of respiratory function may d. Assess for a fecal impaction.
occur temporarily. B. Check BP
d. elevations in heart rate are common with
this type of injury. Rationale: The BP should be assessed
B. Full function of the patients arms will be immediately in a patient with an injury at the
retained T6 level or higher who complains of a
headache to determine whether autonomic
The health care provider orders dysreflexia is causing the symptoms,
administration of IV methylprednisolone including hypertension. Notification of the
(Solu-Medrol) for the first 24 hours to a patient's health care provider is appropriate
patient who experienced a spinal cord injury after the BP is obtained. Administration of
at the T10 level 3 hours ago. When an antiemetic is indicated after autonomic
evaluating the effectiveness of the dysreflexia is ruled out as the cause of the
medication the nurse will assess nausea. The nurse may assess for a fecal
impaction, but this should be done after
a. blood pressure and heart rate. checking the BP and lidocaine jelly should
b. respiratory effort and O2 saturation. be used to prevent further increases in the
c. motor and sensory function of the legs. BP
d. bowel sounds and abdominal distension
C. Motor and sensory function of the legs The nurse discusses long-range goals with
a patient with a C6 spinal cord injury. An
A patient with a paraplegia resulting from a appropriate patient outcome is
T10 spinal cord injury has a neurogenic a. transfers independently to a wheelchair.
reflex bladder. When the nurse develops a b. drives a car with powered hand controls.
plan of care for this problem, which nursing c. turns and repositions self independently
action will be most appropriate? when in bed.
d. pushes a manual wheelchair on flat,
a. Teaching the patient how to self- smooth surfaces
catheterize D. Pushes manual wheelchair
b. Assisting the patient to the toilet q2-3hr
c. Use of the Credé method to empty the A 26-year-old patient with a C8 spinal cord
bladder injury tells the nurse, "My wife and I have
d. Catheterization for residual urine after always had a very active sex life, and I am
voiding worried that she may leave me if I cannot
function sexually." The most appropriate
response by the nurse to the patient's a. Continuous cardiac monitoring for
comment is to bradycardia
b. Administration of methylprednisolone
a. advise the patient to talk to his wife to (Solu-Medrol) infusion
determine how she feels about his sexual c. Assessment of respiratory rate and depth
function. d. Application of pneumatic compression
b. tell the patient that sildenafil (Viagra) devices to both legs
helps to decrease erectile dysfunction in C. Assessment of respiratory rate and depth
patients with spinal cord injury.
c. inform the patient that most patients with My BP is normal, HR >100, RR >20, skin is
upper motor neuron injuries have reflex cold and clammy, slight decrease to urinary
erections. output, confused, and respiratory alkalosis.
d. suggest that the patient and his wife work What stage of shock
with a nurse specially trained in sexual Pre-Shock
counseling
D. Sexual counseling My BP systolic BP <80-90, HR 100-150,
rapid shallow respirations with crackles
A 25-year-old patient has returned home heard on auscultation, skin has become
following extensive rehabilitation for a C8 mottled with petechiae, urine output has
spinal cord injury. The home care nurse become severely decreased, I'm lethargic
visits and notices that the patient's spouse and have gone in to metabolic acidosis.
and parents are performing many of the What stage of shock
activities of daily living (ADLs) that the Shock
patient had been managing during
rehabilitation. The most appropriate action I'm no longer able to breathe on my own,
by the nurse at this time is to my HR is erractic or I've gone in to asystole,
my skin has become jaundiced, I'm anuric,
a. tell the family members that the patient become unresponsive and now have
can perform ADLs independently. profound acidosis. What stage of shock
b. remind the patient about the importance End-Organ
of independence in daily activities.
c. recognize that it is important for the The nurse is triaging four clients injured in a
patient's family to be involved in the train derailment. Which client should receive
patient's care and support their activities. priority treatment?
d. develop a plan to increase the patient's
independence in consultation with the with A 42-year-old with dyspnea and chest
the patient, spouse, and parents asymmetry
D. Develop POC to increase independence b 17-year-old with a fractured arm
C 4-year-old with facial lacerations
When caring for a patient who was admitted D.A 30-year-old with blunt abdominal
24 hours previously with a C5 spinal cord trauma
injury, which nursing action has the highest Answer A is correct. Following the ABCDs
priority? of basic emergency care, the client
withdyspnea and asymmetrical chest should An unresponsive client is admitted to the
be cared for first because thesesymptoms emergency room with a history of
are associated with flail chest. Answer D is diabetes mellitus. The client's skin is cold
incorrect because he shouldbe cared for and clammy, and the blood pressure
second because of the likelihood of organ reading is 82/56. The first step in
damage and bleeding.Answer B is incorrect emergency
because he should be cared for after the treatment of the client's symptoms
client withabdominal trauma. Answer C is would be:
incorrect because he should receive care
lastbecause his injuries are less severe A.Checking the client's blood sugar
B. Administering intravenous dextrose
Direct pressure to a deep laceration on the C. Intubation and ventilator support
client's lower leg has failed to stop the D. Administering regular insulin
bleeding. The nurse's next action should be Answer A is correct. The client has
to: symptoms of insulin shock and the first step
is
A. Place a tourniquet proximal to the to check the client's blood sugar. If
laceration. indicated, the client should be treated with
B. Elevate the leg above the level of the intravenous dextrose. Answer B is wrong
heart. because it is not the first step the nurse
C. Cover the laceration and apply an ice should take. Answer C is wrong because it
compress does not apply to the client's
D. Apply pressure to the femoral artery symptoms. Answer D is wrong because it
Answer B is correct. If bleeding does not would be used for diabetic ketoacidosis,not
subside with direct pressure, the insulin shock.
nurseshould elevate the extremity above the
level of the heart. Answers A and D aredone A client with a history of severe depression
only if other measures are ineffective, so has been brought to the emergency room
they are incorrect. Answer C wouldslow the with an overdose of barbiturates. The nurse
bleeding but will not stop it, so it's incorrect should pay careful attention to
the client's:
The nurse is preparing to administer
Lactate Ringer's to a client with A. Urinary output
hypovolemic shock. Which intervention is B. Respirations
important in helping to stabilize the client's C. Temperature
condition? D. Verbal responsiveness
Answer B is correct. Barbiturate overdose
A. Warming the intravenous fluids results in central nervous system
B. Determining whether the client can take depression, which leads to respiratory
oral fluids failure. Answers A and C are important to
C. Checking for the strength of pedal pulses the client's overall condition but are not
D. Obtaining the specific gravity of the urine specific to the question, so they are
A. Warming the intravenous fluids incorrect. The use of barbiturates results in
slow, slurred speech, so answer D is A 56-year-old patient presents in triage with
expected, and therefore incorrect. left-sided chest pain, diaphoresis, and
dizziness. This patient should be prioritized
A client is to receive antivenin following a into which category?
snake bite. Before administering the a. High urgent
antivenin, the nurse should give priority to: b. Urgent
A. Administering a local anesthetic c. Non-urgent
B. Checking for an allergic response d. Emergent
C. Administering an anxiolytic Answer: D
D.Withholding fluids for 6 - 8 hours -
Answer B is correct. The nurse should Chest pain is considered an emergent
perform the skin or eye test before priority, which is defined as potentially life-
administering antivenin. Answers A and D threatening. Patients with urgent priority
are unnecessary and therefore incorrect. need treatment within 2 hours of triage (e.g.
Answer C would help calm the client but is kidney stones).Non-urgent conditions can
not a priority before giving the antivenin, wait for hours or even days. (High urgent is
making it incorrect. not commonly used; however, in5-tier triage
systems, High urgent patients fall between
You are working in the triage area of an ED, emergent and urgent in terms of the
and four patients approach the triage desk timelapsing prior to treatment).
at the sametime. List the order in which you
will assess these patients. The emergency service team brings a client
to the emergency department. The client
a. An ambulatory, dazed 25-year-old male was found lying in an alley near a Dumpster
with a bandaged head wound by a policeman, who reports that the client
b. An irritable infant with a fever, petechiae, is a homeless victim. An assessment is
and nuchal rigidity performed, and the client is suspected of
c. A 35-year-old jogger with a twisted ankle, having frostbite of the hands. Which of the
having pedal pulse and no deformity following findings would the nurse note in
d. A 50-year-old female with moderate this condition?
abdominal pain and occasional vomiting
Answer: B, A, D, C a) a white appearance to the skin that is
- insensitive to touch
An irritable infant with fever and petechiae b) a pink edematous hand
should be further assessed for other c) black fingertips surrounded by an
meningeal signs. The patient with the head erythematous rash
wound needs additional history and d) resd skin with edema in the nail beds
assessment for intracranial pressure. The A
patient with moderate abdominal pain is - Assessment findings in frostbite include a
uncomfortable, but not unstable at this white or blue appearance, and the skin will
point. For the ankle injury, medical be hard, cold, and insensitive to touch. As
evaluation can be delayed 24-48 hours if thawing occurs, the skin becomes flushed,
necessary blisters or blebs develop, or tissue edema
appears. Gangrene develops in 9 to 15 following is the most important instruction
days the nurse can give the parent?

An emergency department nurse prepares A: This too shall pass.


to treat a client who has frostbite of the toes B: Take the child immediately to the ER
from prolonged exposure during an ice- C: Contact the Poison Control Center
fishing trip. Which of the following would the quickly
nurse anticipate to be prescribed for this D: Give the child syrup of ipecac
condition? (C) The poison control center will have an
exact plan of action for this child.
a) rapid rewarming of the toes in hot water
b) rapid and continuous rewarming of the A mother arrives at the emergency
toes in a warm water bath until flushing of department with her child, stating that she
the skin occurs just found the child sitting on the floor next
c) rapid and continuous alternating cold and to an empty bottle of aspirin. On
hot soaks of the toes assessment, the nurse notes that the child
d) rapid rewarming of the toes by soaking in is drowsy but conscious. The nurse
cold water for 45 minutes anticipates that the physician will prescribe
B which of the following?
- Acute frostbite is ideally treated with rapid
and continuous rewarming of the tissue in a a) ipecac syrup
water bath for 15 to 20 minutes or until b) activated charcoal
flushing of the skin occurs. Slow thawing or c) magnesium citrate
interrupted periods of warmth are avoided d) magnesium sulfate
because this can contribute to increased B
cellular damage. Thawing can cause - Whereas ipecac is administered to induce
considerable pain, and the nurse would vomiting in certain poisoning situations, it is
administer analgesics as prescribed. not recommended as the initial treatment in
the hospital setting for ingestion of
Which of the following strategies is NOT salicylates. This is because ipecac does not
effective for prevention of Lyme disease? totally remove the poison from the child's
system. In this situation, the child is
A. Insect repellant on the skin and clothes conscious and the ingested substance
when in a Lyme endemic area. (aspirin) would not damage the esophagus
B. Long sleeved shirts and long pants. or lungs from vomiting. However, activated
C. Prophylactic antibiotic therapy prior to charcoal would be prescribed as an antidote
anticipated exposure to ticks. in this poisoning situation, because its
D. Careful examination of skin and hair for action is to absorb ingested toxic
ticks following anticipated exposure. substances and thus decrease absorption.
C. Prophylactic antibiotic Options C and D are unrelated to treatment
for this occurrence.
A parent calls the pediatric clinic and is
frantic about the bottle of cleaning fluid her The nurse is caring for a client admitted with
child drank 20 minutes. Which of the spinal cord injury. The nurse minimizes the
risk of compounding the injury most A client comes into the ER after hitting his
effectively by: head in an MVA. He's alert and oriented.
Which of the following nursing interventions
A. Keeping the client on a stretcher should be done first?
B. Logrolling the client on a firm mattress
C. Logrolling the client on a soft mattress A. Assess full ROM to determine extent of
D. Placing the client on a Stryker frame injuries
D. Stryker frame B. Call for an immediate chest x-ray
C. Immobilize the client's head and neck
Spinal immobilization is necessary after D. Open the airway with the head-tilt chin-lift
spinal cord injury to prevent further damage maneuver
and insult to the spinal cord. Whenever C. Immobilize head and neck
possible, the client is placed on a Stryker
frame, which allows the nurse to turn the All clients with a head injury are treated as if
client to prevent complications of immobility, a cervical spine injury is present until x-rays
while maintaining alignment of the spine. If confirm their absence. ROM would be
a Stryker frame is not available, a firm contraindicated at this time. There is no
mattress with a bed board should be used. indication that the client needs a chest x-
ray. The airway doesn't need to be opened
A 20-year-old client who fell approximately since the client appears alert and not in
30' is unresponsive and breathless. A respiratory distress. In addition, the head-tilt
cervical spine injury is suspected. How chin-lift maneuver wouldn't be used until the
should the first-responder open the client's cervical spine injury is ruled out.
airway for rescue breathing?
Which of the following interventions
A. By inserting a nasopharyngeal airway describes an appropriate bladder program
B. By inserting a oropharyngeal airway for a client in rehabilitation for spinal cord
C. By performing a jaw-thrust maneuver injury?
D. By performing the head-tilt, chin-lift
maneuver A. Insert an indwelling urinary catheter to
C. Jaw-thrust maneuver straight drainage
B. Schedule intermittent catheterization
If the client has a suspected cervical spine every 2 to 4 hours
injury, a jaw-thrust maneuver should be C. Perform a straight catheterization every 8
used to open the airway. If the tongue or hours while awake
relaxed throat muscles are obstructing the D. Perform Crede's maneuver to the lower
airway, a nasopharyngeal or oropharyngeal abdomen before the client voids
airway can be inserted; however, the client B. Intermittent cath Q2-4H
must have spontaneous respirations when
the airway is open. The head-tilt, chin-lift Intermittent catherization should begin every
maneuver requires neck hyperextension, 2 to 4 hours early in the treatment. When
which can worsen the cervical spine injury. residual volume is less than 400 ml, the
schedule may advance to every 4 to 6
hours. Indwelling catheters may predispose
the client to infection and are removed as B. Limiting bladder cath to once Q12H
soon as possible. Crede's maneuver is not
used on people with spinal cord injury. The most frequent cause of autonomic
dysreflexia is a distended bladder. Straight
When discharging a client from the ER after catherization should be done every 4 to 6
a head trauma, the nurse teaches the hours, and Foley catheters should be
guardian to observe for a lucid interval. checked frequently to prevent kinks in the
Which of the following statements best tubing. Constipation and fecal impaction are
described a lucid interval? other causes, so maintaining bowel
regularity is important. Other causes include
A. An interval when the client's speech is stimulation of the skin from tactile, thermal,
garbled. or painful stimuli. The nurse administers
B. An interval when the client is alert but care to minimize risk in these areas.
can't recall recent events.
C. An interval when the client is oriented but A nurse assesses a client who has episodes
then becomes somnolent. of autonomic dysreflexia. Which of the
D. An interval when the client has a following conditions can cause autonomic
"warning" symptom, such as an odor or dysreflexia?
visual disturbance.
C. A. Headache
B. Lumbar spinal cord injury
A lucid interval is described as a brief period C. Neurogenic shock
of unconsciousness followed by alertness; D. Noxious stimuli
after several hours, the client again loses D. Noxious Stimuli
consciousness. Garbled speech is known
as dysarthria. An interval in which the client Noxious stimuli, such as a full bladder, fecal
is alert but can't recall recent events is impaction, or a decub ulcer, may cause
known as amnesia. Warning symptoms or autonomic dysreflexia. A headache is a
auras typically occur before seizures. symptom of autonomic dysreflexia, not a
cause. Autonomic dysreflexia is most
A client with a spinal cord injury is prone to commonly seen with injuries at T10 or
experiencing autonomic dysreflexia. The above. Neurogenic shock isn't a cause of
nurse would avoid which of the following dysreflexia.
measures to minimize the risk of
recurrence? After falling 20', a 36-year-old man sustains
a C6 fracture with spinal cord transaction.
A. Strict adherence to a bowel retraining Which other findings should the nurse
program expect?
B. Limiting bladder catheterization to once
every 12 hours A. Quadriplegia with gross arm movement
C. Keeping the linen wrinkle-free under the and diaphragmatic breathing
client B. Quadriplegia and loss of respiratory
D. Preventing unnecessary pressure on the function
lower limbs C. Paraplegia with intercostal muscle loss
D. Loss of bowel and bladder control
A. High doses of Solu-Medrol are used within
24 hours of spinal injury to reduce cord
A client with a spinal cord injury at levels C5 swelling and limit neurological deficit. The
to C6 has quadriplegia with gross arm other drugs aren't indicated in this
movement and diaphragmatic breathing. circumstance
Injury levels C1 to C4 leads to quadriplegia
with total loss of respiratory function. The nurse is caring for a client with a T5
Paraplegia with intercostal muscle loss complete spinal cord injury. Upon
occurs with injuries at T1 to L2. Injuries assessment, the nurse notes flushed skin,
below L2 cause paraplegia and loss of diaphoresis above the T5, and a blood
bowel and bladder control. pressure of 162/96. The client reports a
severe, pounding headache. Which of the
A 30-year-old was admitted to the following nursing interventions would be
progressive care unit with a C5 fracture appropriate for this client? Select all that
from a motorcycle accident. Which of the apply.
following assessments would take priority?
A. Elevate the HOB to 90 degrees
A. Bladder distension B. Loosen constrictive clothing
B. Neurological deficit C. Use a fan to reduce diaphoresis
C. Pulse ox readings D. Assess for bladder distention and bowel
D. The client's feelings about the injury impaction
C. Pulse ox readings E. Administer antihypertensive medication
A, B, D, E
After a spinal cord injury, ascending cord
edema may cause a higher level of injury. The client has signs and symptoms of
The diaphragm is innervated at the level of autonomic dysreflexia. The potentially life-
C4, so assessment of adequate threatening condition is caused by an
oxygenation and ventilation is necessary. uninhibited response from the sympathetic
Although the other options would be nervous system resulting from a lack of
necessary at a later time, observation for control over the autonomic nervous system.
respiratory failure is the priority. The nurse should immediately elevate the
HOB to 90 degrees and place extremities
A client is admitted with a spinal cord injury dependently to decrease venous return to
at the level of T12. He has limited the heart and increase venous return from
movement of his upper extremities. Which the brain. Because tactile stimuli can trigger
of the following medications would be used autonomic dysreflexia, any constrictive
to control edema of the spinal cord? clothing should be loosened. The nurse
should also assess for distended bladder
A. Acetazolamide (Diamox) and bowel impaction, which may trigger
B. Furosemide (Lasix) autonomic dysreflexia, and correct any
C. Methylprednisolone (Solu-Medrol) problems. Elevated blood pressure is the
D. Sodium bicarbonate most life-threatening complication of
C. Solu-Medrol autonomic dysreflexia because it can cause
stroke, MI, or seizures. If removing the front to back in a woman, or in expanding
triggering event doesn't reduce the client's circles working outward from the meatus in
blood pressure, IV antihypertensives should a man. It isn't necessary to measure the
be administered. A fan shouldn't be used urine. The catheter doesn't need to be
because cold drafts may trigger autonomic rotated during removal.
dysreflexia.
While in the ER, a client with C8 tetraplegia
The nurse is caring for a client who suffered develops a blood pressure of 80/40, pulse
a spinal cord injury 48 hours ago. The nurse 48, and RR of 18. The nurse suspects
monitors for GI complications by assessing which of the following conditions?
for:
A. Autonomic dysreflexia
A. A flattened abdomen B. Hemorrhagic shock
B. Hematest positive nasogastric tube C. Neurogenic shock
drainage D. Pulmonary embolism
C. Hyperactive bowel sounds C. Neurogenic shock
D. A history of diarrhea
B. Hematest positive NG drainage Symptoms of neurogenic shock include
hypotension, bradycardia, and warm, dry
After spinal cord injury, the client can skin due to the loss of adrenergic
develop paralytic ileus, which is stimulation below the level of the lesion.
characterized by the absence of bowel Hypertension, bradycardia, flushing, and
sounds and abdominal distention. sweating of the skin are seen with
Development of a stress ulcer can be autonomic dysreflexia. Hemorrhagic shock
detected by hematest positive NG tube presents with anxiety, tachycardia, and
aspirate or stool. A history of diarrhea is hypotension; this wouldn't be suspected
irrelevant. without an injury. Pulmonary embolism
presents with chest pain, hypotension,
A 40-year-old paraplegic must perform hypoxemia, tachycardia, and hemoptysis;
intermittent catheterization of the bladder. this may be a later complication of spinal
Which of the following instructions should cord injury due to immobility.
be given?
The nurse is planning care for the client in
A. "Clean the meatus from back to front." spinal shock. Which of the following actions
B. "Measure the quantity of urine." would be least helpful in minimizing the
C. "Gently rotate the catheter during effects of vasodilation below the level of the
removal." injury?
D. "Clean the meatus with soap and water."
D. Soap and Water A. Monitoring vital signs before and during
position changes
Intermittent catheterization may be B. Using vasopressor medications as
performed chronically with clean technique, prescribed
using soap and water to clean the urinary C. Moving the client quickly as one unit
meatus. The meatus is always cleaned from D. Applying Teds or compression stockings
C. Moving the client quickly as one unit A client with a cervical spine injury has
Gardner-Wells tongs inserted for which of
Reflex vasodilation below the level of the the following reasons?
spinal cord injury places the client at risk for
orthostatic hypotension, which may be A. To hasten wound healing
profound. Measures to minimize this include B. To immobilize the cervical spine
measuring vital signs before and during C. To prevent autonomic dysreflexia
position changes, use of a tilt-table with D. To hold bony fragments of the skull
early mobilization, and changing the client's together
position slowly. Venous pooling can be B. Immobilize cervical spine
reduced by using Teds (compression
stockings) or pneumatic boots. Vasopressor Gardner-Wells, Vinke, and Crutchfield tongs
medications are administered per protocol. immobilize the spine until surgical
stabilization is accomplished
A 22-year-old client with quadriplegia is
apprehensive and flushed, with a blood The nurse is evaluating neurological signs
pressure of 210/100 and a heart rate of 50 of the male client in spinal shock following
bpm. Which of the following nursing spinal cord injury. Which of the following
interventions should be done first? observations by the nurse indicates that
spinal shock persists?
A. Place the client flat in bed
B. Assess patency of the indwelling urinary A. Positive reflexes
catheter B. Hyperreflexia
C. Give one SL nitroglycerin tablet C. Inability to elicit a Babinski's reflex
D. Raise the head of the bed immediately to D. Reflex emptying of the bladder
90 degrees C. Babinski's reflex
D. Raise HOB 90 degrees
Resolution of spinal shock is occurring
Anxiety, flushing above the level of the when there is a return of reflexes (especially
lesion, piloerection, hypertension, and flexors to noxious cutaneous stimuli), a
bradycardia are symptoms of autonomic state of hyperreflexia rather than flaccidity,
dysreflexia, typically caused by such reflex emptying of the bladder, and a
noxious stimuli such as a full bladder, fecal positive Babinski's reflex.
impaction, or decubitus ulcer. Putting the
client flat will cause the blood pressure to
increase even more. The indwelling urinary
catheter should be assessed immediately
after the HOB is raised. Nitroglycerin is
given to reduce chest pain and reduce
preload; it isn't used for hypertension or
dysreflexia.

You might also like